Você está na página 1de 149

INTRODUCCIÓN

Los métodos de resolver ecuaciones de segundo grado por ejemplo a la ecuación


x 2  49  0 nos llevan a encontrar raíces “imposibles” o “imaginarias” en el campo
de los números reales. Es un hecho conocido que el cuadrado de un número real
no puede ser nunca negativo; en nuestro caso, x 2  49 representa una
contradicción evidente en el campo de los números reales. La existencia de tales
ecuaciones y el estudio de sus soluciones o raíces imaginarias nos llevan a
introducir los números complejos.

A semejanza con el estudio, desarrollado en forma axiomática, de los números


reales, daremos el mismo enfoque al estudiar los números complejos.

NÚMEROS COMPLEJOS
La definición de números complejos está ligada a la noción de pares ordenados de
números reales. Recordamos que dos pares ordenados de números reales (a;b) y
(c;d) son iguales si sus respectivos elementos son iguales; es decir.

(a;b)  (c;d)  a  c y b  d

Sea C  (a;b) / a  y b  el conjunto de todos los pares ordenados de


números reales, definiremos a continuación dos operaciones sobre C:

ADICIÓN DE PARES ORDENADOS


La adición de pares ordenados se ejecuta sumando los primeros elementos y los
segundos respectivamente, así:
(a;b)  (c;d)  (a  c;b  d)

MULTIPLICACIÓN DE PARES ORDENADOS


Si se tiene los pares ordenados (a;b) y (c;d) , la multiplicación de éstos se ejecuta
así:
(a;b)  (c;d)  (ac  bd;ad  bc)
Definición: El conjunto C y las operaciones de adición y multiplicación de pares
ordenados, definidos anteriormente, se denomina el sistema de los números
complejos.
Notación: utilizaremos las letras w, z, etc. Para representar los números
complejos (pares ordenados de números reales), de modo que se tenga en mente
lo siguientes:
z ϵ C si solo sí z = (a ; b), a ; b ϵ ℝ
w ϵ C si solo sí w = (x ; y), x ; y ϵ ℝ
PROPIEDADES:

Para todo z1;z2;z3 perteneciente a los complejos se cumple:

A1 : z1 + z2 es complejo (ley de clausura para la adicción)


A 2 : z1 + z2 = z2 + z1 (ley conmutativa para la adición)
A 3 : z1 +(z2 + z3 ) = (z1 + z2 )+ z3 (ley asociativa para la adición)
A 4 : Existe un único elemento r  , de la forma r = (0;0) , tal que z +r = z para
todo complejo z. (Existencia del elemento aditivo).
A 5 : existe un único elemento  z  , de la forma -z = (-a;-b) , tal que
z +(-z) = r = (0;0) para todo complejo z. (Existencia del elemento inverso aditivo).
M1 : z1  z2 Es un complejo (ley de clausura para la multiplicación)
M2 : z1  z2 = z2  z1 (ley conmutativa para la multiplicación)
M3 : (z1  z2 )  z3 = z1  (z2  z3 ) (ley asociativa para la multiplicación)
M4 : z1(z2 + z3 ) = z1z2 + z1z3 (ley distributiva)
M5 : existe un único elemento w  , de la forma w = (1;0) , tal que z  w = z para
toda z complejo. (Existencia del elemento neutro multiplicativo).
M6 : existe un único elemento z1  , tal que z  z1  z1  z  (1;0) , para todo z en
y z  (0;0) . (Existencia del elemento inverso multiplicativo).
La demostración de estas propiedades se hace en base a los axiomas de los
números reales. Demostraremos únicamente ; las demás quedan como ejercicio
para el lector.

Demostración de A 2

Sean: z1  (a;b) y z2  (c;d) , tal que a; b; c y d ϵ ℝ

z2 + z1  (a;b)  (c;d)  (a  c;b  d)

 (c  a;d  b) ; (ley conmutativa en ℝ)

 (c;d)  (a;b)  z2 + z1
Demostración de M3

Sean: z1  (a ;b) , z2  (c ;d) y z3  (x ;y) , tal que a, b, c, d, x e y ϵ .

 (z1  z2 )  (ac  bd,ad  bc)  (x,y)


 (ac  bd)x  (ad  bc)y ;(ac  bd)y  (ad  bc)x 
 (acx  bdx  ady  bcy ;acy  bdy  adx  bcx)
 (acx  ady  bdx  bcy ;acy  adx  bdy  bcx)
 a(cx  dy)  b(dx  cy) ;a(cy  dx)  b(dy  cx)
 (a;b)  (cx  dy;cy  dx)
 (a;b)  (c;d)  (x;y)
 z1  (z2  z3 )

PARTE REAL E IMAGINARIA DE UN NÚMERO COMPLEJO

Si se tiene el numero complejo z=(a ; b), entonces al número real a se denomina


parte real, y al número real b parte imaginaria del mundo complejo.
Notación:
a  Re(z)
b  Im(z)

LOS NÚMEROS REALES COMO SUBCONJUNTO DE LOS COMPLEJOS

Los números z = (a; 0) forman un subconjunto especial de , el cual es


(a ;0) / a   . Se puede establecer una correspondencia, uno a uno, entre este
subconjunto y el conjunto de los números reales, a saber:

 Para todo z = (a ; 0) le hacemos corresponder el real a.


 Además se verifica en este subconjunto de , las operaciones de adición y
multiplicación de pares ordenados. Veamos:
(a ; 0) + (b ; 0) = (a+b ; 0), le corresponde el real a+ b

(a ; 0).(b ; 0) = (ab ; 0), le corresponde el real ab.


Nota: Se ha tenido en cuenta lo siguiente:

Si al par ordenado (a ; 0) le corresponde el número real a y al par ordenado (b ; 0)


le corresponde el número real b; entonces (a ; 0) + (b ; 0) le corresponde número
real a + b.
Es decir, al número complejo (a+b; 0) le corresponde el número real a + b: en
forma similar se verifica para el producto.
De donde se deduce que los números complejos de la forma (a; 0) se comportan,
con respecto a la adición y multiplicación, exactamente igual que los números
reales. Por esta razón, es usual no hacer distinción entre (a; 0), y a es decir
(a ;0)  a , que lo tomaremos como (a ;0)  a .
Por lo tanto, ℝ es subconjunto de .
En adelante podremos representar a un número real como un numero complejo,
cuya parte imaginaria es igual a cero.

FORMA CARTESIANA DE UN NÚMERO COMPLEJO – EL NÚMERO “ i ”


Denotaremos el numero complejo (0 ; 1) por “ i ”, es decir i = (0; 1). Luego:

i 2 = i. i = (0 ; 1).(0 ; 1) = (-1 ; 0) = -1  i= 1
OBSERVACIONES:

1. la representación de i = 1 se debe a Euler. Así se le denomina la unidad


imaginaria.

2. La ecuación z 2  1  0 , que no tiene solución en los reales, tiene en las


soluciones z   i, las cuales se han encontrado de la siguiente manera:
z2  1  0  z 2  1  z   1  z   i

3. Consideramos, ahora un numero complejo cualquiera z = (a ; b). Como:


z = (a ; b) = (a ; 0) + (0 ; b)
Y por otro lado:
(a ; 0) = a y (0 ; b) lo escribimos como (0 ; b) = (0 ; 1).(b ; 0), donde
(0 ; 1) = i y (b ; 0) = b.

Entonces ocurre:
z  (a;b)  (a;0)  (0;b)
 (a;0)  (0;1)  (b;0)
 a  ib
Luego, z = (a ; b) se puede representar como:
z  a  ib

Que se le conoce como la forma cartesiana de un número complejo, donde:


a = Re(z) , b = Im(z)
4. La igualdad de 2 números complejos se reduce a comprobar la la igualdad
de la parte real y la imaginaria , respectivamente; es decir, si:
z1 = (a ; b) y z 2 = (c ; d), luego si z1  z2 entonces: a = c y b = d
En forma equivalente: a + ib = c + id  a = c y b = d.
5. El numero r = (0 ; 0) que representa el cero complejo, se denota en su
forma cartesiana por (0 ; 0) = 0 + i0, le corresponde el número real cero. De
igual modo, para el número w = (1 ; 0) = 1+ 0i, le corresponde el número
real 1.
6. Un número complejo se dice que es puramente real, si la parte imaginaria
es igual a cero. Su representación es: z = (0 ; b) = 0 + ib, donde Re(z) = 0.
Nota: observe que a los números de esta forma les corresponde el número
real, ya que: z1 = (a ; b) = a + i 0 = a
7. Un número complejo se dice que es imaginario puro, si la parte real es igual
a cero. Su representación es: z = (0 ; b) = 0 + ib, donde Re(z) = 0.
REPRESENTACIÓN GEOMÉTRICA DE UN NÚMERO COMPLEJO
Como z = (a ; b) es un par ordenado, la representación geométrica de un numero
complejo es la representación usual de un par ordenado.
Eje
Y
imaginario
b ● P = (a ; b) El punto P representa a:

z1 = (a ; b) = a + bi

X
0 a Eje real

La correspondencia entre puntos del plano y los números complejos es biunívoco;


es decir, a cada punto del plano le corresponde un numero complejo, y viceversa,
a cada número complejo le corresponde un punto en el plano. Por esta razón se
suele identificar con el plano XY, al cual se le denomina plano complejo.
En el plano complejo, el eje X recibe el nombre de eje real (están representados
sobre este eje, los números puramente reales); y al eje Y se le denomina eje
imaginario (sobre él se representa los números imaginarios puros ib, con b real).

2
DIFERENCIA ENTRE NÚMEROS COMPLEJOS Y VECTORES EN
En los números complejos también se puede realizar la operación multiplicación
de un escalar (número real) por un numero complejo. En efecto, teniendo en
cuenta que t = (t ; 0), se tiene:
tz = t(a ; b) = (t ; 0).(a ; a) = (ta – 0b ; tb + 0a) = (ta ; tb), con t real.
En forma equivalente:

𝑡𝑧 = 𝑡(𝑎; 𝑖𝑏) = 𝑡𝑎 + 𝑖𝑡𝑏 = (𝑡𝑎; 𝑡𝑏)


De donde resulta que multiplicar un numero complejo por un escalar, es la misma
operación que multiplicar un escalar por un vector en R2. Este hecho motiva la
siguiente interrogante:¿serán iguales los números complejos a los vectores
bidimensionales?.

La respuesta es no. En los números complejos, a diferencia de los vectores, hay


definida la operación de multiplicación de pares ordenados (números complejos),
la cual por la ley de clausura nos da un nuevo par ordenado. Esto motiva que en
los números complejos también se pueda definir la operación inversa a la
multiplicación, a diferencia de los vectores en donde no existe la multiplicación y
su operación inversa.

Además geométricamente, un vector representa una dirección en el plano ,


mientras que un número complejo nos da una ubicación en el plano referida
siempre, al origen del sistema de coordenadas.
Si bien es cierto que existen grandes semejanzas entre los vectores en R2 y los
números complejos, no hay que confundir estos dos conjuntos de pares
ordenados, que tienen naturaleza cualitativamente diferente.

OPERACIONES CON NUMEROS COMPLEJOS


DIFERENCIA Y COCIENTE

 Dado dos números complejos 𝑥 = (𝑎; 𝑏)𝑦 𝑣 = (𝑐; 𝑑), el numero 𝑧 solución de
la ecuación 𝑢 + 𝑧 = 𝑦 se llama la diferencia de 𝑣 menos 𝑢.
Notación: 𝑧 = 𝑣 − 𝑢 = (𝑐 − 𝑎; 𝑑 − 𝑏) , en la forma cartesiana:
𝑧 = (𝑐 + 𝑖𝑑) − (𝑎 + 𝑖𝑏) = (𝑐 − 𝑎) + 𝑖(𝑑 − 𝑏)

 Dado dos números complejos 𝑢 = (𝑎; 𝑏) y 𝑣 = (𝑐; 𝑑), 𝑣 ≠ 0,el numero


complejo 𝑧
Solución de 𝑢 = 𝑣𝑧se llama el cociente de 𝑢⁄𝑣.
Veamos cómo se calcula dicha solución 𝑧:
Reemplazando 𝑢, 𝑣, 𝑧 en 𝑢 = 𝑣𝑧 tenemos:
𝑎 + 𝑖𝑏 = (𝑐 + 𝑖𝑑)(𝑥 + 𝑖𝑦) = (𝑐𝑥 − 𝑑𝑦) + 𝑖(𝑐𝑦 + 𝑑𝑥)
Igualando las partes real e imaginaria, queda:

𝑐𝑥 − 𝑑𝑦 = 𝑎

𝑐𝑦 + 𝑑𝑥 = 𝑏

Resolviendo para 𝑥 e 𝑦:
𝑎𝑐+𝑏𝑑 𝑏𝑐−𝑎𝑑
𝑥= ; 𝑥=
𝑐+𝑑 2 𝑐 2 +𝑑 2

Luego,
𝑢 𝑎+𝑖𝑏 𝑎𝑐+𝑏𝑑 𝑏𝑐−𝑎𝑑
𝑧= = =( ) + ( 𝑐 2+𝑑2 ) 𝑖
𝑣 𝑐+𝑖𝑏 𝑐 2 +𝑑 2

OBSERVACIONES:

1. Nótese que la condición 𝑣 = 𝑐 + 𝑖𝑑 ≠ 0 es equivalente a la condición 𝑐 2 +𝑑 2 > 0


2. El uso de la notación 𝑧 = 𝑥 + 𝑖𝑦 simplifica las operaciones definidas en el campo
de los números complejos. De aquí en adelante adoptaremos esta
representación , con la cual las operaciones anteriores se pueden enunciar
como:

Adición: (𝑎 + 𝑖𝑏) + (𝑐 + 𝑖𝑑) = (𝑎 + 𝑐) + 𝑖(𝑏 + 𝑑)


Diferencia: (𝑎 + 𝑖𝑏) − (𝑐 + 𝑖𝑑) = (𝑎 − 𝑐) + 𝑖(𝑏 − 𝑑)
Multiplicación: (𝑎 + 𝑖𝑏). (𝑐 + 𝑖𝑏) = (𝑎𝑐 − 𝑏𝑑) + 𝑖(𝑎𝑑 + 𝑏)
(𝑎+𝑖𝑏) (𝑎𝑐+𝑏𝑑)
Cociente: = , siempre que 𝑐 2 + 𝑑 2 > 0.
(𝑐+𝑖𝑑) (𝑐 2 𝑑 2 )

Ejemplos 1. Hallar 𝑧 = (4 + 3𝑖). (4 + 2𝑖). (1 − 𝑖)

Solución:
Como la multiplicación de números complejos tiene la propiedad asociativa, no
interesa
El orden en que se empiecen a multiplicar los factores. Luego, se tiene:

𝑧 = (4 + 3𝑖). (4 + 2𝑖). (3 − 𝑖)(1 − 𝑖) = [(16 − 6) + 𝑖(8 + 12)]. (3 − 𝑖)(1 − 𝑖)

= (10 + 20𝑖). (3 − 𝑖). (1 − 𝑖)

= [(30 − (−20)) + 𝑖(60 − 10)]. (1 − 𝑖)

= (50 − 50𝑖). (1 − 𝑖)

= 100 + 0𝑖 = 100
Ejemplo 2. Hallar 𝑧 = 𝑖 ⁄[(𝑖 − 1). (𝑖 − 2). (𝑖 − 3)]

Solución:
𝑖 (0+1)
𝑧 = [(𝑖−1)(𝑖−2)(𝑖−3)] =
(−1+𝑖)(−2+𝑖)(−3+𝑖)

(0+𝑖)
= [(2−1)+(−1−2)𝑖](−3+1)

(0+𝑖)
=
(1−3𝑖)(−3+𝑖)

(0+𝑖)
= [(−3−(−3)+(1+9)𝑖]

(0+𝑖)
=
(0+10𝑖)

(0+10) 𝑖(0−0)
= +
02 +102 02 +102
10
= + 0𝑖
100

= 1⁄10
Observaciones:

1. El último ejemplo nos indica que al hacer el cociente de dos cantidades


imaginarias puras, la unidad imaginaria 𝑖puede ser considerada como un factor,
tanto en el numerador como en el denominador, y por lo tanto puede
simplificarse. En efecto, a partir de (𝛼)

0+𝑖 𝑖 1
𝑧= = =
0+10𝑖 10𝑖 10

Que el resultado obtenido anteriormente.

2. La multiplicación de números complejos se simplifica si se tiene en cuenta que


dicha operación cumple las leyes del algebra para la multiplicación de binomios
en efecto:

𝑧 = (3 + 2𝑖). (2 + 5𝑖) = (6 − 10) + (15 + 4)𝑖 = −4 + 19𝑖


Este mismo resultado se logra si se dispone la multiplicación como:

3+2𝑖×
2+5𝑖
6+4𝑖
15𝑖+10𝑖 2
6+19𝑖+10𝑖 2
Pero como 𝑖 2 = −1,entonces:
𝑧 = 6 + 19𝑖 + 10𝑖 2 = 6 + 19𝑖 − 10 = −4 + 19𝑖

Ejemplo 3. Si 𝑤 = (1 + 𝑧)(1 − 𝑧), donde 𝑧 = 𝑐𝑜𝑠𝑎 + 𝑖𝑠𝑒𝑛𝑎; ℎ𝑎𝑙𝑙𝑎𝑟 𝑤.


Solución:
Reemplazando 𝑧 en 𝑤, tiene:

(1+𝑐𝑜𝑠 𝑎+𝑖𝑠𝑒𝑛 𝑎) [(1+𝑐𝑜𝑠 𝑎)+𝑖𝑠𝑒𝑛 𝑎]


𝑤 = [1−(𝑐𝑜𝑠 = [1−𝑐𝑜𝑠 𝑎)−𝑖 𝑠𝑒𝑛 𝑎]
𝑎+𝑖 𝑠𝑒𝑛 𝑎)]

Luego, aplicando la fórmula del cociente:


1+cos 𝑎)(1−cos 𝑎)+(𝑠𝑒𝑛 𝑎)(−𝑠𝑒𝑛 𝑎) (𝑠𝑒𝑛 𝑎)(1−cos 𝑎)−(1+cos 𝑎)(−𝑠𝑒𝑛 𝑎)
𝑤= +
(1−cos 𝑎)2 +(−𝑠𝑒𝑛 𝑎)2 (1−cos 𝑎)2 +(−𝑠𝑒𝑛 𝑎)2

1−𝑐𝑜𝑠 2 𝑎−𝑠𝑒𝑛2 𝑎 𝑠𝑒𝑛 𝑎−𝑠𝑒𝑛 𝑎.cos 𝑎+𝑠𝑒𝑛 𝑎+𝑠𝑒𝑛 𝑎.cos 𝑎


= +
1−2 cos 𝑎+𝑐𝑜𝑠2 𝑎+𝑠𝑒𝑛2 𝑎 1−2 cos 𝑎+𝑐𝑜𝑠 2 𝑎+𝑠𝑒𝑛2 𝑎
0 2𝑠𝑒𝑛 𝑎
= + 𝑖
2(1−𝑐𝑐𝑜𝑠 𝑎) 2(1−cos 𝑎)

𝑠𝑒𝑛 𝑎
= 𝑖
1−cos 𝑎

Ejemplo 4. Si 𝑧 = √2(cos 𝑎 + 𝑖 𝑠𝑒𝑛 𝑎)) cos 𝑏 + 𝑖 𝑠𝑒𝑛 𝑏), donde: 0 < 𝑎 <
𝜋⁄2; 0 < 𝑏 <; 𝜋⁄2

𝑎 = 𝑎𝑟𝑐 𝑡𝑔 2, 𝑏 = 𝑎𝑟𝑐 𝑡𝑔 3; ℎ𝑎𝑙𝑙𝑎𝑟 𝑧.

Solución:
Del enunciado se tiene:
𝜋 2√5 √5
Si 𝑎 = 𝑎𝑟𝑐 𝑡𝑔 2 ⇒ 2 = 𝑡𝑔 𝑎 ⇒ 0 < 𝑎 < 2 ; luego: 𝑠𝑒𝑛 = , cos 𝑎
5 5

𝜋 3√10 √10
Si 𝑏 = 𝑎𝑟𝑐 𝑡𝑔 3 ⇒ 3 = 𝑡𝑔 𝑏 ⇒ 𝑜 < 𝑏 < 2 ; luego: 𝑠𝑒𝑛 𝑏 = , cos 𝑏 =
10 10
Reemplazando en la ecuación dada:

𝑧 = √2(cos 𝑎 + 𝑖 𝑠𝑒𝑛 𝑎)(cos 𝑏 + 𝑖 𝑠𝑒𝑛 𝑏)


= [8 cos 𝑎. cos 𝑏 − 𝑠𝑒𝑛 𝑎. 𝑠𝑒𝑛 𝑏) + 𝑖(𝑠𝑒𝑛 𝑎. 𝑐𝑜𝑠𝑏 + cos 𝑎. 𝑠𝑒𝑛 𝑏)]
√5 √10 2√5 3√10 2√5 √10 √5 3√10
= √2 [( × − × ) +( × + × ) 𝑖]
5 10 5 10 5 10 5 10

√50 6√50 √50 3√50


= √2 [( − ) +( + ) 𝑖]
50 50 25 50

5√50 5√50
= √2 [− + 𝑖]
50 50

√50 √50
= √2 [− + 𝑖]
10 10
10 10
=− + 𝑖
10 10

= −1 + 𝑖

Ejemplo 5. ¿Para qué puntos del plano se cumple que 𝑅(𝑧) = 𝐼𝑚(𝑧)?

Solución:
Si 𝑧 = 𝑎 + 𝑏𝑖, entonces 𝑅𝑒(𝑧) = 𝑎 y 𝐼𝑚(𝑧) = 𝑏.
De la condición del enunciado 𝑅𝑒(𝑧) = 𝐼𝑚(𝑧), se tiene 𝑎 = 𝑏. Luego:
𝑧 = 𝑎 + 𝑎𝑖 = 𝑎(1 + 𝑖) , para todo"𝑎" real.

Nota: Los puntos del plano que satisfacen lo anterior son de la forma:

𝑧 = 𝑎(1 + 𝑖) = (𝑎; 𝑎), donde 𝑎 ∈ 𝑹

Pues al número complejo 𝑧 le corresponde el punto 𝑧 del plano, y viceversa.


Observando los puntos 𝑧 = (𝑎; 𝑎), se deduce que dichos puntos están sobre la
recta identidad 𝑦 = 𝑥 lo cual puede comprobarse también, dando valores
diferentes al numero 𝑎.
CONJUGADO DE UN NUMERO COMPLEJO

El conjunto de un número complejo 𝑧 = (𝑎; 𝑏) = 𝑎 + 𝑏𝑖, denotado por 𝑧̅, se define


por 𝑧 = (𝑎; −𝑏), que geométricamente representa el punto simétrico de 𝑧 respecto
al eje real.

Y
b z=(a; b)

a X

-b z=(a; -b)
Ejemplo: Dado 𝑧 = 3 + 5𝑖, entonces el conjugado de 𝑧 de 𝑧 = 3 − 5𝑖.

PROPÍEDADES. Para todo 𝑧1 ; 𝑧2 complejos, se cumple:

1. 𝑧𝟏 +𝑧2 = 𝑧1 + 𝑧2 5. (𝑧) = 𝑧
𝑧+𝑧
2. 𝑧1 −𝑧2 = 𝑧1 − 𝑧2 6. 𝑅𝑒(𝑧) = 2

3. 𝑧1 . 𝑧2 = 𝑧1 . 𝑧2 7. 𝐼𝑚(𝑧) = (𝑧 − 𝑧 )/2𝑖

𝑧 𝑧
4. (𝑧1 ) = 𝑧1 , siempre que 𝑧2 ≠ 0 8. 𝑧 es un número real si solo si 𝑧 = 𝑧
2 2

La demostración de estas propiedades es inmediata, a partir de la definición de


números conjugado. Demostraremos las propiedades 2,2 y 6; las demás quedan
como ejercicio.
DEMOSTRACIONES:

Sean: 𝑧1 = (𝑎; 𝑏)𝑦 𝑧2 = (𝑐; 𝑑), donde 𝑎; 𝑏; 𝑐 y 𝑑 ∈ 𝑹

2. 𝑧1 + 𝑧2 = (𝑎 + 𝑐; 𝑏 + 𝑑)

⇒𝑧1 +𝑧2 =[𝑎 + 𝑐; −𝑐 − 𝑑]

= (𝑎; −𝑏) + (𝑐; −𝑑)

= 𝑧1 + 𝑧2
3. 𝑧1 . 𝑧2=(𝑎𝑐−𝑏𝑑;𝑎𝑑+𝑏𝑐)

⇒𝑧1 . 𝑧2 = [𝑎𝑐 − 𝑏𝑑; −(𝑎𝑑 + 𝑏𝑐)]

= (𝑎𝑐 − 𝑏𝑑; −𝑎𝑑 − 𝑏𝑐)

= (𝑎; −𝑏). (𝑐; 𝑑)

= 𝑧1 .𝑧2

6. sea 𝑧 = (𝑎; 𝑏) ⇒ 𝑧 = (𝑎 ; −𝑏), luego:


1 1
(𝑧 + 𝑧) = [(𝑎; 𝑏) + (𝑎; −𝑏)]
2 2

1
= 2 (2𝑎; 0)

= (𝑎: 0)

=𝑎

= 𝑅𝑒(𝑧)
1
⇒𝑅𝑒(𝑧) = 2 (𝑧 + 𝑧)

MODULO DE UN NUMERO COMPLEJO

Dado 𝑧 = 𝑎 + 𝑏𝑖 , el módulo de 𝑧 es un número real denotado por |𝑧|, que se


define por la siguiente relación:

|𝑧| = √𝑎2 +𝑏 2

Ejemplo. Dado 𝑧 = −3 − 4𝑖, se sigue de la definición que:


|𝑧| = √[(−3)2 +(−4)2 = 5
OBSERVACIONES:
1. El módulo de un complejo siempre es una cantidad positiva.
2. Geométricamente, el modulo nos representa la magnitud del radio vector de
origen (0,0) y extremo final al punto cuyas coordenadas representan a z.
Y
b z = (a;b)

ΙzΙ

0 a X

PROPIEDADES

Para todo 𝑧 ;𝑧1 ;𝑧2 complejo se cumple:

1. |𝑧| ≥ 0 ; |𝑧| = 0 Si solo si 𝑧 = 0


2. |𝑅𝑒(𝑧)| ≤ |𝑧| 𝑦 |𝐼𝑚(𝑧)| ≤ |𝑧|
3. |𝑧̅| = |−𝑧| = |𝑧|
4. |𝑧 2 | = 𝑧. 𝑧̅
5. |𝑧1 + 𝑧2 | ≤ |𝑧1 | + |𝑧2 | ( Desigual Triangular )
6. |𝑧1 . 𝑧2 | = |𝑧1 ||𝑧2 |
𝑧1 |𝑧1 |
7. | | = , siempre que 𝑧2 ≠ 0
𝑧2 |𝑧2 |

Demostraciones:
Sea 𝑧 = (𝑎; 𝑏) ⇒ 𝑧̅ = (𝑎; −𝑏), donde 𝑎; 𝑏 ∈ 𝑅
2. Demostraciones la primera parte: |𝑅𝑒(𝑧)| ≤ |𝑧|
como 𝑏 ∈ 𝑅 ⇒ 𝑏 2 ≥ 0 ⇒ 𝑎2 + 𝑏 2 ⇔ 𝑎2 ≤ 𝑎2 + 𝑏 2

⇔ −√𝑎2 + 𝑏 2 ≤ 𝑎 ≤ √𝑎2 + 𝑏 2

⇔ |𝑎| ≤ √𝑎2 + 𝑏 2 …(1)

Pero,𝑅𝑒(𝑧) = 𝑎 ≤ |𝑧| = √𝑎2 + 𝑏 2

Luego; en (1):
|𝑅𝑒(𝑧)| ≤ |𝑧|

En forma similar se demuestra que |𝐼𝑚(𝑧)| ≤ |𝑧|


4. Si 𝑧 = (𝑎; 𝑏), entonces:

|𝑧| 2 = 𝑎2 + 𝑏 2

= (𝑎2 + 𝑏 2 ) + 𝑖(𝑎𝑏 − 𝑎𝑏)

= 𝑎2 + 𝑏 2 + 𝑖𝑎𝑏 − 𝑖𝑎𝑏

= 𝑎2 + 𝑖𝑎𝑏 + 𝑏 2 − 𝑖𝑎𝑏

= 𝑎(𝑎 + 𝑖𝑏) + 𝑏(𝑏 − 𝑖𝑎)

= 𝑎(𝑎 + 𝑖𝑏) − 𝑏(𝑖𝑎 − 𝑏)

= 𝑎(𝑎 + 𝑖𝑏) − 𝑏(𝑖𝑎 + 𝑖 2 𝑏)

= 𝑎(𝑎 + 𝑖𝑏) − 𝑏𝑖(𝑎 + 𝑖𝑏)

= (𝑎 + 𝑖𝑏)(𝑎 − 𝑖𝑏)

= 𝑧. 𝑧̅

⇒ |𝑧| 2 = 𝑧. 𝑧̅

OBSERVACIONES:
1. La propiedad (4) tiene mucha importancia para simplificar la operación de
división
En efecto, se tenía que:
𝑎+𝑏𝑖 𝑎𝑐+𝑏𝑑 𝑏𝑐−𝑎𝑑
= + 𝑖 … . . (∝)
𝑐+𝑑𝑖 𝑐 2 +𝑑 2 𝑐 2 +𝑑 2
Si observamos los denominadores en los sumandos del 2do. miembro se
tiene :
𝑐 2 + 𝑑 2 = (√𝑐 2 + 𝑑 2 ) 2 = |𝑐 + 𝑑𝑖| 2
Vemos que resulta el modulo al cuadrado del divisor. Teniendo en cuenta
la propiedad (4) se sugiere el siguiente artificio:
𝑎+𝑏𝑖 𝑐−𝑑𝑖 (𝑎𝑐+𝑏𝑑)+(𝑏𝑐−𝑎𝑑)𝑖
× =
𝑐+𝑑𝑖 𝑐−𝑑𝑖 𝑐 2 +𝑑 2
𝑎𝑐+𝑏𝑑 𝑏𝑐−𝑎𝑑
= + 𝑖
𝑐 2 +𝑑 2 𝑐 2 +𝑑 2
Con lo cual se obtiene el resultado de (∝). Luego, la operación de división
o cociente de complejos se reduce a multiplicar al dividendo y al divisor por
el conjugado del divisor.
3−5𝑖
Ejemplo: Si 𝑧 = , entonces:
5−3𝑖

3−5𝑖 5−3𝑖
𝑧= ×
5−3𝑖 5−3𝑖
15−(−15) (−25+9)
= + 𝑖
52 +32 52 +32
30 16
= − 𝑖
34 34
15 8
= − 𝑖
17 17

2. La distancia entre 2 puntos, o sea entre 2 complejos, 𝑧1 = 𝑎 + 𝑏𝑖 y 𝑧2 = 𝑐 +


𝑑𝑖 se define como:

|𝑧2 − 𝑧1 | = |(𝑐 − 𝑎) + 𝑖(𝑑 − 𝑏)| = √(𝑐 − 𝑎) 2 + (𝑑 − 𝑏) 2

Geométricamente:

z ₂=(c;d)
(0;d)
z₁Ι
Ιz₂-
(0;b)
z₁=(a;b)

0
(a;0) (c;0) X
CONJUNTOS ESPECIALES DE NUMEROS COMPLEJOS

Por medio de la noción de modulo describiremos algunos conjuntos de complejos.

1. 𝐴 = {𝑧/|𝑧 − 𝑧0 | = 𝑟 ; 𝑟 > 0 ; 𝑧0 fijo }

Nos describe el lugar geométrico de todos los puntos 𝑧 a una distancia 𝑟 del
punto 𝑧0 . Es decir, 𝐴 es la circunferencia del centro 𝑧0 y radio 𝑟 . La
demostración de que el lugar geométrico |𝑧 − 𝑧0 | = 𝑟 representa una
circunferencia, es sencilla. Veamos:

Sean 𝑧 = (𝑥; 𝑦) y 𝑧0 = (𝑥0 ; 𝑦0 ) , entonces:

𝑧 − 𝑧0 = (𝑥 − 𝑥0 ; 𝑦 − 𝑦0 )

Luego:

𝑟 = |𝑧 − 𝑧0 | = √(𝑥 − 𝑥0 ) 2 + (𝑦 − 𝑦0 ) 2

⇔ (𝑥 − 𝑥0 ) 2 + (𝑦 − 𝑦0 ) 2 = 𝑟 2 ,

Lo cual representa una circunferencia con centro en 𝑧0 = (𝑥0 − 𝑦0 ) y radio


𝑟. (Ver figura1.)

Fig.1 Fig.2
Y Y

z₀
r

z₀

0 X 0 X

2. 𝐵 = {𝑧/|𝑧 − 𝑧0 | < 𝑟}

Nos representa un círculo o disco de radio 𝑟 y centro 𝑧0 , en el cual no se


incluye la circunferencia. (Ver figura 2.)

3.𝐶 = {𝑧/|𝑧 − 𝑧0 | ≤ 𝑟}

𝐶 es un circulo o disco de radio 𝑟 en el cual se incluye la circunferencia .


(Ver figura 3.)
Nótese que 𝐶 = 𝐵 ∪ 𝐴

Y X
0

Z₀

Fig.3

Ejemplo: Determine los puntos 𝑧 = 𝑥 + 𝑖𝑦 del


plano complejo, que satisfacen:
|𝑧 − 1| ≤ 3|𝑧 + 1|

Solución:

Como 𝑧 = 𝑥 + 𝑖𝑦 ⇒ 𝑧 − 1 = 𝑥 + 𝑖𝑦 − 1 = (𝑥 − 1) + 𝑖𝑦

⇒ 𝑧 + 1 = (𝑥 + 1) + 𝑖𝑦

Tomando modulo y reemplazando en |𝑧 − 1| ≤ 3|𝑧 + 1| , se obtiene:

√(x − 1)2 + y 2 ≤ 3√(x − 1)2 + y 2

𝑥 2 − 2𝑥 + 1 + 𝑦 2 ≤ 9(𝑥 2 + 2𝑥 + 1 + 𝑦 2

−8𝑥 2 + 20𝑥 + 8𝑦 2 ≥ −8
5
−8𝑥 2 + 𝑥 + 𝑦 2 ≥ −1
2
5 2 2
𝑥 2 + 2 𝑥 + (54) + 𝑦 2 ≥ (54) − 1

(𝑥 + 5⁄4)2 +𝑦 2 ≥ 9⁄16

Esta última expresión, definida en el plano, representa dos partes:

1) La igualdad (𝑥 + 5⁄4)2 +𝑦 2 ≥ 9⁄16 representa a la circunferencia, de cetro


(− 5⁄4 ; 0) y radio 𝑟 = 3⁄4.
2) La desigualdad (𝑥 + 5⁄4)2 +𝑦 2 ≥ 9⁄16representa a la parte externa de la
circunferencia definida en (1).En forma equivalente podríamos decir que nos
representa la parte externa al círculo definida por la circunferencia en mención.
Pasando la ecuación a simbolismo complejo, nos queda, por simple identificación:

𝑧 = (𝑥; 𝑦); 𝑧0 = (− 5⁄4 ; 0) y 𝑟 = 3⁄4

Luego:

(𝑥 + 5⁄4)2 +𝑦 2 ≥ 9⁄16 |𝑧 − 𝑧0 |2 ≥ 3⁄4

|𝑧 − 𝑧0 | ≥ 3⁄4

En conclusión, los puntos que cumplen |𝑧 − 1| ≤ 3|𝑧 + 1| son los puntos exteriores
al círculo |𝑧 − 𝑧0 | < 3⁄4 de centro 𝑧0 = − 5⁄4 + 0𝑖 = − 5⁄4y radio 𝑟 = 3⁄4

FORMA POLAR EN EL NUMERO COMPLEJO

Sea 𝑧 = (𝑥 + 𝑖𝑦) un complejo diferente de cero.

Esto es equivalente a considerar |𝑧| = 𝑟 ≠ 0. y

Sea 𝛼 el ángulo, medido en radianes, formado Y


por la posición positiva del eje x i por el vector →
𝑜𝑧 r=|𝑧|
(ver la figura), medido en sentido opuesto al
movimiento de las agujas del reloj, igual como en α
trigonometría. De la figura se deducen las
siguientes relaciones. 0 x x
α+π
𝑥 = 𝑟 cos 𝛼
𝑦 = 𝑟 sen 𝛼
Luego:

𝑧 = 𝑥 + 𝑖𝑦 = 𝑟 cos 𝛼 + 𝑖𝑟 𝑠𝑒𝑛𝛼
Entonces:

𝑧 = 𝑟(cos 𝛼 + 𝑖 𝑠𝑒𝑛𝛼) ….(*)

A la expresión (*) se le conoce como la representación polar de un numero


complejo z, donde r representa el módulo de z, a “α”se le denomina el argumento
de z, arg. (z) o también conocido como amplitud de z.
ARGUMENTO PRINCIPAL DE UN NUMERO COMPLEJO

Se denomina argumento principal de z, cuando “a” asume valores posibles que se


encuentra entre 0 y 2𝜋(0 ≤ 𝛼 < 2𝜋). A tal valor se le denomina el conjunto
principal de z y se la denota Arg(z), es decir:

𝛼 = arg(𝑧) = 𝐴𝑟𝑔(𝑧) + 2𝑘𝜋 ; k=0, 1, 2…

Esto nos indica, que conociendo el argumento principal Arg (z), cualquier
argumento se obtiene sumándole 2kπ, k= 0; 1; 2……, al argumento principal.

Definición: Los números reales r y α=arg(z)se denomina las coordenadas polares


de z y son utilizados en su representación polar .

Nota: Según lo anterior, la representación polar de z se efectuará en base al


argumento principal de z; es decir, 0 ≤ 𝛼 < 2𝜋, sin perdida de generalidad.

Observaciones:

1) Algunos textos definen −𝜋 < 𝛼 < 𝜋 como el argumento principal de z.

En ciertos casos conviene considerar más este rango que el hemos dado.

2) Para calcular “α” tenemos en cuenta lo siguiente:


i.Ponemos en número complejo z=x+iy bajo la forma z=(x; y), con lo cual
representamos geométricamente z, (en el que observa en que cuadrante
está “α”).
ii.Calculamos “α” a partir de:

𝑦
𝛼 = arg 𝑡𝑔( ) ; 0 ≤ α < 2𝜋
𝑥
(ver la figura anterior).
𝑦 𝑥
Nota: también se puede usar: senα = , cos α=|𝑧| (ver la figura anterior), con
|𝑧|
lo cual se tiene:
𝑦 𝑥
𝛼 = arc sen (|𝑧|) ó α = arc cos (|𝑧|)
Ejemplo1: Hallar la representación polar de z=1+i
Y
Solución:
Se tiene que z=1+i=(1;1)
Z= (1; 1)
1
Entonces: 𝑟 = |𝑧| = √12 + 12 = √2 𝑟 = |𝑧|
Cálculo del argumento: 𝛼 = 𝜋⁄4
𝑦 0 X
𝛼 = 𝐴𝑟𝑔(𝑧) = 𝑎𝑟𝑐 𝑡𝑔 ( ) = 𝑎𝑟𝑐 𝑡𝑔1 = 𝜋⁄4 1
𝑥
Luego, como 0 ≤ α = 𝜋⁄4 < 2𝜋, entonces tendremos la representación polar:

𝑧 = 𝑟(𝑐𝑜𝑠𝛼 + 𝑖𝑠𝑒𝑛 𝛼) = √2(𝑐𝑜𝑠 𝜋⁄4 + 𝑖𝑠𝑒𝑛 𝜋⁄4)

𝑧 = 1 + 𝑖 = √2(𝑐𝑜𝑠 𝜋⁄4 + 𝑖𝑠𝑒𝑛 𝜋⁄4)

Observación: veamos que sucede si no usamos el argumento principal:

Sabemos que: 𝛼 = arg(𝑧) = 𝐴𝑟𝑔(𝑧) + 2𝑘𝜋 , 𝑘 = 0; 1; 2 …

Pero: 𝑐𝑜𝑠𝛼 = cos(𝜋⁄4 + 2𝑘𝜋) = 𝑐𝑜𝑠 𝜋⁄4 = √2⁄2

𝑠𝑒𝑛𝛼 = s 𝑒𝑛(𝜋⁄4 + 2𝑘𝜋) = 𝑠𝑒𝑛 𝜋⁄4 = √2⁄2

𝑧 = √2(cosα + isenα)

= √2(cos 𝜋⁄4 + 𝑖𝑠𝑒𝑛 𝜋⁄4)

= √2(√2⁄2 + 𝑖√2⁄2)

=1+i
Como se observa, se llega a la expresión original.

Ejemplo 2: Hallar la representación polar de: z=2+3i


Solución:
Y
Vemos que z=2+3i= (2; 3)

𝑟 = |𝑧| = √22 + 32 + √13 Z= (2; 3)


3
Calculo del argumento: 𝑟 = |𝑧|
𝑦
𝛼 = 𝐴𝑟𝑔(𝑧) = arg 𝑡𝑔 ( ⁄𝑥)
α
= arg 𝑡𝑔 (3⁄2)
0 2 X
Luego, como 𝛼 = arg 𝑡𝑔 (3⁄2), con 0 ≤ 𝛼 < 2𝜋; entonces

Se tendrá la representación polar siguiente:

𝑧 = 𝑟(𝑐𝑜𝑠𝑐 + 𝑖𝑠𝑒𝑛𝛼) = √13(𝑐𝑜𝑠𝛼 + 𝑖𝑠𝑒𝑛𝛼)

Dónde: 𝛼 = 𝑎𝑟𝑡 𝑡𝑔(3⁄2) ∈ [0; 2𝜋⟩

Nota: también podemos ver qué ocurre si 𝛼 = 𝑎𝑟𝑡 𝑡𝑔(3⁄2) + 2𝑘𝜋 , 𝑘 = 0; 1,2 ….

LA EXPONENCIAL COMPLEJA

Definición: sea z=x+iy, se define 𝑒 𝑧 como:

𝑒 𝑧 = 𝑒 𝑥 (𝑐𝑜𝑠𝑦 + 𝑖𝑠𝑒𝑛𝑦)

Donde 𝑒 𝑧 es la función exponencial real, 𝑒 es la base de los logaritmos neperianos.

Caso particular: si z= iy es imaginario puro, se tiene:

𝑒 𝑖𝑦 = 𝑐𝑜𝑠𝑦 + 𝑖𝑠𝑒𝑛𝑦 , 𝑦𝑎 𝑞𝑢𝑒 𝑒 𝑥 = 𝑒 0 = 1

Luego, la representación polar es equivalente a la siguiente forma:

𝑧 = 𝑟(cos 𝛼 + 𝑖𝑠𝑒𝑛𝛼) = 𝑟𝑒 𝑖𝛼 𝑧 = 𝑟𝑒 𝑖𝛼

Donde, 𝑟 = |𝑍| 𝑦 𝛼 = 𝐴𝑟𝑔(𝑧)

Ejemplo1:

Sabemos que 𝑧 = 1 + 𝑖 = √2(cos 𝜋⁄4 + 𝑖𝑠𝑒𝑛 𝜋⁄4) , donde 𝑟 = |𝑧| = √2 𝑦 𝑎 = 𝜋⁄4.

Luego, z puede escribirse en la forma equivalente:


𝜋⁄
𝑧 = √2𝑒 4

Ejemplo2:
De igual modo, z=2+3i puede escribirse como:
𝑖𝑎𝑟𝑐 𝑡𝑔3⁄
𝑧 = √13𝑒 2

PROPIEDADES:

1. 𝑒 𝑧1 . 𝑒 𝑧2 = 𝑒 𝑧1+𝑧2
1
2. 𝑒 −𝑧 = 𝑒 𝑧
3. 𝑒 𝑧 ≠ 0; ∀𝑧 ∈ ∁
4. |𝑒 𝑧 |𝑒 𝑥 ; 𝑦 = 𝐴𝑟𝑔(𝑒 𝑧 ), 𝑑𝑜𝑛𝑑𝑒 𝑧 = 𝑥 + 𝑖𝑦
5. 𝑒 𝑧 = 1 𝑧 = 2𝑘𝜋; 𝑘 = 0 ± 1; ±2; …
Demostraremos las propiedades 1y4:
Demostraciones:
Sean z = x + iy , z1=𝑥1+𝑖 𝑦1 𝑦 z2=𝑥2+𝑖 𝑦2 ; luego:
a. 𝑒 𝑧1 . 𝑒 𝑥1 = 𝑒 𝑥1 (𝑐𝑜𝑠𝑦1 + 𝑖𝑠𝑒𝑛𝑦1 ); 𝑒 𝑧2 = 𝑒 𝑥2 (𝑐𝑜𝑠𝑦2 + 𝑖𝑠𝑒𝑛𝑦2 )

𝑒 𝑧1 . 𝑒 𝑧2 = 𝑒 𝑥1 (𝑐𝑜𝑠𝑦1 + 𝑖𝑠𝑒𝑛𝑦1 ). 𝑒 𝑥2 (𝑐𝑜𝑠𝑦2 + 𝑖𝑠𝑒𝑛𝑦2 );

= 𝑒 𝑥1 . 𝑒 𝑥2 (𝑐𝑜𝑠𝑦1 + 𝑖𝑠𝑒𝑛𝑦1 )(𝑐𝑜𝑠𝑦2 + 𝑖𝑠𝑒𝑛𝑦2 )

= 𝑒 𝑥1+𝑥2 [(𝑐𝑜𝑠𝑦1 . 𝑐𝑜𝑠𝑦1 − 𝑠𝑒𝑛𝑦1 . 𝑠𝑒𝑛𝑦2 ) + 𝑖(𝑐𝑜𝑠𝑦1 . 𝑠𝑒𝑛𝑦2 + 𝑠𝑒𝑛𝑦1 𝑐𝑜𝑠𝑦2 )]

= 𝑒 𝑥1+𝑥2 [cos(𝑦1 +𝑦2 ) + 𝑖𝑠𝑒𝑛(𝑦1 +𝑦2 )]

= 𝑒 𝑧1+𝑧2

4. 𝑒 𝑧 = 𝑒 𝑥 (𝑐𝑜𝑠𝑦 + 𝑖𝑠𝑒𝑛𝑦) |𝑒 𝑧 | = 𝑒 𝑥 √(𝑐𝑜𝑠𝑦)2 + (𝑠𝑒𝑛𝑦)2 = 𝑒 𝑥

Calculo del argumento:

𝑒 𝑧 = (𝑒 𝑥 𝑐𝑜𝑠𝑦) + 𝑖(𝑒 𝑥 𝑠𝑒𝑛𝑦)


(𝑒 𝑥 𝑠𝑒𝑛𝑦)
𝛼 = 𝐴𝑟𝑔(𝑒 𝑧 ) = 𝑎𝑟𝑐 𝑡𝑔 ( 𝑥 ) = 𝑎𝑟𝑐 𝑡𝑔 (𝑡𝑔𝑦) = 𝑦
(𝑒 𝑐𝑜𝑠𝑦)

PROPIEDADES COMPLEJAS
Propiedad1.sean𝑧1 = 𝑟1 𝑒 𝑖𝛼1 𝑦 𝑧2 = 𝑟2 𝑒 𝑖𝛼2 , las representaciones polares de
𝑧1 𝑦 𝑧2 entonces:

𝑧1 𝑧2 =(𝑟1 𝑒 𝑖𝛼1 )(𝑒 𝑖𝛼2 )

= (𝑟1 . 𝑟2 )𝑒 𝑖𝛼1 . 𝑒 𝑖𝛼2

= 𝑟1 𝑟2 𝑒 𝑖(𝛼1+𝛼2) … … … . (𝑎)

NOTA: ¿será 𝑧1 𝑧2 = 𝑟1 𝑟2 𝑒 𝑖(𝛼1+𝛼2) la representación polar de 𝑧1 𝑧2 ?Para


responder esta pregunta, veamos de qué manera puede calcularse la
representación polar de 𝑧1 𝑧2.

Usando la definición, la representación polar de 𝑧1 𝑧2 puede escribirse


como:

𝑧1 𝑧2 = | 𝑧1 𝑧2 |𝑒 𝑖𝐴𝑟𝑔( 𝑧1 𝑧2 )

= | 𝑧1 ||𝑧2 |𝑒 𝑖𝐴𝑟𝑔( 𝑧1 𝑧2 )

= 𝑟1 . 𝑟2 𝑒 𝑖𝐴𝑟𝑔( 𝑧1 𝑧2 )
……. (b)

Igualamos (a) (b) pues son representaciones del mismo número:

𝑒 𝑖(𝛼 1 + 𝛼2 ) = 𝑒 𝑖𝐴𝑟𝑔( 𝑧1 + 𝑧2 )

𝑒 𝑖𝐴𝑟𝑔( 𝑧1 + 𝑧2)
=1
𝑒 𝑖(𝛼 1 + 𝛼2)
𝑧1 + 𝑧2 ) 1 + 𝛼2 )
𝑒 𝑖𝐴𝑟𝑔( . 𝑒 −𝑖(𝛼 =1
𝑧1 𝑧2 )−𝛼 1− 𝛼2 ]
𝑒 𝑖[𝐴𝑟𝑔( =1
𝐴𝑟𝑔( 𝑧1 𝑧2 ) − 𝛼 1− 𝛼2 = 2𝐾𝜋 ; 𝐾 = 0; ±1; ±2, ….
𝐴𝑟𝑔( 𝑧1 𝑧2 ) = 𝛼 1+ 𝛼2 + 2𝐾𝜋
; K=0; ±1……..
𝐴𝑟𝑔( 𝑧1 𝑧2 ) = 𝐴𝑟𝑔( 𝑧1 ) + 𝐴𝑟𝑔(𝑧2 ) + 2𝐾𝜋

OBSERVACIONES
a) En la última formula, k puede ser diferente de cero, de modo que el
argumento principal dl producto de 2complejos puede no ser igual a la suma
de los argumento principales de dichos números. Veamos esto en el siguiente
ejemplo :

Ejemplo. Sean 𝑧1 = 𝑧2 = −1

i. Hallar Arg ( 𝑧1)=Arg(𝑧2 )


ii. Hallar 𝐴𝑟𝑔( 𝑧1 . 𝑧2 )
iii. ¿Qué puede decirse acerca de 𝐴𝑟𝑔( 𝑧1 . 𝑧2 )𝑦 𝐴𝑟𝑔( 𝑧1 ) + 𝐴𝑟𝑔(𝑧2 )
iv. ¿Qué afirmaría si aplica la última formula encontrada?

Solución: Y
i. 𝑧1 = 𝑧2 = −1 = −1 + 0𝑖 = (−1; 0) Escriba aquí la ecua
Luego:
0 𝛼
𝛼 = 𝐴𝑟𝑔( 𝑧1 ) = 𝐴𝑟𝑔(𝑧2 ) = 𝑎𝑟𝑐 𝑡𝑔 (−1) = 𝑎𝑟𝑐 𝑡𝑔 (0) 𝑧1 = 𝑧2
Pero del grafico 0
−1 𝑋
𝐴𝑟𝑔( 𝑧1 ) = 𝐴𝑟𝑔(𝑧2 ) = 𝜋
Nota: obsérvese que si no hubiéramos hecho el grafico, se tendría, a partir de
la igualdad α = 𝑎𝑟𝑐 𝑡𝑔 (0) 𝛼 = 0 ó 𝛼 = 𝜋, dos valores para el argumento
(los cuales pertenecen a [0; 2𝜋⟩π).si este fuera el caso, lo más recomendable
seria aplicar cualquiera de las otras dos fórmulas dadas para calcular el
argumento, como veremos en la pregunta (ii).
ii. 𝑧1 𝑧2 = (−1)(−1) = 1 = 1 + 0𝑖 = (1; 0)
1
Luego, 𝐴𝑟𝑔( 𝑧1 𝑧2 ) = 𝑎𝑟𝑐 cos( ) = 𝑎𝑟𝑐 𝑐𝑜𝑠1 = 0
√12+02

iii) Entonces: 𝐴𝑟𝑔(𝑧1 𝑧2 ) = 0 ≠ 𝐴𝑟𝑔(𝑧1 ) + 𝐴𝑟𝑔(𝑧2 ) = 2𝜋.


iv) Sin embargo, aplicando la fórmula encontrada:
𝐴𝑟𝑔(𝑧1 𝑧2 ) = 0 ≠ 𝐴𝑟𝑔(𝑧1 ) + 𝐴𝑟𝑔(𝑧2 ) = 2𝜋 ; 𝑘 = 0 ; ± 1 ; ± 2 … . .
Tenemos: 0 = 𝜋 + 𝜋 + 2𝑘𝜋, lo cual se verifica únicamente para 𝑘 = −1.

Conclusión:
Observamos que el 𝐴𝑟𝑔(𝑧1 𝑧2 ), (argumento del producto), y la suma 𝐴𝑟𝑔(𝑧1 ) +
𝐴𝑟𝑔(𝑧2 ) , (suma de argumentos, lo cual también se puede representar como 𝛼1 +
𝛼2 , difieren 2𝑘𝜋, 𝑘 = 0; ±1; ±2 ….Esto podría tentarnos a decir que a no es la
representación polar del producto 𝑧1 𝑧2 .
b) Para entender lo anterior, veamos cómo afecta 2𝑘𝜋 a los argumentos
principales, para lo cual calculamos el seno y el coseno de 𝐴𝑟𝑔(𝑧1 𝑧2 ).
cos[𝐴𝑟𝑔(𝑧1 𝑧2 ) ] = cos[𝐴𝑟𝑔(𝑧1 ) + 𝐴𝑟𝑔(𝑧2 ) + 2𝑘𝜋]
= cos[𝐴𝑟𝑔(𝑧1 ) + 𝐴𝑟𝑔(𝑧2 )]
= cos(𝛼1 + 𝛼2 )

sen[𝐴𝑟𝑔(𝑧1 𝑧2 ) ] = sen[𝐴𝑟𝑔(𝑧1 ) + 𝐴𝑟𝑔(𝑧2 ) + 2𝑘𝜋]


= sen[𝐴𝑟𝑔(𝑧1 ) + 𝐴𝑟𝑔(𝑧2 )]
= sen(𝛼1 + 𝛼2 )
Con lo cual tenemos que, efectivamente, (a) es la representación polar de 𝑧1 𝑧2 .
(𝛼1 𝛼2 )
𝑧1 𝑧2 = 𝑟1 𝑟2 𝑒 𝑖 = 𝑟1 𝑟2 [𝑐𝑜𝑠(𝛼1 + 𝛼2 ) + 𝑖 𝑠𝑒𝑛 (𝛼1 + 𝛼2 )]

Se concluye que, para multiplicar 2 complejos en su forma polar, se multiplican


sus módulos y se suman sus argumentos, respectivamente. (Ver figura).

(𝛼1 𝛼2 )
Nota: Obsérvese que la representación polar 𝑧1 𝑧2 = 𝑟1 𝑟2 𝑒 𝑖 no nos indica
que 𝛼1 + 𝛼2 sea el argumento principal de 𝑧1 𝑧2 ; para que lo sea, habría que
sumarle 2𝑘𝜋 (con , 𝑘 = 0; ±1; ±2; ….), según sea el caso. Con lo cual, el nuevo
argumento " 𝑎1 + 𝑎2 + 2𝑘𝜋" estaría variando en [0 ; 2𝜋) y sería el argumento
principal de 𝑧1 𝑧2 .
Ejemplo:
Sean 𝑧1 = 𝑧2=−1. Sabemos que 𝐴𝑟𝑔(𝑧1 ) = 𝐴𝑟𝑔(𝑧2 ) 𝑧1 𝑧2 = 1.1𝑒 𝑖(𝜋+𝜋) = 𝑒 𝑖(2𝜋) .
Pero 2𝜋 no es el argumento principal de 𝑧1 𝑧2 , ya que 2𝜋[0; 2𝜋⟩. Luego, el
argumento principal se obtendrá, en este caso, restándole 2𝜋(𝑘 = −1); es decir:
𝐴𝑟𝑔(𝑧1 𝑧2 ) = 2𝜋 − 2𝜋 = 0.
Tal como lo habíamos afirmado en el ejemplo anterior.
Multiplicación (𝑐𝑎𝑠 𝑜𝑘 = 0)

Propiedad 2. Sean: 𝑧1=𝑟 𝑒 𝑖∝1 y


1

𝑧2=𝑟2𝑒 𝑖𝑎2
Luego
𝑧1 𝑟1 𝑒 𝑖∝1
=
𝑧2 𝑟2 𝑒 𝑖∝2
𝑟1
= 𝑒 𝑖(∝1−∝2)
𝑟2
Con |𝑧2 | = 𝑟2 ≠ 0

Para dividir 2 complejos, en su forma polar, se dividen sus módulos y se resta el


argumento del divisor del argumento del dividendo.

Ejemplo 1. Hallar 𝑧1 . 𝑧2 , 𝑠𝑖 𝑧1 = 1 + 𝑖 𝑦 𝑧2 = −1 + √3𝑖


Solución:
Pasando a sus respectivas representaciones polares, tenemos:
𝑧1 = √2𝑒 𝑖(𝜋/4) ; (ya se efectuó en un ejemplo anterior)
𝑧2 = −1 + √3𝑖 = (−1; √3)

|𝑧2 | = √(−1)2 + (√3)2 = 2

Según la gráfica:
𝑠𝑒𝑛𝛽 = √3/2𝛽 = 60°
 ∝= 𝜋 − 60° = 2𝜋/3
Luego: 𝑧2 = 2𝑒 𝑖(2𝜋/3)
Realizando la multiplicación:

𝑧1 𝑧2 = 2√2. 𝑒 𝑖[(𝜋/4)+2𝜋/3]
= 2√2. 𝑒 𝑖[11𝜋/12]
= 2√2[cos(11𝜋/12) + 𝑖𝑠𝑒𝑛(11𝜋/12)]
−√2 − √6 √6 − √2
= 2√2 ( +𝑖 )
4 4

= −1 − √3 + 𝑖(√3 − 1)

Nota: Obsérvese 11𝜋/12 𝜖[0; 2𝜋] que es el argumento principal de 𝑧1 𝑧2

Ejemplo 2. Hallar 𝑧1 /𝑧2


Solución:
𝑧1 √2𝑒 𝑖𝜋/4 √2 −𝑖5𝜋/12
= = 𝑒
𝑧2 2𝑒 𝑖𝜋/3 2
𝑧1 √2 −5𝜋 −5𝜋
 = [cos ( ) + 𝑖𝑠𝑒𝑛 ( )]
𝑧2 2 2 2

√2 √6 − √2 −(√6 + √2)
= [ +𝑖 ]
2 4 4

√3 − 1 −(√3 + 1)
= +𝑖
4 4

Nota: De igual modo, −5𝜋/12[0; 2𝜋⟩ no es el argumento principal de 𝑍1 /𝑍2 . Para


encontrarlo, de modo que pertenezca a [0; 2𝜋⟩, habría que sumarle 2𝑘𝜋 (en este
caso con 𝑘 = 1).
Luego, el argumento principal de 𝑍1 /𝑍2 es:
−5𝜋/12 + 2𝜋 = 19𝜋/12
𝑛 𝑠𝑢𝑚𝑎𝑛𝑑𝑜𝑠

Propiedad 3. (𝑒 𝑧 )𝑛 =⏟ 𝑧 𝑧 𝑧
𝑒 .𝑒 .𝑒 .…….𝑒 = 𝑒 𝑧 𝑧+𝑧+........+𝑧
= 𝑒 𝑛𝑧
𝑛 𝑓𝑎𝑐𝑡𝑜𝑟𝑒𝑠

Caso particular: si 𝑧 = 𝑖𝑎, entonces:


𝑛
(𝑒 𝑖𝑎 ) = 𝑒 𝑖𝑛𝑎 ; FORMULA DE LA MOIVRE

POTENCIA DE UN NUMERO COMPLEJO


Sea 𝑍 = 𝑟𝑒 𝑖∝ , luego:
𝑛
𝑧 𝑛 = (𝑟𝑒 𝑖∝ )  𝑧 𝑛 = 𝑟 𝑛 𝑒 𝑖𝑛∝ ó
en forma equivalente:
𝑧 𝑛 = 𝑟 𝑛 (cos 𝑛 ∝ +𝑖𝑠𝑒𝑛 𝑛 ∝) ; 𝑛 = 0; 1; 2; 3; … … …

Ejemplo 1. 𝐷𝑎𝑑𝑜 𝑧 = 4 + 4√3 𝑖 , hallar 𝑧 4 .


Solución:
Primero llevamos 𝑧 a la forma polar, para lo cual hallamos:
2
|𝑧| = 𝑟 = √42 + (4√3) = 8

4√3
∝ = 𝐴𝑟𝑔 (𝑧) = 𝑎𝑟𝑐 𝑡𝑔 = 𝑎𝑟𝑐 𝑡𝑔 √3 = 𝜋/3
4
De donde, 𝑧 en su forma polar: 𝑧 = 8𝑒 𝑖4𝜋/3 . Luego:
4
𝑧 4 = (8𝑒 𝑖𝜋/3 ) = 84 𝑒 𝑖4𝜋/3
4𝜋 4𝜋
 𝑧 4 = 4096 (cos + 𝑖 𝑠𝑒𝑛 )
3 3
1 √3
= 4096 [− + 𝑖 (− )]
2 2

= 2048 − 2048√3 𝑖

1 √3
Ejemplo 2. Dado 𝑧 = − + 𝑖 , hallar 𝑧 −3 .
2 2
Solución:
Teniendo en cuenta que:
−𝑛 1 1 1
𝑧 −𝑛 = (𝑟𝑒 𝑖∝ ) = 𝑟 −𝑛 𝑒 −𝑛𝑖∝ = ( ) ( ) =
𝑟 𝑛 𝑒 𝑖𝑛∝ 𝑧𝑛
1
Tenemos: 𝑧 −3 = , y nuestro problema es ahora calcular 𝑧 3 .
𝑧 3,

Llevando 𝑧 a su forma polar:


2
1 2 √3
𝑟 = |𝑧| = √(− ) + ( ) = 1
2 2

√3/2 2𝜋
 ∝ = 𝑎𝑟𝑐 𝑡𝑔 ( ) = 𝑎𝑟𝑐 𝑡𝑔 (−√3) =
−1/2 3

De donde: 𝑧 = 𝑟𝑒 𝑖∝ = 1. 𝑒 𝑖2𝜋/3 ; entonces:

3
𝑧 3 = (13 𝑒 2𝜋/3 ) = 13 . 𝑒 𝑖2𝜋  𝑧 3 = cos 2𝜋 + 𝑖𝑠𝑒𝑛2𝜋 = 1 + 𝑖(0) = 1

Finalmente:
1 1
x-3= = =1
𝑧3 (1)

Nota: Usando la propiedad 2 de la exponencial compleja, obtenemos la conclusión


de que la formula de Moivre para n∈ ℤ.

Así, la solución del ejercicio anterior podría haberse hallado de manera directa, de
la manera siguiente:

Sabemos que: z=𝑒 1(2𝜋/3)

⇒ z-3=𝑒 −3(2𝜋/3) = 𝑒 𝑖(−2𝜋) = 𝑒 𝑖(0) = 1

RAICES DE NÚMEROS COMPLEJOS


Un complejo w es llamado una raíz enésima de un complejo z si y solo si wn= z.

Notación: w=z1/n
Deducción de la Fórmula de Radiación
Sean: z=reia y w = teib ………………………………………….………………(*)
Nuestro problema consiste en hallar t y b, con lo cual se conocería w. Por
definición
De(*): wnz
(teib)n= reia
⟹ tnei(nb) = reia
Igualando términos semejantes: r=tn ⟹ t= r1/n

eia = einb ⟹ ei(nb-a) = 1


Entonces:
nb – a = 2kπ
𝑎+2𝑘π
⟹b=
π
Por lo tanto:
W= r1/n𝑒 i(a+2kπ)/n
Que es equivalente a:
a+2kπ a+2kπ
wk= r1/n[𝑐𝑜𝑠 ( ) + 𝑖𝑠𝑒𝑛 ( )]………………………………..……………(**)
𝑛 𝑛

Observación: k es un entero que varía de 0,1,…, n-1. Observe que hay “n”
raíces. K no puede valer n, ya que en este caso el argumento (∝/𝑛 + 2π) y el
argumento ∝/𝑛 que corresponde para valores de k mayores que n.

Ejemplo 1: Hállese las raíces de orden 4 de z=-2048-2048√3i


Solución:

Levamos z a su forma polar: z = 2048√(−√3)2 + (−1)2= 4096 = r


y

-2048
0 x

-2048√3

∝=arc tg (-2048√3/-2048)= arc tg(√3)= 4π/3; (ver gráfico).


Luego, reemplazando en (**), para n=4:
4 2kπ+4π/3 2kπ+4π/3
wk= √4096 [𝑐𝑜𝑠 ( ) + 𝑖𝑠𝑒𝑛 ( )] , k = 0; 1; 2; 3.
4 4

Las raíces serán, respectivamente:


W0= 8[cos π/3+i sen π/3]=4+4√3i ; (k=0)
W1= 8[cos5π/6+i sen 5π/6]=-4√3i+4i ; (k=1)
W2= 8[cos 4π/3+i sen 4π/3]=-4-4√3i ; (k=2)
W3= 8[cos 11π/6+i sen 11π/6]=4√3+4i ; (k=3)

Ejemplo 2: Hállese una fórmula par alas raíces n-ésimas de la unidad.


Solución:
Como z=1, y en su forma polar; r=1 y ∝=arc tg0=0. Luego, en la ecuación (**):
2kπ 2kπ
wk = cos + 𝑖 𝑠𝑒𝑛 , 𝑘 = 0; 1; … : 𝑛 − 1
𝑛 𝑛
Si por ejemplo n = 3, las 3 raíces de la unidad serían:
1 √3 1 √3
w0 = 1 ; w1 = - + i ; w2 =- − i i
2 2 2 2

RAÍCES DE POLINOMIOS

INTRODUCCIÓN
Definiremos un polinomio en la variable z ∈ ℂ, como una cuya forma generales:
𝑛
𝑛 𝑛−1 𝑛−2
P(Z) = 𝑎𝑛 𝑧 +𝑎𝑛−1 𝑧 + 𝑎𝑛−2 𝑧 + ⋯ + 𝑎1 𝑧 + 𝑎0 = ∑ 𝑎𝑘 𝑧 𝑘
𝑘=0

Donde 𝑎𝑘 son constantes 𝑎𝑘 ∈ ℂ 𝑦 𝑛 ∈ ℕ.


El dominio y rango de la función polinomio los consideramos en los números
complejos ℂ .

OBSERVACIONES:
a) Si an≠ 0 ⇒ P(z) es un polinomio de grado n, lo cual se denota por gdo P(z)= n.
A“an” se le denomina coeficiente principal del polinomio.
b) Si n=0 y a0≠0⇒ gdo P(z)=0
c) Si n=0 y a0=0⇒ gdo P(z) no existe. (algunos autores consideran que es de
grado-∞).
d) Si grado de P es n=1⇒ P(z)= a0+ a1z : Polinomio Lineal
Si grado de P es n=2⇒ P(z)= a0+ a1z + a2z2: Polinomio Cuadrático
Si grado de P es n=3⇒ P(z)= a0+ a1z+ a2z2 + a3z3: Polinomio Cúbico
Si grado de P es n=4⇒ P(z)= a0+ a1z + a2z2 + a3z3+ a4z4 : Polinomio de Grado 4
e) Si z = x + i(0) ⇒ P(z) Es un polinomio de Variable Real, es decir:
P(z)= P(x) = 𝑎𝑛 𝑧 𝑛 +𝑎𝑛−1 𝑧 𝑛−1 + ⋯ 𝑎1 𝑥 + 𝑎0
Donde los coeficientes 𝑎𝑘 pueden pertenecer a ℤ, ℚ, 𝕀, ℝ ó ℂ

CEROS O RAICES DE UN POLINOMIO


Definición: Un número r tal que P(r)=0 se dice que s un cero o raíz del polinomio P.
En forma equivalente se dice que r es una solución de la ecuación P(z) = 0.
Teorema Fundamental del Algebra
Todo polinomio de grado positivo n, definido sobre el campo de los números
complejos, tiene un cero en dicho campo.
La demostración de este teorema, de vital importancia en la teoría de solución de
Ecuación Polinomiales, no la haremos en el presente texto, ya que ella escapa a
los alcances teóricos usados. Pero si veremos a continuación los teoremas más
importantes relacionados con la solución de ecuaciones polinomiales.
TEOREMA1. Algoritmo de la División.
Sea P un polinomio de grado n y sea D un polinomio de grado m; (D≠0). Existen
polinomios únicos Q, de grado n-m, y R tales que:
P= QD+R
Donde gdo R< gdo D = m.
Al término P se denomina polinomio dividendo, D polinomio divisor, Q polinomio
cociente y R el resto de la división de P entre D.

Definición: Un polinomio S se dice o es divisor de un polinomio P, si p =QD; es


decir, si el resto R es igual a cero.

Caso Particular :
Sea D= (z-r) ⇒ gdo D = m=1 ⇒ R es una constante.
Este caso particular da lugar a los siguientes corolarios del teorema 1.
Corolario 1. (Teorema del Residuo): El resto de la división de un polinomio P(z)
por z-r es P(r).

Demostración:
Del algoritmo de la división se tiene:
P(z) = (z-r)Q(z)+R(z) , donde R(z) es el resto.
Luego:
P(r) = (r-r)Q(r)+ R(r) ⇒ P(r)= R(r)=resto.

Corolario 2. (Teorema del Factor): Un polinomio P es divisible por (z-r) ⇒r es un


cero de P (⇔P(R)=0).

Demostración:
(⟹) r es un cero de P ⇔ P(r)=0
Por el Corolario 1: P(r)=R=0
Luego: P(z)=(z-r)Q ⇒P es divisible por (z-r)
(⟸)P es divisible por (z-r) ⇒P(z)=(z-r)Q.
Por el Corolario 1: P(r)=(r-r)Q=0=R⇒r es un cero de P.
Corolario 3: Sea r≠s. Si P(z) es divisible por (z-r) y (z-s) ⇒ P(z) es divisible por el
producto (z-r)(z-s).

Demostración:
P es divisible entre (z-r) ⇒ ∃Q1 (𝑧) tal que:
P(z)=(z-r) Q1 (z) ….(𝛼)
Pero P es divisible entre (z-s) y por el teorema del factor y (𝛼) se tiene:
P(s)=0=(s-r) Q1 (s)
Como s≠r ⇒ Q1 (s)=0⇒ Q1 (z) es divisible entre z-s:
⇒ Q1 (z)=(z-s)Q(z) ….(𝛽)
Reemplazando (𝛽) en (𝛼):
P(z)=(z-r) (z-s) Q(z)

DEFINICIÓN: Si el polinomio DK divide al polinomio P y si ninguna las potencias


superiores del polinomio DK divide a P ⇔ D es un factor de multiplicidad k del
polinomio P. Es decir:
(1) 𝑃 = 𝐷 𝐽 𝑄 ; ∀𝐽 = 1; 2 ; … : 𝑘
DK es un factor de multiplicidad ⇔ {
(2) 𝑃 = 𝐷𝑘+1 𝑄 + 𝑅 ; R≠0
DK del polinomio P

Un caso especial del colorario 3 lo obtenemos si D=(z-r) es un factor multiplicidad


K de P, de donde se sigue:
P=(z-r)kQ(z), con Q(r) ≠0
Si existieran varios factores de diversa multiplicidad, teniendo en cuenta el
corolario 3, P sería divisible entre el producto de dichos factores; esto es, si (z-𝑟𝑗 )
es un factor de multiplicidad j y si P(z) es un polinomio de grado n, se cumple que:
P(z) = (z-r1) (z-r2)2 (z-r3)3… (z-rk)k ; ∀j=1,…,k : tal que:
𝑘

∑ 𝑗 = 𝑛 ; 𝑟𝑗 ∈ ℂ
𝑗=1
Un problema de capital importancia, en relación a las raíces de multiplicidad k,
Daremos el siguiente teorema que responde a nuestra interrogante.

TEOREMA2. (Factor de Multiplicidad k)


r es un raíz o cero de multiplicidad k de P(z)
⇔P(r)= P’(r)=…=P(k-1)(r)=0 y P(k)(r) ≠0
Donde P(j)(r) es la j-ésima derivada de P evaluada en r.

TEOREMA 3. Si un polinomio P(z) tiene grado n>0, entonces P tiene


exactamente n ceros o raíces en el campo complejo. Es decir,
P(z)=a(z-z1)(z-z2)2…(z-zn)

Demostración:
Usaremos el primer principio de inducción matemática para efectuar la
demostración. Sea:
𝑛
𝑆 =○ {P(𝑧) = 𝑎𝑛 𝑧 𝑛 +𝑎𝑛−1 𝑧 𝑛−1 + ⋯ + 𝑎1 𝑧 + 𝑎0 = 0 ; tiene n raíces}

i) 1 ∈ S; en efecto , n=1⇒ P(z)=a0+ a1z es un polinomio de grado 1 que


tiene raíz:
r=-a0/ a1 , a1≠0
ii) Asumamos que m ∈ S; esto es, todo polinomio de grado m posee m
raíces.
iii) Podremos que m+1∈ S;
Sea P un polinomio de grado m+1; por el teorema fundamental del
álgebra, P posee una raíz y por el teorema del factor se tiene:
P(z)=(z-r)Q(z), gdoQ = m
Como Q es de grado m, por (ii) entonces Q posee m raíces. Luego, P
posee m+1 raíces.
Corolario1:
P(z) = an z n +an−1 z n−1 + ⋯ + a1 z + a0 = 0 y
Q(z) = bn z n +bn−1 z n−1 + ⋯ + b1 z + b0 = 0
Son de grado n y si P (z) = Q (z) para n+1 valores distintos de s, entonces
P=Q,
La que es equivalente:
𝑎𝑘 = 𝑏𝑘 ; A K = 0; 1;…n

TEOREMA 4. Si un polinomio P con coeficientes reales tiene la raíz r


entonces (Conjugado de r) también es cero de P; es decir:

P(r) = 0 → P(r) = 0

Demostración:
Sea P (z) =∑n k
k=0 an z , 𝑎𝑘 ∈ R. Como r es cero de P, se tiene que:

P(r) = ∑𝒏𝒌=𝟎 𝒂𝒌 𝒓𝒌 = 0
Tomando conjugada en (α):

→ 𝑃(𝑟) = ∑𝑛𝑘=0 𝑎𝑘 𝑟 𝑘 = 𝑜 = 0
𝑘
→ 𝑃(𝑟) = ∑𝑛𝑘=0 𝑎𝑘 𝑟 𝑘 = ∑𝑛𝑘=0 𝑎k𝑟 𝑘 = ∑𝑛𝑘=0 𝑎𝑘 𝑟 𝑘 = ∑𝑛𝑘=0 𝑎𝑘 𝑟 = P (𝑟) = 0
De donde se concluye que 𝑟 es un cero de P.

Observación. Este teorema es valido sólo si los 𝑎𝑘 son valores reales. Si los 𝑎𝑘
son constantes complejas, no se puede decir que si r es una raíz entonces 𝑟
también es raíz.
Veamos el siguiente contra-ejemplo:

P (z) = 𝑧 3 – (3 + 6i) 𝑧 2 + (-9 + 12i) z + (11 + 2i) = 0

Se puede comprobar que r= 1+2i es una raíz. En efecto:

P (1 + 2i) = (1 + 2𝑖)3 - (3 + 6𝑖)(1 + 2𝑖)2 + (-9 + 12i) (1 + 2i) + 11 +2i

= -11 – 2i + 33 + 6i – 33 – 6i + 11 + 2i = 0

Sin embargo, 𝑟=1-2i no es raíz de P(x), pues:

P (1 + 2i) = (1 + 2𝑖)3 - (3 + 6𝑖)(1 + 2𝑖)2 + (-9 + 12i) (1 + 2i) + 11 +2i

= -11 – 2i + 15 + 30i + 15 + 30i + 11 + 2i


= 64i ≠ 0

Corolario 1: Todo polinomio con coeficientes reales siempre puede ser factor
izado hasta obtener factores lineales con coeficientes reales (abab) y factores
cuadráticos con coeficientes reales (a𝑧 2 +bz+c).

La demostración es inmediata teniendo en cuenta los teoremas 3y 4.

Por el teorema 3:

P (z) = a (z - 𝑧1 ) (z - 𝑧2 )… (z - 𝑧1 )… (z - 𝑧𝑘 )

Por el teorema 4, aparece como factor (z - 𝑧1 ); luego al multiplicar:

P (z) = a (z - 𝑧1 ) (z - 𝑧1 ) (z - 𝑧2 )… (z - 𝑧𝑘 ) ; k≤n–1

= a [ 𝑧 2 - (z - 𝑧1) z + |𝑧1 |2 ] (z - 𝑧2 )… (z - 𝑧𝑘 )

= a [ 𝑧 2 – 2Re ( 𝑧1 ) z + |𝑧1 |2 ] (z - 𝑧2 )… (z - 𝑧𝑘 )

Donde se observa que el polinomio cuadrático 𝑧 2 -2Re (𝑧1 ) z+|𝑍1 | 2


tiene
coeficiente reales.

Corolario 2: Si P (z) es de grado impar, entonces P tiene al menos un cero


real, es decir, una raíz real.

OBSERVACIÓN GENERAL
Si P(z) es un polinomio de grado n impar con coeficientes reales , sabemos
que P(z)=0 tiene n raíces, las cuales se distribuyen por pares: para cada raíz
compleja y no estricta-mente real, su conjugada también es raíz; pero si el
grado es impar ha existir una raíz no compleja (es decir, real) necesariamente,
pues si todas fueran estrictamente complejas, también sus conjugadas serían
raíces y el recuento total sería un numero par de raíces, contradiciendo el
hecho de que el grado de P(z), es n impar.

Ejemplo. Si 𝑟𝑒 −𝑖𝑎 , para r>0; a Є IR, es una raíz de P (z), polinomio con
coeficiente enteros, definido en C, de grado n positivo; demostrar que
𝑟𝑒 𝑖(𝑎−2𝑘𝜋) es otra raíz para k Є Z.
Solución:
Sea 𝑧0 =𝑟𝑒 −𝑖𝑎 , entonces:

𝑟𝑒 𝑖(𝛼−2𝑘𝜋) =𝑟𝑒 𝑖𝛼 =𝑧̅0


Si 𝑧0 es una raíz de P (z)=0, polinomio con coeficientes reales; entonces 𝑧̅0
es una raíz de P (z)=0.

MÉTODO DE LA DIVISIÓN SINTÉTICA


El teorema del factor muestra que el problema de hallar los ceros de un polinomio
esta ligado al hecho de hallar divisores lineales de p. para lo cual probaremos con
factores de la forma z – r y comprobaremos si, efectivamente, el resto de la
división de p por z – r es o no igual a cero.

Desarrollamos a continuación el llamado método de la división sintética, el cual


nos permitirá hallar el cociente y el residuo de la división de p (z) entre (z - r).

Se tendrán en cuenta las siguientes recomendaciones:


a) Ordenaremos los coeficientes del polinomio p (z) en orden descendente, de la
mayor potencia de z al termino independiente. Si alguna potencia de z no
apareciera en el polinomio p (z), se considera que tiene el coeficiente cero.
b) Para un divisor de la forma (z – r), el resto de la división de p (z) por z – r es
R=p(r).
c) Los coeficientes del polinomio cociente Q se obtienen a partir del algoritmo de
división:

P (z)= (z – r) Q (z) + R,

Donde Q (z) es un polinomio de grado n – 1.


En efecto, supongamos que:

P (z)=𝑎𝑛 𝑧 𝑛 + 𝑎𝑛−1 𝑧 𝑛−1 +… + 𝑎1 z + 𝑎0

Como Q (z) es de grado n – 1,

Q (z)=𝑏𝑛 𝑧 𝑛−1 +𝑏𝑛−2 𝑧 𝑛−2 +… + 𝑏1 z + 𝑏0

Donde debemos hallar los 𝑏𝑗 ;𝑗=0 ;1 ;… ; 𝑛−1

Remplazando:

P (z) = (z - r) ( 𝑏𝑛 𝑧 𝑛−1 +𝑏𝑛−2 𝑧 𝑛−2 +… + 𝑏1 z + 𝑏0 ) + R


= 𝑏𝑛 𝑧 𝑛 + (𝑏𝑛−2 - 𝑟𝑏𝑛−1 )𝑧 𝑛−1 +… + (𝑏0 - 𝑟𝑏1 ) z + (R - 𝑟𝑏0 )
Por el corolario (1) del teorema (3) se tiene:

𝑎𝑛 = 𝑎𝑛−1 ; 𝑎𝑛−1 = 𝑏𝑛−1 - 𝑟𝑏𝑛−1 ;…; 𝑎1 = 𝑏0 - 𝑟𝑏1 ; 𝑎0 = R - 𝑟𝑏0


De donde:

𝑎𝑛 = 𝑏𝑛−1 ; 𝑏𝑛−2 = 𝑎𝑛−1 + 𝑟𝑏𝑛−1 ;…; 𝑏0 = 𝑎1 + 𝑟𝑏1 ; R = 𝑎0 + 𝑟𝑏0


En general:

𝑏𝑛−𝑗 = 𝑎𝑛−𝑗+1 + 𝑟𝑏𝑛−𝑗+1 , j = 1; 2;…; n

d) No es necesario memorizar la anterior formula para hallar los 𝑏𝑘 .


Es conveniente tener en cuenta la siguiente distribución:

Coeficientes de P an an-1 an-2 a1 a0

Raíz o cero: z  r  0  z  r ran


R  a0  rb0
an an-1+ ran …

Coeficientes de Q bn-1 bn-2 ….b0 =b1 + ra1


Ejemplo 1: Hallar el cociente y el residuo de la división de
P (z) = 𝑧 2 - 𝑧 3 + 2𝑧 2 + 1
Por (z - 1)

Solución:
Distribuyendo los coeficientes de P:

Termino
z5 z4 z3 z2 z
independiente

Coeficientes 1 0 -1 2 0 1
de P
z r  0  z  r 1 1 0 2 2

Coeficientes 1 1 0 2 2 3 Resto
de Q
z4 z3 z2 z Termino
independiente

Luego, Q (z) = 𝑧 4 + 𝑧 3 + 𝑧 2 + 2z + 2 y R=3

Observación:
Si r fuera un cero de P (z), se tendrá R = 0 y se podría volver a usar el
método de la división sintética para hallar el cociente y el resto de la
división de Q (z) entre (z- 𝑟1).

RAICES REALES DE UN POLINOMIO


Definición. Sean p y q dos números enteros, se dice que estos números son
primos relativos si sus únicos divisores comunes son 1 y -1.

Teorema 5: Sea P (z)= 𝑎𝑛𝑍 𝑛 + 𝑎𝑛−1 𝑧 𝑛−1 +….+ 𝑎1 𝑧 + 𝑎0 , donde los 𝑎𝑖 ∈ . Si el


numero racional p /q, (donde p y q son primos relativos), es una raíz de P (z)
entonces p divide a 𝑎0 y q divide 𝑎𝑛 .
Observación. El anterior teorema también se puede aplicar en el caso de los
coeficientes 𝑎𝑖 sean números racionales, ya que al multiplicar P (z) por el mínimo
denominador común de los coeficientes obtendremos en polinomio con
coeficientes enteros.
Nota: Este teorema nos indica que las posibles raíces racionales (si es que
existen) son de la forma r = p /q , donde p es divisor de 𝑎0 , q es divisor de 𝑎𝑛 ;
pero no nos indica que todas las raíces reales sean de la forma anterior.

Corolario: P (z)= 𝑧 𝑛 + 𝑎𝑛−1 𝑧 𝑛−1 +….+ 𝑎1 𝑧 + 𝑎0 = 0; 𝑎𝑖 ∈ .

Son valores enteros divisores de 𝑎0


10 11 8 2
Ejemplo. Si P (x) = 2𝑥 5 - 𝑥4 + 𝑥 3 + 𝑥 2 - 𝑥1 -
3 6 3 3

a) Hallar las raíces de P (x)=0


b) Descomponer P (x) en el mayor número de factores con coeficientes
reales.

Solución:

a) Multiplicando P (x) por el mínimo común denominador e igualando a cero,


tenemos:

P (x)= 12𝑥 5 − 20𝑥 4 − 11𝑥 3 + 16𝑥 2 − 6𝑥 − 4 = 0 … (1)

Como P (x) es de grado 5, P (x) tiene 5 raíces (teorema 3), y además por ser de
grado impar tiene al me nos una raíz real (corolario 2).
Para calcula las raíces aplicamos el teorema 5:
vemos que:

𝑎0 = 4 ⇒ los divisores de 𝑎0 : p = ±1; ±2; ±4.

(Observa que el signo de 𝑎0 no afecta, ya que los divisores se toman don doble
signo).

𝑎5 = 12 ⇒ Divisores de 𝑎5 : q = ±1:±2; ±3; ±6; ±12.

Posibles raíces racionales:


r= p/q= ±1 ; ±2 ; ±4 ±1/2 ; ±1/3 ±1/6 ; ±1/12 ±2/3 ; ±4/3 .

(Observar que en total hay 18 posibles raíces racionales).


Aplicamos el método de división sintética:
Hacemos la prueba con r=1, (divisor de la forma x-1). Luego:

12 -20 11 16 -6 -4

r=1 12 -8 3 19 13

12 -8 3 19 13 9=P (1)

Como P (1) = 9 ≠ 0 ⇒ r= 1 no es raíz.

Ahora para r= -1, (divisor de la forma x+1)


12 -20 11 16 -6 -4

r=-1 -12 32 -43 27 -21


f 12 -32 43 -27 21 -25= P (-1)

Como P (-1) = -25 ≠ 0 ⇒ r=- 1 no es raíz.

Continuando con el proceso encontramos que r=- ½ es una raíz. (Es decir, existe
un divisor de la forma x +1/2)

Comprobando:
12 -20 11 16 -6 -4

r=-1/2 -6 13 -12 -2 4
j 12 -26 24 4 -8 0= P (-1/2)

Luego, P (x)= (x +1/2) ( 12𝑥 4 − 26𝑥 3 + 24𝑥 2 + 4𝑥 − 8) = 0

⇒ P (x)= (2x +1) ( 6𝑥 4 − 13𝑥 3 + 12𝑥 2 + 2𝑥 − 4) = 0 … (2)

(Corolario 2, teorema1)
El siguiente paso es tomar el polinomio de grado 4, que ha quedado como factor, y
se sigue el mismo proceso, es decir:

Sea Q(x)= 6𝑥 4 − 13𝑥 3 + 12𝑥 2 − 2𝑥 − 4


Haciendo nuevamente la prueba con r= -1⁄2
6 -13 12 2 -4

r=- 1⁄2 -3 8 -10 4


f 6 -16 20 -8 0= Q (-1⁄2)

Como Q (-1⁄2) =0 ⇒ r= -1⁄2 es raíz ⇒ un factor es x+1⁄2

⇒ Q(x) = (x +1/2) ( 6𝑥 3 − 16𝑥 2 + 20𝑥 − 8)

⇒ Q(x) = (2x +1) ( 3𝑥 3 − 8𝑥 2 + 10𝑥 − 4)

Reemplazamos Q(x) en (2):

P(x) = (2𝑥 + 1)2 (3𝑥 2 − 8𝑥 2 + 10𝑥 − 4)= 0 … (3)

(Corolario 3, teorema 1)

Ahora, tenemos M(x) = 3𝑥 2 − 8𝑥 2 + 10𝑥 − 4.

Haciendo nuevamente la prueba con r= -1⁄2, se tiene:

3 -8 10 -4

r=- 1⁄2 -3/2 19/4 -59/8


f 6 -19/2 59/4 -19/8= M (-1⁄2)

Como M (-1⁄2) =-91/8 ≠ 0 ⇒ r= -1⁄2 no es raíz.

Continuando con el proceso, trabajamos ahora con r = 2⁄3


3 -8 10 -4

r=- 1⁄2 2 -4 4
f 3 -6 6 0 = M (2⁄3)

Como M (2⁄3) = 0 ⇒ r = 2⁄3 es raíz ⇒ un factor es 𝑥 − 2⁄3

⇒ M(x) = (𝑥 − 2⁄3) ( 3𝑥 2 − 6𝑥 + 6)
⇒ M(x) = (3𝑥 − 2) ( 𝑥 2 − 2𝑥 + 2)= 0

Reemplazamos en (3):

P(x) = (2𝑥 + 1)2 (3𝑥 − 2) ( 𝑥 2 − 2𝑥 + 2)= 0 … (4)

(Corolario 3, teorema 1)

Ahora hacemos N (𝑥)= 𝑥 2 − 2𝑥 + 2 , igualamos a cero y resolvemos:

𝑥 2 − 2𝑥 + 2= 0 ⇒ 𝑥 + 1 ± 𝑖 ⇒ 𝑥1 = 1 + 𝑖 y 𝑥2 = 1 − 𝑖

(Teorema 4)

Luego, los divisores (factores) son [𝑥 − (1 + 𝑖 )] y [𝑥 − (1 − 𝑖 )]

Reemplazamos en (4)

P(x) = (2𝑥 + 1)2 (3𝑥 − 2) ( 𝑥 − 1 − 𝑖) ( 𝑥 − 1 + 𝑖) = 0

(Corolario 3, teorema 1)

Luego, las raíces son:


(2𝑥 + 1)2 = 0 ⇒ 𝑥 − 1/2 (multiplicidad k = 2)

(3𝑥 − 2) = 0 ⇒ 𝑥 = 2/3

𝑥−1−𝑖 =0 ⇒ 𝑥 =1+𝑖

𝑥−1+𝑖 =0 ⇒ 𝑥 =1−𝑖 conjugados. (Teorema 4)

Conclusión, las raíces son:

𝑥1 = 𝑥2 = −1/2 ; 𝑥3 = 2/3 ; 𝑥4 = 1 + 𝑖 ; 𝑥3 = 1 − 𝑖

Nota: Obsérvese que hemos tenido que trabajar con parte de la posible raíces
racionales de la forma r= p/q, por lo cual el método resulta muy laborioso. El único
fundamento para esto ha sido: conforme se encontró la raíz racional, se fue
bajando el grado del cociente Q(x) hasta llegar a un cociente de grado 2, el cual se
resolvió por los métodos conocidos.
b) como hemos llegado a P (x)= (2𝑥 + 1)2( 3𝑥1 − 2) ( 𝑥 − 1 − 𝑖) (𝑥 − 1 + 𝑖), lo cual
se puede escribir como P (x)= (2𝑥 + 1)2( 3𝑥1 − 2)[𝑥 − (1 + 𝑖)][𝑥 − (1 − 𝑖)],
donde los dos últimos factores son conjugados; entonces con el corolario 1 del
Teorema 4, multiplicamos los factores conjugados y obtenemos un factor
cuadrático con coeficientes reales:
[𝑥 − (1 + 𝑖)][𝑥 − (1 − 𝑖)] = 𝑥 2 -2𝑥 + 2

Luego, P(x) queda bajo la forma:

P(x) = (2x+1)² (3x-2) (x²-2x+2)


Pero este no es el polinomio original, pues este polinomio se obtuvo al multiplicar
el polinomio original por el mínimo denominador común. El polinomio original se
obtiene dividiendo el polinomio encontrado entre el mínimo denominador común.

Luego:

P(x) = 12x⁵- 10/3x⁴ + 11/6x³ +8/3x²-x- 2/3

P(x)= 1/6(2x+1)² (3x-2) (x²-2x+2)

TEOREMA 6 (Del valor intermedio)

Sea ƒ continua en [a;b].


Si ƒ(a)<0<ƒ(b) ó ƒ(b)<0<ƒ(a)

→ Ǝ c € <a;b>tal que ƒ(c) = 0


Y Y

ƒ(a)<0

a b
0 c b X a c
X
ƒ(a)<0 ƒ(b)<0
Nota: este teorema nos asegura que existe una raíz real r=c€<a;b> si es que ƒ(a).
ƒ(b)<0; pero nos indica que sea la única raíz en dicho intervalo.

Observación: Este teorema, junto con el anterior nos brinda uno de los métodos
más efectivos para el cálculo de la raíces de un polinomio o para descomponer
(factorizar) en factores lineales y cuadráticos. Para esto se recomienda lo
siguiente:

Para calcular las raíces racionales, (si es que existen) se aplica el teorema 6, con
lo cual se encuentra el intervalo <a;b> y después se toan únicamente las posibles
raíces racionales que caen dentro del intervalo del Teorema 6, con las cuales tiene
que hacer la prueba para comprobar si son o no raíces. Si ninguna de ellas
satisface, entonces diremos que existe una raíz real en dicho intervalo, la cual se
calcula por el método de aproximaciones sucesivas, como se verá más adelante.
Ejemplo. Dada la ecuación:

6x⁷ +11x⁶ -22x⁵ -37x⁴ +36x³ +56x² - 16x -16=0

Hallar las raíces, expresando las soluciones no reales en la forma r+si.

Solución:
Sea el polinomio

P(x) = 6x⁷ +11x⁶ -22x⁵ -37x⁴ +36x³ +56x² - 16x -16=0 … (1)

Como P(x) es de grado 7 raíces. Además como P(x) es de grado impar, entonces
por lo menos existe una raíz real

Aplicando teorema 5:

Como a₀ = 16 →divisores de a₀: p=±1; ±2; ±4; ±8; ±16

a₇= 6 → divisores de a₇: q= ±1; ±2; ±3; ±6

Posibles raíces racionales:


r=p/q = ±1; ±2; ±4; ±8; ±16; ±2/3; ±1/3; ±4/3; ±8/3; ±16/3; ±1/2; ±1/6
Aplicamos el teorema 6:
P(0) = -16<0 y P(1) = 18>0

Luego, en el intervalo <0; 1> existe por lo menos una raíz real r tal que P(r) = 0.
Las posibles raíces racionales en este intervalo son: r = 2/3; 1/3; ½; 1/6.

Haciendo la prueba con r = 2/3


6 11 -22 -37 36 56 -16 -16

R=2/3 4 10 -8 -30 4 40 16
6 15 -12 -45 6 60 24 0 = P(2/3)

Luego r = 2/3 es una raíz → x – 2/3 es un factor de P(x)

Reemplazando en (1)
P(x) = (x-2/3) (6x⁶ +5x⁵ -12x⁴ -45x³ +6x² + 60x +24) = 0

→ P(x) = (3x-2) (2x⁶ +5x⁵ -4x⁴ -15x³ +2x² + 20x +8) = 0

Ahora operamos con Q(x) = 2x⁶ +5x⁵ -4x⁴ -15x³ +2x² + 20x +8

Haciendo nuevamente la prueba con r = 2/3

2 5 -4 -15 2 20 8
R=2/3 4/3 38/9 4/24 -802/81 -1280/243 7160/486

2 19/3 2/9 -401/27 -604/81 3580/243 11048/486=Q(2/3)

Como Q(2/3) ≠0 → r = 2/3 no es raíz de Q (x). lo mismo sucede con r=1/3, r=1/2 y
r=1/6.
Aplicando nuevamente el teorema 6 para encontrar otro intervalo, (se puede
aplicar sobre P(x) ó Q(x)), donde se encuentre una raíz.
Como Q(0)= 8>0 y Q(-1)<0, entonces en <-1; 0> existe por lo menos una raíz real
r tal que Q(r)=0
Las posibles raíces racionales en <-1; 0> son r= -1/3, -2/3, -1/2, -1/6

Haciendo la prueba con r= -1/3:


2 5 -4 -15 2 20 8
R=1/3 -2/3 -13/19 49/27 -130/27 76/81 -1696/243

2 13/3 -49/9 130/9 -76/27 1696/81 Q(-1/3)≠0

→ r = - 1/3 no es raíz.
Operando ahora con r = -1/2

2 5 -4 -15 2 20 8
R=-1/2 -1 -2 3 6 -4 -8

2 4 -6 -12 8 16 0=Q(-1/2)

Luego r = -1/2 es una raíz → (x+1/2) es un factor de Q (x)

→ Q(x)=(x+1/2) (2x⁵ +4x⁴ -6x³ -12x² + 8x +16) = 0

→ Q(x)=(2x+1) (x⁵ +2x⁴ -3x³ -6x² + 4x +8) = 0

Luego reemplazamos Q(x) en (2):

P(x) = (3x-2) (2x+1) (x⁵ +2x⁴ -3x³ -6x² + 4x +8)=0 …


(3)
Continuando con el proceso encontramos que r = 2 es una raíz de-.

T(x) = x⁵ +2x⁴ -3x³ -6x² + 4x +8; (el grado de Q es impar)

Ejecutando con r = -2
1 2 -3 -6 4 8

R=-2 -2 0 6 0 -8
1 0 -3 0 4 0=T(-2)

→ T(x) = (x+2) (x⁴ -3x² + 4) = 0

T(x) Reemplazando en (3)

P(x) = (3x-2) (2x+1) (x+2) (x⁴ -3x² + 4) = 0


Continuando con la ecuación bicuadrada: U(x) = x⁴ -3x² + 4 = 0

Resolviendo se obtiene.

X²₁ = 3 + √7 i y X²₂ = 3 - √7 i
2 2

Las raíces conjugadas porque los coeficientes de Q(x) son reales.


Reemplazando U(x) en 4

P(x) = (3x-2) (2x+1) (x+2) X² = 3 + √7 i X² = 3 - √7 i = 0 … (5)


2 2

Calculamos las raíces de x² - 3 + √7 i = 0 que no tiene por qué ser conjugada.


2
x² = 3 + √7 i = 2 + √7 i; (se ubica en el primer cuadrante)
2 3 2

Pasando a la forma polar:

Modulo r = 2 ; cos ᾀ = ¾ Y sen ᾀ = √7/4

3 √7
⟹ 𝑥 2 = 2ℯ 𝑖𝛼 ; Donde cos 𝛼 = y sen 𝛼 =
4 4

Las raíces son:


𝛼+2𝜋 𝛼+2𝜋
𝑤𝑘 = 21⁄2 [cos ( ) + sen ( ) 𝑖] , 𝑘 = 0; 1
2 2

Para 𝑘 = 0:

𝑤0 = √2(cos 𝛼/2 + 𝑖 sen 𝛼/2) …(𝐼)

Para 𝑘 = 1:
𝛼+2𝜋 𝛼+2𝜋
𝑤1 = √2 [cos ( ) + sen ( ) 𝑖] …. (II)
2 2

=_√2(cos 𝛼/2 + 𝑖 sen 𝛼/2)


√7
Por trigonometría, si cos 𝛼 = 3/4 , sen 𝛼 = y 𝛼 perteneciente al primer
4
cuadrante, se tiene:
√7 1
cos(𝑎/2)= 2√2 y sen 𝑎/2 = 2√2

Luego (I) y (II):


√7 𝑖 √7 𝑖
𝑤0 = + 𝑦 𝑤1 = − [ + ]
2 2 2 2

3+√7𝑖
Entonces𝑥 2 − = 0 es equivalente a:
2

√7+𝑖 √7+𝑖
(𝑥 − ) (𝑥 + ) =0 …(III)
2 2

3+√7𝑖
En forma similar se calculan las raíces de 𝑥 2 − = 0, para encontrar su
2
equivalente:
−√7+𝑖 −√7+𝑖
(𝑥 − ) (𝑥 + )=0 …. (IV)
2 2

Reemplazando 3 y 4 en 5, se obtiene :
√7+𝑖 √7+𝑖 −√7+𝑖 −√7+𝑖
𝑃(𝑥) = (3𝑥 − 2)(2𝑥 + 1)(𝑥 + 2) (𝑥 − ) (𝑥 + ) (𝑥 − ) (𝑥 + )=0
2 2 2 2

√7+𝑖 √7+𝑖
Las raíces p(x) son:𝑥1 = 3⁄2; 𝑥2 = −1⁄2; 𝑥3 = −2; 𝑥4 = ; 𝑥5 = − ; 𝑥6 =
2 2
−√7+𝑖 −√7+𝑖
; 𝑥7 = − .
2 2

OBSERVACION GENERAL

El teorema 6 no nos asegura lo contrario en caso de ocurrir f(a). f(b) >0 no se


puede concluir que :

Si: 𝑓(𝑎). 𝑓(𝑏) > 0 → ∃𝑐 ∈ 〈𝑎; 𝑏〉 tal que 𝑓(𝑐) = 0

Tal como se puede observar gráficamente:


Y f(x)

f(b)

f(a)

0 a b X

En este caso: 𝑓(𝑎). 𝑓(𝑏) > 0 y es verdad que ∃𝑐 ∈ 〈𝑎; 𝑏〉 tal que 𝑓(𝑐) = 0

Y
f(b) f(x)

f(a)

0 a b X

En este caso: 𝑓(𝑎). 𝑓(𝑏) > 0 y es verdad que ∃𝑐 ∈ 〈𝑎; 𝑏〉 tal que 𝑓(𝑐) = 0.

 En este caso habría que analizar la naturaleza de 𝑓(𝑥) si fuera posible ,


hacer uso de las derivadas (criterios de primara y segunda derivadas ,
puntos de inflexión, etc)

Ejemplo dada la ecuación 9𝑥 7 + 10𝑥 5 + 82𝑥 3 + 9𝑥 = 6(𝑥 6 +𝑥 4 + 9𝑥 2 )

a) Encontrar sus soluciones reales

b) Expresar sus soluciones no reales en forma 𝑟 + 𝑠𝑖, donde 𝑖 − √−1 𝑦 𝑟, 𝑠 ∈


𝑅.
Solución
 La ecuación puede expresarse como:

𝑥[(9𝑥 6 +10𝑥 4 + 82𝑥 2 + 9𝑥) − 6(𝑥 5 + 𝑥 3 + 9𝑥)] = 0 …(1)

Donde 𝑥 = 0es una solución; es decir, 𝑥1 = 𝑜 es una raíz. Ahora tenemos

𝑄(𝑥) = (9𝑥 6 + 10𝑥 4 + 82𝑥 2 + 9) − 6(𝑥 5 + 𝑥 3 + 9𝑥 …(2)

ó 𝑄(𝑥) = 9𝑥 6 +6𝑥 5 + 10𝑥 4 − 6𝑥 3 + 82𝑥 2 − 54𝑥 + 9 ...(3)

Las posibles raíces racionales son𝑟 = 𝑝⁄𝑞 = ±1; ±3; ±9; ± 1⁄3; ± 1⁄9.

Aplicando el teorema 6

𝑄(0) > 9 𝑦 𝑄(1) = 44 > 0

Entonces tenemos 𝑄(0). 𝑄(1) > 0 y no podemos afirmar que no existe un 𝑟 ∈ 〈0; 1〉
tal 𝑄(𝑟) = 0. Si se continúa haciendo una prueba de las posibles raíces
racionales, se encontrara por ejemplo que:

𝑄(−1) > 9 𝑦 𝑄(1) = 44 > 0

Como Q(x) es algo complicado de derivar (criterios de primara y segunda


derivada, puntos de inflexión, etc.) Lo que haremos es analizar Q (x) en la
expresión (2).

Observando (2) concluimos que ningún numero negativo puede ser solución, que
ya que el primer paréntesis es positivo y el segundo es negativo, por lo tanto, la
diferencia es positiva y en consecuencia no puede ser igual a cero.

Usando la división sintética se comprueba que la única raíz racional es 𝑟 = 1⁄3,


(observar que 𝑟 ∈ 1⁄3 ∈ 〈−1; 0〉,entonces:

9 -6 10 -6 82 -54 9

1/3 3 -1 3 -1 27 -9
9 -3 9 -3 81 -27 0

1/3 3 0 3 0 27
9 0 9 0 81 0
Luego, 𝑥2 = 𝑥3 = 1⁄3 es una raíz de multiplicación 𝑘 = 2 , en consecuencia el
factor (𝑥 − 1⁄3)2 es de multiplicidad 2. El cociente que queda es :

𝑃(𝑥 ) = 9𝑥 4 + 9𝑥 2 + 81 = 0

La cual el una expresión bicuadrática. Resolviendo en forma similar al ejemplo


anterior, obtenemos:
1 1 1 1
𝑥4 = − (√5 + √7𝑖); 𝑥5 = − (√5 − √7𝑖); 𝑥6 = (√5 + √7𝑖); 𝑥7 = (√5 − √7𝑖)
2 2 2 2
 por lo tanto los raíces son:
1 1
𝑥1 = 0; 𝑥2 = 𝑥3 = 1⁄3; 𝑥4 = − (√5 − √7𝑖); 𝑥5 − (√5 − √7𝑖); 𝑥6
2 2
1 1
= (√5 + √7𝑖); 𝑥7 = (√5 − √7𝑖)
2 2

RAICES DE POLINOMIOS POR APROXIMACIONES SUCESIVAS

Consideramos el polinomio de grado n:

𝑃(𝑥) = 𝑎𝑛 𝑥 𝑛 + 𝑎𝑛−1 𝑥 𝑛−1 + ⋯ + 𝑎1 𝑥 + 𝑎0

Como se sabe, este polinomio tiene n raíces, entonces si r es la meyor raíz del
módulo, se tiene que:

r ‹ max 1; an –1 +… + a0
an an

esta expresión sirve como una localización de la raíces del polinomio


𝑃(𝑥)(considerar un circulo de radio r en el plano complejo).

Observación: se recomienda usar esta regla cuando el polinomio es complejo

REGLAS DE DESCARTES
El numero de raíces reales positivas, o bien es igual al número de cambios de
signo (N) de sucesiones: 𝑎𝑛 ; 𝑎𝑛−1 ; … ; 𝑎0 ó bien igual a N menos un numero par
Teniendo la sucesión en el orden indicado, los cambios de signo se presentan
cuando de un coeficiente positivo sea a uno negativo o viceversa. Para esto se
empieza de izquierda a derecha o de derecho a izquierda.

Ejemplo. Sea el polinomio 𝑃(𝑥) = 𝑥 4 − 10𝑥 3 + 32𝑥 2 + 38𝑥 + 15. La sucesión: 1: -


10; 32; -38; 15 tiene N=4 cambios de signo, entonces este polígono poseerá:

 𝑁 = 4raíces reales ó
 𝑁 − 2 = 2 raíces reales ó
 𝑁 − 4 = 0 raíces reales

Observación
Para el número de raíces negativas, se sigue el mismo procedimiento con los
coeficientes de P (-x)
Ejemplo

Sea el polinomio P (x) = x⁴ -5x³ +5x² + 5x -6. La sucesión: 1; -5; 5; 5; -6 implica


N=3; por lo tanto, este polinomio tiene 3 ó 1 raíz real positiva.

Para P (-x) = x⁴ -5x³ +5x² + 5x -6, la sucesión: 1; -5; 5; 5; -6 implica N = 1 en


consecuencia tiene una raíz real negativa. Siendo el número de raíces igual a 4
(teorema 3), el número de raíces complejas un numero para (teorema 4) y como
existe una raíz real negativa, entonces concluimos que existe al menos unas raíz
positiva.

REGLA DE BUDAN
Sean Nₐ y Nₑ el número de cambios de signos en las sucesiones.

P(a); P’(a);…; P⁽ᵑ⁾ (a)

P (b); P’ (b);…; P⁽ᵑ⁾ (b)

Respectivamente entonces el número de ceros o raíces de P(x) en [a; b] es igual


bien a (Nₐ - Nₑ) ó (Nₐ - Nₑ) menos un numero par.

Ejemplo

Del polinomio P(x)=x⁴- 10x³+ 32x²-18x+15, tomemos a=0 y b=2 luego evaluamos
los polinomios y las derivadas en a = 0 y b = 2

→ P(x)= x⁴- 10x³+ 32x²-18x+15 → P (0)=15; P(2)=43


P´(x)= 4x³- 30x²+64x-18 →P’(0)= -18; P’(2)=22

P’’(x)= 12x²-60x+64 → P’’(0)=64; P’’(2)= -8


P’’’(x)=24x-60 → P’’’(0)=-60; P’’’(2)= -12

P’’’’(x)=24 → P’’’’(0)=24; P’’’’(2)= 24


La sucesión es: 15; -18; 64; -60; 24 → Nₐ= 4
Na – Nb = 4 – 2 = 2
43; 22; -8; -12; 24 → Nₐ= 2
Luego en [0; 2] existen 2 (Nₐ - Nₑ), raíces reales o ninguna, (Nₐ - Nₑ - 2)

Nota: obsérvese que para aplicar esta regla es necesario darse valores de a y b
del intervalo [a; b]. Este intervalo se puede encontrar fácilmente si es que se elige
como el del teorema 6

Ejemplo

Dado x³ - x – 4 = 0. Hallar el número de raíces reales.

Solución

Las posibles raíces racionales son ±1; + 2; ± .4

Como el polinomio es de grado 3, entonces de la regla de Descartes,


concluimos:

•Hay una raíz real positiva y 2 negativa ó

• Hay una raíz real positiva y 2 raíces complejas.

Aplicando el Teorema 6, vemos que existe una raíz r e (1; 2) ya que /(I) • /(2) < O
; pero esto no quiere decir que ésta sea la única raíz en (1; 2} .

Aplicando la regla de Budan se confirma que existe exactamente una raíz r e (I ; 2),
la cual es positiva/Esta raíz se calculará por el método de Newton-Raphson, mediante
aproximaciones sucesivas.

TEOREMA

• Si los coeficientes de la ecuación:

anXᶯ + an-1Xᶯ-1+ … +a₁x+aₒ=0


Son positivos y, r y R denotan respectivamente el mínimo y el máximo de las
razones:

an-1; an-2; a₁; aₒ


an an-1 a₂ a₁

Entonces la ecuación no tiene raíces fuera de la región anular.

r≤IzI≤R

Ejemplo. Sea P4(x) = x4 +10x3 +32x2 +18 + 15 , entonces las razones son: 10 ; 3,2
; 0,5625 ; 0,83. Luego, r = 0,5625 y R = 10. Por lo tanto, r < |z| < R , esto indica que
las raíces están en [-10 ; -0,5625] o en [0,5625 ; 10].

Nota: Obsérvese que este teorema es de bastante utilidad sí es que


trabajamos con polinomios de variable, real.

MÉTODO DE NEWTON-RAPHSON

Uno de los métodos más usados para resolver ecuaciones es el método de Newton-
Raphson, debido a su gran velocidad para obtener una buena aproximación a la
raíz cuando un valor inicial es elegido cercano a la raíz exacta.

La figura da una descripción exacta, y para esto consideramos la siguiente hipótesis:


"La función/es continua en [a0 ; b0] y f(a0) • f(b0) < O".

Partiendo de una estimación inicial x0, trazamos una recta tangente a la curva en
el punto (x0; /(x0)) y la intersección de esta recta con el eje X es la nueva
aproximación a la raíz de la ecuación. La repetición del proceso lleva al método de
Newton-Raphson:

Xk+1= Xk – ƒ (Xk)
Ƒ’ (Xk)

Donde Xk es la K-ésima aproximación.


Y

Ƒ(x)

0 Xk+1 Xk X

Nota: Para la estimación inicial Xₒ, esta puede elegirse como:

Xₒ = a + b
2

Ejemplo

En la función ƒ(x)= x³-x-4


Cuando sabemos que la raíz positiva esta en [1; 2] tomamos xₒ = 1.5 como valor
inicial y empezamos el proceso iterativo.

K Xk Ƒ(Xk) Ƒ’ (Xk)

0 1.5 -2.125 5.75

1 1.8695625 0.6650774 9.485822

2 1.7994498 0.0272037 8.71400585

3 1.796328

Cuando observamos, en solamente 3 iteraciones hemos obtenido una


aproximación a la raíz con 3 cifras significativas exactos: 1.79.
PROBLEMAS RESUELTOS
OPERACIONES CON NÚMEROS COMPLEJOS - FORMA CARTESIANA
PROBLEMA 1.
a) Hallar 2 números complejos z1 y zz cuya suma sea el número real x y cuya
diferencia sea el número imaginario y i.'
b) Hallar todos los números complejos cuya suma es 4 y su producto es
8.

Solución:

a) De la condición del enunciado:

z₁ + z₂ = x
z₁ - z₂ = iy
Sumando (1) y (2):

2zl= x + iy → z₁ = x/2 + iy/2

Restando (2) de (1):

2z2 = x – iy → z₂= x/2 – iy/2

b) Sean zx , z2 los números que debemos hallar.


Por la primera condición:
z1+z2=4 ... (3)

Para que la ecuación (3) se cumpla, los números complejos deben ser conjugados,
es "decir:
z 1 =(a + ni) y z2 = (b-ni); ... (4)

Luego se tiene:

( a + ni ) + ( b – ni ) = 4 → a + b=4 … (5)

Por la segunda condición:

z₁ z₂ = 8

(a+ni) (b-ni)= 8

ab + n² + ( nb – na )i = 8 … (6)
Para que la ecuación (6) se cumpla el coeficiente de "i" debe ser nulo (0), es
decir: •
nb - na = 0 => a = b ... (7)
Luego de (7) en (5):
a + (a) = 4 => a=b = 2 ...(8)

Luego reemplazando (8) en (6), se obtiene:


(2) (2)+ n 2+(0) = 8 => n= ± 2

Finalmente reemplazando los valores de n, a y b en (4), se obtiene

z₁ = 2+2i y z₂ = 2-2i ó z₁ = 2-2i y z₂ = 2+2i

Resumiendo los números complejos serán: 2 + 2 y 2 - 2i

PROBLEMA 2
Hallar todos los valores reales de x y los correspondientes de w que hacen que el
numero complejo w = (x-i) [(x+3)-9i] sea imaginario puro.
Solución:

Si w es un número imaginario puro, entonces w = iy, luego:


W = yi = x(x+3)-i (x+3) – 9xi + 9i²

yi = (x²+3x-9) – (10x + 3)I … (1)


Para que w sea un número imaginario puro la expresión (I), de la ecuación, que
representa a la parte real debe ser nula:

x₁ = -3+3 √5/2

x²+3x-9 = 0

x₂= -3-3√5/2

Por comparación de (II) en la ecuación (1) se tiene:


Y = -10x-3

Reemplazando los valores de x₁ y x₂ en (2)

y₁ = -10 (-3+3√5/2) -3 = 12-15√5 → w₁ = (12-15√5)i


y₂ = -10 (-3-3√5/2) -3 = 12+15√5 → w₂ = (12+15√5)i

PROBLEMA 3. Demostrar

a) Si z₁ + z₂ = z₁ + z₃ entonces z₂ = z₃ (Propiedad de cancelación para la


suma)
b) Si z₁ ≠ 0 y si z₁ z₂ = z₁ z₃ entonces z₂ = z₃ (propiedad de cancelación para
la multiplicación)
c) Si z₁ z₂ = 0 entonces z₁ = 0 ó z₂ = 0

Solución:

a) Como z₁ £ C → Ǝ - z₁ £ C tal que z₁ + (-z₁) = -z₁ + z₁ = 0 = (0; 0).


Luego - z₁ + (z₁ + z₁) = - z₁ + (z₁ + z₃)
→ (-z₁ + z₁) + z₂ = (-z₁ + z₁) + z₃
→ 0 + z₂ = 0 + z₃ → z₂ = z₃
b) Como z₁ ≠ 0 → Ǝ - z₁¯¹ = 1; luego
z₁¯¹ (z₁ z₂) = z₁¯¹ (z₁ z₃) → (z₁¯¹ z₁) z₂ = (z₁¯¹ z₁) z₃
1. z₂ = 1. z₃ → z₂ = z₃
c) supongamos que z₁ ≠ 0 entonces Ǝ z₁¯¹; luego
z₁¯¹ (z₁ z₂) = z₁¯¹ (0) → 1z₂ = 0 → z = 0
En forma análoga se muestra que z₁ = 0 suponiendo que z₂ ≠ 0

CONJUGADO Y MODULO DE UN NUMERO COMPLEJO


PROBLEMA 4 Demostrar

a) Iz₁ + z₂I ≤ I z₁I + Iz₂I


b) ¿En qué caso se cumple la igualdad?

Solución:

a) Partimos del miembro de la izquierda para efectuar la demostración:


I z₁ + z₂I² = (z₁ + z₂) (z₁ + z₂)
= (z₁ + z₂) (z₁ + z₂)
= Iz₁I² +z₁ z₂ + z₁ z₂ + Iz₂I²
=Iz₁I² + Iz₂I² + z₁ z₂ + z₁ z₂
= Iz₁I² + Iz₂I² + 2 Re (z₁ z₂)

Pero se tiene:

z1  z2  2 Re  z1 z2   z1  z2  2 z1 z2  z1  z2  2 z1  z2
2 2 2 2 2 2

Entonces:
Iz₁ + z₂I² ≤ (Iz₁I + Iz₂I)²

Iz₁ + z₂I ≤ Iz₁I + Iz₂I


Nota: Observe que Re (z1 z2) ≤ |za| Iz2I, ya que Re(z₁ z2) ≤ IRe (zl z2 )| < |za z2|,
(propiedad 2 de módulo de un número complejo).
b) Sean los números complejos:
z₁ = x₁ + iy₁ z₂ = x₂ + iy₂
La igualdad se cumple si y solo si:
Re (z₁ z) = Iz₁I Iz₂I
Luego z₂ + x₂ - iy₂, entonces
z₁ z₂ = (x₁ + y₁i) (x₂ - y₂i)
= (x₁ x₂ + y₁ y₂) + (x₂ y₁ - x₁ y₂)i
→ Re (z₁ z₂) = x₁ x₂ + y₁ y₂
Además se tiene:
Iz₁I = √x₁² + y₁² y Iz₂I = √x₂² + y₂²
Reemplazando (3) y (4) en (2)
x₁ x₂ + y₁ y₂ = √x₁² + y₁² √x₂² y₂²
x₁² + y₂² + x₂² y₁² - 2x₁ y₁ x₂ y₂ =0

(x₁ y₂ - x₂ y₁)² = 0

x₁ y₂ = x₂ y₁
𝑥₁ 𝑥₂
→ 𝑦₁ = 𝑦₂ = k ; (y₁ ≠ 0; y₂ ≠ 0)

Ahora si a esta última expresión lo igualamos a una constante:


𝑥₁ 𝑥₂
= 𝑦₂ = k → x₁ = ky₁ y x₂ = ky₂
𝑦₁

Luego reemplazando estos valores en (1):

𝒛𝟏 = 𝒌𝒚𝟏 + 𝒊𝒚𝟏 = 𝒚𝟏 (𝒌 + 𝒊) → I𝒛𝟏 I = I𝒚𝟏 I √𝒌𝟐 + 𝟏

𝒛𝟐 = 𝒌𝒚𝟐 + 𝒊𝒚𝟐 = 𝒚𝟐 (𝒌 + 𝒊) → I𝒛𝟐 I = I𝒚𝟐 I √𝒌𝟐 + 𝟏

→ 𝒛𝟏 + 𝒛𝟐 = (𝒚𝟏 + 𝒚𝟐 ) (𝒌 + 𝒊) → |𝒛𝟏 + 𝒛𝟐 | = |𝒚𝟏 + 𝒚𝟐 | √𝒌𝟐 + 𝟏

Luego:

|𝒛𝟏 + 𝒛𝟐 | = |𝒛𝟏 | + |𝒛𝟐 |


→ |𝒚𝟏 + 𝒚𝟐 | √𝒌𝟐 + 𝟏 = |𝒚𝟏 | √𝒌𝟐 + 𝟏 + |𝒚𝟐 | √𝒌𝟐 + 𝟏

→ |𝒚𝟏 + 𝒚𝟐 | = |𝒚𝟏 | + |𝒚𝟐 |

Esta última igualdad se cumple si 𝑦1 y 𝑦2 son ambos positivos o ambos negativos.

Conclusión: La igualdad |𝑧1 + 𝑧2 | = |𝑧1 | + |𝑧2 | se cumple si y solo si 𝑥1 / 𝑦1 =


𝑥2 / 𝑦2 donde 𝑦1 y 𝑦2 son ambos positivos o ambos negativos y, 𝑥1 y 𝑥2 son
números reales.

Ejemplo. Sean y ₁ = -4 y y 2 = -10, (ambos negativos) y como x l /y l = x2 /y 2 ,


entonces elegimos: Xj =3 x2 =15/2
Luego:
z₁ = 3 - 4i y z₂ = 25/2; Iz₁ + z₂I = 35/2
Se comprueba que: Iz₁ + z₂I = Iz₁I + Iz₂I
PROBLEMA 5. Demostrar: │z 1 – z2│≥ ││z1 -│z2││, ¿en qué caso se cumple la
igualdad?
Solución:
Partimos del artificio y propiedad de desigualdades:
│z1│=│z1 – z2 +z2│ ≤ │z1 – z2│+ │z2│ │z1│- │z2│ ≤ │z1 – z2│ … (1)
│z2│=│z2 – z1 +z1│ ≤ │z2 – z1│+ │z1│ │z2│- │z1│ ≤ │z1 – z2│ … (2)

De (1): │z1│- │z2│ ≤ │z1 – z2│


De (2): │z1│- │z2│ ≥ │z1 – z2│

De estas dos últimas desigualdades:


-│z1 – z2│≤ │z1│- │z2│ ≤ │z1 - z2│

Luego el valor absoluto de:


││z1│- │z2││ ≤ │z1 – z2 │ … (3)
Veamos qué caso se cumple la igualdad:

│z1 – z2│ = √(x1 – x2)2 + (y1 – y2)2


│z1│ = √ x12 + y12 ; │z2│ = √ x22 + y22

Reemplazando en (3), elevando al cuadrado y simplificando, se llega a:


(x2y1 – x1y2)2 ≥ 0

La igual se cumple si y solo si x2 y1 – x1 y2 = 0, esto de:

x1 = x2 ; (y1 ≠ 0 ; y2 ≠ 0)
y1 y2

Ahora, sea:
x1 = x2 =k x1 = ky1 y x2 = ky2

y1 y2
Luego se tiene:

z1  ky1  y1i  y1 (k  i )  z1  y1 k 2 1

z 2  ky2  y 2 i  y 2 (k  i )  z 2  y 2 k 2 1

 z1  z 2  ( y1  y 2 ) (k  i )  z1  z 2  y1  y 2 k 2 1

Luego:

z1  z 2  z1  z 2   y1  y 2  y1  y 2 k 2  1  y1  y 2 k 2 1

 y1  y 2  y1  y 2

Esta última igualdad se verifica si y1 e y2 son ambos positivos o ambos negativos.

Conclusión: La igualdad z1  z 2  z1  z 2 se cumple si sólo si x1/y1 = x2/y2 donde

y1 y y2 son ambos positivos o ambos negativos.

Como ejemplo tomar z1=3 - 4i y z2 = 15/2 – 10i del ejemplo anterior y comprobar

que: z1  z 2  z1  z 2

PROBLEMA 6. Demostrar:
2 2

a) z1  z 2  z1  z 2  2 z1  z 2
2 2

2

b) z1  z 2  z1  z 2
2 2

Solución:

a) Como:

z1  z 2  z1  z 2  z1  z 2 
2

 z1  z 2  z 1  z 2 

 z1 z 1  z1 z 2  z 1 z 2  z 2 z 2

También:

z1  z 2   z1  z 2  z1  z 2 
2

 z1 z 1  z1 z 2  z 1 z 2  z 2 z 2

Luego, sumando (1) y (2) obtenemos:

z1  z 2  z1  z 2  2z1 z 2  z 2 z 2 
2 2


 2 z1  z 2
2 2

b) Aplicando la desigualdad triangular:

z1  z 2  z1   z 2   z1   z 2  z1  z 2

 z1  z 2  z1  z 2

Re( z )  Im( z )
PROBLEMA 7. Probar que:  z  Re( z )  lm( z )
2
Solución:Sea: z = x + (y, donde: x = Re(z) y y = lm(z). El problema equivale a

x y
 x2  y2  x  y
2
probar que: Para (  ) sabemos que
( )

(x - y)2  0 ;  x, y  R, entonces : x  y   0
2

x 2  y 2  2 xy

2 x 2  2 y 2  x 2  y 2  2 xy

 
 2 x 2  y 2  x  y   0

2 x2  y2  x  y

x y x y
 x2  y2  
2 2

Re( z )  lm( z )
 z 
2

Para   : sabemos que x y  0 ;  x, y  R, entonces

x y 0
x2  y2  2 x y  x2  y2
2x y 0

x 2  y 2  2 xy  x 2  y 2

x  y 2
 2 x y  x2  y2 
x  y  2
 x2  y2  0

x  y  x2  y2

 Re( z )  lm( z )  z
Entonces, de (1) y (2)

Re( z )  Im( z )
 z  Re( z )  Im( z )
2

PROBLEMA 8. Hallar z. Tal que:

a) z  z  1 2i
b) z  z  2i

2 Solución

Sea el número complejo: z  x  iy

a) En la expresión z  z  1  2i :
x  iy  ( x  iy )  1  2( x  iy )

x 2  y 2  ( x  iy )  1  2( x  iy )

 x 2  y 2  (1  x)  i ( x  iy )
2 y 2  2 x  3  4 y  (4  4 x  2 y  2 xy)i

Igualando la parte imaginaria

0=4+4x+2y+2yx  y  2

Igualando la parte real

2 y 2  2 x  3  4 y

2(2)2  2 x  3  4(2)

 x  3/ 2
Luego reemplazando (2) y (3) en (1)

z  3 / 2i

b) En la expresión z  z  2  i :
x  iy  ( x  iy )  2  i

x 2  y 2  x  iy  2  i

 x 2  y 2  2  x  (1  y )i
2 y 2  3  4 x  2 y  (4  2 x  4 y  2 xy)i

Igualando la parte real:


z 1  2 z 1 y

r  4/3

5/ 3 0 x

Elevando al cuadrado y simplificando, se tiene:


x2 y 2
 1 ….(*)
4 3
Los puntos z  ( x; y )  x  iy que satisfacen la desigualdad son los puntos que
pertenecen a la elipse determinada por(*) y los puntos interiores a ella. Los cuales
se muestran en la siguiente figura:

z  1  4  z 1
0
-2 2 x

 3
PEOBLEMA 11. Hallar z tal que: 3z  2  1 y

Solución: 3z  2  1

Sea z  x  iy , luego reemplazando se tiene:


r 1 3

 3x  2    3 y 2   1 2 3 0 x

Elevando al cuadrado y simplificando se llega a:

2
 2 1
x   y 
2

 3 9

Si tomamos z0   2 3    2 3;0 , la desigualdad

anterior se puede expresarse como:

z 2 3  1 3

Luego los puntos z  ( x; y )  x  iy que satisfacen la desigualdad, son los puntos


interiores a la circunferencia de radio r  1 3 y centro en z0    2 3;0 , tal como se
muestra en la figura.

PROBLEMA 12. Hallar z tal que:

z2 z2 z2


a) 2 b) 2 c) 2
z2 z2 z2
solución:

a) Sea z  x  iy , y considerando que:


z2
2 z 2  2 z 2
z2
Luego reemplazando se tiene:
y

z2
 x  2  y2  2  x  2  y2
2 2
2
z2
Elevando al cuadrado y simplificando:

(
10
; 0) 0 x
3

 x  10 / 3  y 2  64 / 9
2

Luego, los puntos z  ( x; y )  x  iy que


satisfacen la desigualdad, son aquellos
que están en la circunferencia de centro
zo   10 / 3;0 y radio r  8 3 , tal como se

muestra en la figura.

z2
b) Para la expresión  2 , análogamente se obtiene:
z2
y

z2
 x  10 / 3  y 2  64 / 9
2
2
z2

Luego los puntos z  ( x; y )  x  iy que


satisfacen la desigualdad, son aquellos
que están en el interior de la circunferencia
de radio r  8 3 y centro zo   10 / 3;0 . (
10
3
; 0) 0 x
z2
c) Para la expresión  2 ,análogamente:
z2
y

 x  10 / 3  y 2  64 / 9
2

Los puntos z  ( x; y )  x  iy que satisfacen


la desigualdad, son aquellos que están
en el exterior de la circunferencia de radio
r  8 3 y centro zo   10 / 3;0 . (
10
; 0) 0 x
3

z2
2
z2

PROBLEMA 13. Sea p  0 y p  1

1 z
a) Probar que  p representa una circunferencia.
1 z
b) ¿Qué representa la igualdad de la parte (a), si p  1 ?

Solución:

1 z
a) Sea z  x  iy y considerando que: p 1 z  p 1 z
1 z
Luego, reemplazando se tiene:
1  x   y 2  p 1  x   y 2
2 2

De donde:

 p2  1  2 4 p2
 x    y 
p2 1   p 2  1
2

 p2  1 
Lo cual representa una circunferencia de centro  ;0  y radio
 1 p
2

2p
r 2
p 1

b) Si p  1 . Reemplazando en (1):

1  x   y 2  p 1  x   y 2
2 2

1  x2  2x  y 2  1  x2  2x  y 2
x0

Luego, z  0  iy  iy representa un numero imaginario puro.

1  ti
PROBLEMA 14. Sea z  1 ,con z  1 . Demostrar que z  ; t R
1  ti

Solución:

Sea z  x  iy  1  x  1

Como z  1  z  1 , entonces:
2

x2  y 2  1  x2  1  y 2 ………(*)

2t
Tomemos y  , el cual es un número real. Luego reemplazando en (*)
1 t2

x2  1 
4t 2
 x2 
1  t 
2 2

1  t 2 2
1  t 
2 2
De donde:

1 t2 1 t2 2t 1  ti
x1   z   
1 t 2
1 t 1 t
2 2
1  ti
t 2 1 t 2 1 2t
x2   z   ; (Se descarta, pues para)
1 t 2
1 t 1 t2
2
t  0  z  1

PROBLEMA 15. Hallar todos los z  C tales que: Im  z  1 z    0

Solución:

Sea z  x  iy , de modo que z  0 ;lo cual implica que x 2  y 2  0 .


Luego:

1 z x  iy
 2  2
z z x  y2

1 x  yi
 z  x  yi  2
z x  y2

x3  xy 2  x y  y  y 
2 3

 2  i
x  y2 x2  y 2
Entonces:

1  x 2 y  y 3  y y  x  y  1
2 2

Im  z    
 z x2  y 2 x2  y 2
 y  x 2  y 2  1  0  x 2  y 2  0

  y  0  x2  y 2  1  0   x2  y 2  0

  y  0  x2  y 2  0   x2  y 2 1  0  x2  y 2  0

  x 2  0    x 2  y 2  1

  x  0    x 2  y 2  1
Luego, los z  x  yi que satisfacen, son los x  ( x;0) / x  0 ó z  ( x; y) / z  1 .

Geométricamente, vemos que los x  ( x;0) / x  0 son los números


reales(exceptuando al cero) y los z  ( x; y) / z  1 son aquellos que están en la
circunferencia de centro z0  (0;0) y radio r  1 .

La interpretación geométrica es:

r =1

FORMA POLAR DE UN NÚMERO COMPLEJO


PROBLEMA 16.Expresar en su forma polar los siguientes números complejos:

a) −10 + 10𝑖
b) 10 − 10𝑖
c) 8
d) −8
e) √2 + √6𝑖
f) −5𝑖
g) 5𝑖
Solución:

a) 𝑧 = −10 + 10𝑖 |𝑧| = 𝑟 = 10√2


10 3𝜋 7𝜋
𝜃 = 𝐴𝑟𝑔 𝑧 = 𝑎𝑟𝑐 tan = 𝑎𝑟𝑐 tan(−1) {𝜃1 = ó 𝜃2 =
−10 4 4
Para saber cual de los valores de 𝜃 tomamos como argumento, graficamos 𝑧 .

De la figura, 𝜃 pertenece al segundo cuadrante.

Luego: y
3𝜋 3𝜋 3𝜋
𝜃= 𝑦 𝑧 = 10√2(cos + 𝑖 sin )
4 4 4
10
3𝜋
𝑖( )
O también:𝜃 = 10√2𝑒 4
3𝜋
𝜃=
b) 𝑧 = 10 − 10𝑖 𝑟 = 10√2 4
7𝜋
𝜃 = 𝐴𝑟𝑔(𝑧) = 𝑎𝑟𝑐 tan
−10
= 𝑎𝑟𝑐 tan(−1) x
𝜃= - 10 0 y
10 4
c10
De la figura: 𝜃 =
7𝜋
. 0
4
-
Luego:
10
7𝜋 7𝜋
𝑧 = 10√2(cos + 𝑖 sin )
4 4
7𝜋
x
𝑖( )
O también: 𝑧 = 10√2𝑒 4

c) 𝑧 = 8 + 0𝑖 𝑟=8
0
𝜃 = 𝑎𝑟𝑐 tan 8 = 𝑎𝑟𝑐 tan 0
y
De la figura:𝜃 = 0

Luego:

𝑧 = 8(cos 0 + 𝑖 sin 0)
𝜃=0
𝑖0 0
O también: 𝑧 = 8𝑒 8 x
d) 𝑧 = −8 + 0𝑖 𝑟=8
0
𝜃 = 𝑎𝑟𝑐 tan −8 = 𝑎𝑟𝑐 tan 0
y
De la figura: 𝜃 = 𝜋

Luego:
𝜋 𝜋
𝑧 = 2√2(cos 3 + 𝑖 sin 3 ) 𝜃=𝜋

-8 0 x
O también: 𝑧 = 8𝑒 𝑖𝜋

e) 𝑧 = √2 + √6𝑖 𝑟 = 2√2

√6
𝜃 = 𝑎𝑟𝑐 tan
√2
= 𝑎𝑟𝑐 tan √3 y
𝜋 √6
De la figura:𝜃 = 3

Luego:
𝜋
𝜋 𝜋 𝜃=
3
𝑧 = 2√2 (cos 3 + 𝑖 sin 3 )
√2 x
𝜋
𝑖
O también: 𝑧 = 2√2𝑒 3

f) 𝑧 = 0 − 5𝑖 𝑟=5
x
Conviene usar: 3𝜋
𝜃=
5
2
𝜃 = 𝑎𝑟𝑐 sin(− 5) = 𝑎𝑟𝑐 sin(−1)
y
3𝜋
De la figura: 𝜃 = 2
-5
Luego:
3𝜋 3𝜋
𝑧 = 5 (cos + 𝑖 sin )
2 2
O 𝑧 = 5𝑒 𝑖3𝜋/2

g) 𝑧 = 0 + 5𝑖 𝑟=5
5
𝜃 = 𝑎𝑟𝑐 sin 5 = 𝑎𝑟𝑐 sin(1)
x
𝜋
Se tiene:𝜃 = 2
5
Luego:
𝜋 𝜋
𝑧 = 5(cos 2 + 𝑖 sin 2 ) 𝜋
𝜃=
2
𝜋
O también: 𝑧 = 5𝑒 𝑖 2 y
PROBLEMA 17.

a) Exprese el siguiente número complejo en su forma polar.

3 − 4𝑖
𝑧 =1−𝑖+
(3 − √3) − (1 + 3√3)𝑖
1 − √3𝑖 3−𝑖
b) Si 𝑧 𝜖 ℂ , demostrar que:( 𝑧 )2 = (𝑧 2 )

Solución:
a) Operando la expresión:

3 − 4𝑖
𝑧 =1−𝑖+
(3 − √3) − (1 + 3√3)𝑖
1 − √3𝑖 3−𝑖
3 − 4𝑖
=1−𝑖+
[(3 − √3) − (1 + 3√3)𝑖](3 + 𝑖)
1 − √3𝑖
(3 − 𝑖)(3 + 𝑖)
3 − 4𝑖
=1−𝑖+
1 − √3 + (1 − √3𝑖)

3 − 4𝑖 (1 + √3𝑖)
=1−𝑖+ ×
2(1 − √3𝑖) (1 + √3𝑖)

3 + 4√3 + (3√3 − 4)𝑖


=1−𝑖+
8
11 + 4√3 3√3 − 12
= + 𝑖
8 8

z= 2,397𝑒 𝑖𝛼

𝛼 = 𝑎𝑟𝑐 tan(−0,363)

b) 𝑧 = 𝑟𝑒 𝑖𝜃 𝑧 = 𝑟𝑒 −𝑖𝜃

Luego:

𝑧 2 = 𝑟 2 𝑒 2𝑖𝜃 (𝑧 2 ) = 𝑟 2 𝑒 −2𝑖𝜃 y (𝑧)2 = 𝑟 2 𝑒 −2𝑖𝜃

(𝑧)2 = 𝑟 2 𝑒 −2𝑖𝜃 = (𝑧 2 )
ARGUMENTO DE UN NÚMERO COMPLEJO

PROBLEMA 18. Establezca una condición entre los argumentos de 2 números 𝑧1


y 𝑧2 que cumple la igualdad en la desigualdad triangular.

Solución:
Por condición del problema:

|𝑧1 + 𝑧2 | = |𝑧1 | + |𝑧2 | 𝑅𝑒(𝑧1 𝑧2 ) = |𝑧1 ||𝑧2 | …(*)

Sean 𝑧1 = |𝑧1 |𝑒 𝑖𝜃1 y 𝑧2 = |𝑧2 |𝑒 𝑖𝜃2 𝑧2 = |𝑧2 |𝑒 −𝑖𝜃2

Por lo tanto:

𝑧1 𝑧2 = |𝑧1 ||𝑧2 |𝑒 𝑖(𝜃1 −𝜃2 ) 𝑅𝑒 (𝑧1 𝑧2 ) = |𝑧1 ||𝑧2 |cos(𝜃1 −𝜃2 )

Remplazando en (*):

|𝑧1 ||𝑧2 |cos(𝜃1 −𝜃2 ) = |𝑧1 ||𝑧2 | cos(𝜃1 − 𝜃2 ) = 1

𝜃1 − 𝜃2 = 2𝑘𝜋

𝜃1 = 𝜃2 + 2𝑘𝜋

Luego: 𝐴𝑟𝑔(𝑧1 ) = 𝐴𝑟𝑔(𝑧2 ) + 2𝑘𝜋

PROBLEMA 19. Demostrar que los módulos de 2 números complejos no nulos,


cuyos argumentos se diferencian en 𝜋 + 2𝑘𝜋 , 𝑘 entero, verifican la desigualdad
estricta en la desigualdad triangular.

Solución:

Según el problema anterior, la desigualdad estricta se verifica en la desigualdad


triangular si y solo si:

𝐴𝑟𝑔(𝑧1 ) − 𝐴𝑟𝑔(𝑧2 ) ≠ 2𝑘𝜋


Efectuaremos la demostración para el caso particular que indica el problema, es

decir:

𝐴𝑟𝑔(𝑧1 ) − 𝐴𝑟𝑔(𝑧2 ) = 𝜋 + 2𝑘𝜋 ≠ 2𝑘𝜋

Vemos:|𝑧1 + 𝑧2 | = |𝑧1 | + |𝑧2 | ↔ 𝑅𝑒(𝑧1 + 𝑧2 ) < |𝑧1 ||𝑧2 | …(1)

Sean: 𝑧1 = |𝑧1 |𝑒 𝑖𝜃1 y 𝑧2 = |𝑧2 |𝑒 𝑖𝜃2 → 𝑧2 = |𝑧2 |𝑒 −𝑖𝜃2

→ 𝑧1 𝑧2 = |𝑧1 ||𝑧2 |𝑒 𝑖(𝜃1 −𝜃2 ) …(2)

Pero: 𝜃1 − 𝜃2 = 𝜋 + 2𝑘𝜋

Luego, en (2):

𝑧1 𝑧2 = |𝑧1 ||𝑧2 |𝑒 𝑖(𝜋+2𝑘𝜋) …(3)

También:

𝑒 𝑖𝜋 = cos 𝜋 + 𝑖 sin 𝜋 = −1

𝑒 𝑖(2𝑘𝜋) = cos(2𝑘𝜋) + 𝑖 sin(2𝑘𝜋) = 1

Luego, en (3):

𝑧1 𝑧2 = |𝑧1 ||𝑧2 |𝑒 𝑖(𝜋+2𝑘𝜋)

=|𝑧1 ||𝑧2 |𝑒 𝑖𝜋 . 𝑒 𝑖(2𝑘𝜋)

= −|𝑧1 ||𝑧2 |

Entonces: 𝑧1 𝑧2 = −|𝑧1 ||𝑧2 | → 𝑅𝑒(𝑧1 𝑧2 ) = −|𝑧1 ||𝑧2 |

Remplazando en (1):

|𝑧1 + 𝑧2 | = |𝑧1 | + |𝑧2 | ↔ 𝑅𝑒(𝑧1 𝑧2 ) < |𝑧1 ||𝑧2 |

→ −1 < 1

z
PROBLEMA 20. Si |z| ≠0, probar que: ||z|| ≤ |argz|

Solución:

Sea z = 𝑟𝑒 𝑖θ ; r≠0 ; θ = argz


En la condición:

𝑟𝑒 𝑖θ
| − 1| = |𝑒 𝑖θ − 1|
𝑟

Probaremos que: |𝑒 𝑖θ − 1| ≤ |θ| y


En efecto, 𝑄 = (cos 𝜃 ; sin 𝜃)

|𝑒 𝑖θ − 1| = √(𝑐𝑜𝑠θ − 1)2 𝑠𝑒𝑛2 θ 𝑟=1

𝜃 𝑝(1; 0)
=√2 − 2𝑐𝑜𝑠θ x

Grafiquemos z

Como:

|𝑒 𝑖θ |=|𝑐𝑜𝑠θ + isenθ| = √(𝑐𝑜𝑠 2 θ + 𝑠𝑒𝑛2 θ=1

De la figura se tiene que:

|𝑄𝑃| ≤ θ

|𝑄𝑃|=√(𝑐𝑜𝑠θ − 1)2 + 𝑠𝑒𝑛2 θ=√2 − 2𝑐𝑜𝑠θ ≤ θ ≤ |θ|

Luego:

z
| − 1| ≤ |𝑟𝑒 𝑖θ − 1| ≤ |θ| = |arg(z)|
|z|

PROBLEMA 21. Si z ≠ 0, demostrar que:

|z-1| ≤ ||z| − 1|+|z||arg(z)|

Solución:

Como

z |𝑧 − |𝑧||
| − 1| ≤ |arg(z)| ⇒ ≤ |arg(z)|
|z| |z|

|𝑧 − |𝑧|| ≤ |𝑧||arg(z)| ………………………………………………………………(1)


Pero:

|𝑧 − |𝑧|| = |(𝑧 − 1) − (|𝑧| − 1)| ≥ ||𝑧 − 1| − ||𝑧| − 1|| ≥ |𝑧 − 1| − ||𝑧| − 1| ……(2)

(2) en (1):

|𝑧 − 1| − ||𝑧| − 1| ≤ |𝑧||arg(z)| ⇒ |𝑧 − 1| ≤ ||𝑧| − 1|+|z||arg(z)|

Verificar que la igualdad se cumple si y solo si z es un número real positivo.

PROBLEMA 22. Dado w= -2+2√3 i, hallar un complejo z tal que 2|𝑧| = |𝑤| y que

la diferencia entre su amplitud y la de w sea 90º.

Solución:
7𝜋
𝑖( )
Pasando w a la forma polar, tenemos que: w=4𝑒 6

|𝑤| = 4 , entonces |𝑧| = 2

Ahora sea z=2𝑒 iθ . De la condición del problema, vemos que se presentan dos

casos respecto a la diferencia de los argumentos:

2𝜋 𝜋 7𝜋
1º θ − = ⇒𝜃= ⇒ 𝑧 = 2𝑒 𝑖(7𝜋/6)
3 2 6

𝜋 7𝜋
=2(cos + 𝑖𝑠𝑒𝑛 )
6 6

=-−√3 + 𝑖
2𝜋 𝜋 𝜋
2º −θ= ⇒θ= ⇒ 𝑧 = 2𝑒 𝑖(𝜋/6)
3 2 6

𝜋 𝜋
=2(cos + 𝑖𝑠𝑒𝑛 )
6 6

=√3 + 𝑖
EXPONENCIALES COMPLEJAS-ARGUMENTO DE UN NÚMERO COMPLEJO

FORMULA DE MOIVRE

PROBLEMA 23. Demostrar que:

a) 𝑒 𝑖2𝑛𝜋 =1 b) 𝑒 𝑖𝜋 +1=0

Solución:

a) Por inducción. Sea S= {n/𝑒 𝑖2𝑛𝜋 =1;n∀∈ ℤ}

1. 1∈S, ya que 𝑒 𝑖𝜋 = cos𝜋 + 𝑖𝑠𝑒𝑛2𝜋 = 1

2. Supondremos que n∈S; esto es, 𝑒 𝑖2𝑛𝜋 =1

3. Veamos que (n+1) ∈S. En efecto :

𝑒 𝑖2(𝑛+1)𝜋 =𝑒 𝑖(2𝑛𝜋+2𝜋) =𝑒 𝑖2𝑛𝜋 =1

b) Es equivalente a probar que 𝑒 𝑖𝜋 =-1. En efecto:

𝑒 𝑖𝜋 = cos𝜋 + 𝑖𝑠𝑒𝑛𝜋 = −1

Nótese que en esta fórmula se combinan 5 “importantes” números usados

en Matemáticas: 0;1; 𝜋; i; e.

PROBLEMA 24. Demostrar que 𝑒 𝑧 = 𝑒 𝑧 , siendo z un número complejo .

Solución:

Sea z = x+iy, luego:

𝑒 𝑧 = 𝑒 𝑥+𝑖𝑦 = 𝑒 𝑥 . 𝑒 𝑖𝑦 = 𝑒 𝑥 (𝑐𝑜𝑠𝑦 + 𝑖𝑠𝑒𝑛𝑦)

Tomando conjugada:

𝑒 𝑧 =𝑒 𝑥 (𝑐𝑜𝑠𝑦 + 𝑖𝑠𝑒𝑛𝑦)

= 𝑒 𝑥 (𝑐𝑜𝑠𝑦 − 𝑖𝑠𝑒𝑛𝑦)

= 𝑒 𝑥 𝑒 −𝑖𝑦
= 𝑒 𝑥−𝑖𝑦 =𝑒 𝑧
1
PROBLEMA 25. Si z+𝑧 = 2 cosθ ⇒ z2-2cos θz+1=0

Resolviendo se obtiene:

𝑧 = 𝑐𝑜𝑠θ + i𝑠𝑒𝑛θ
Z= cos θ ± i sen θ ⇒ { 1
𝑧2 = 𝑐𝑜𝑠θ − i𝑠𝑒𝑛θ

Si z= z1= 𝑐𝑜𝑠θ + i senθ = 𝑒 𝑖θ , entonces :


1
zm+𝑧 𝑚 = 𝑒 𝑖𝑚θ +𝑒 −𝑖𝑚θ

= [cos(𝑚θ) + i𝑠𝑒𝑛(𝑚θ)]+ [cos(−𝑚θ) + i𝑠𝑒𝑛(−𝑚θ)]

= 2cos(𝑚θ) ; m∈ ℤ+

Si z=z2= 𝑐𝑜𝑠θ − i senθ = 𝑒 −𝑖θ , (se procede en forma similar)

2 2
PROBLEMA 26. Hallar : a) Re (𝑒 𝑧 𝑖 ); b) Im(𝑒 −𝑧 𝑖 )

Solución:

Sea z= x+yi ⇒ z2= x2-y2+ 2xyi


2𝑖 2 −𝑦 2 +𝑖2𝑥𝑦)𝑖 2 −𝑦 2 )𝑖
a) 𝑒 𝑧 =𝑒 (𝑥 = 𝑒 −2𝑥𝑦+(𝑥
𝑥
Re (𝑒 𝑧 𝑖 ) = 𝑒 −2𝑥𝑦 cos(𝑥 2 − 𝑦 2 )

b) 𝑧 2 = 𝑥 2 − 𝑦 2 − 2𝑥𝑦𝑖 −𝑧 2 = 𝑦 2 − 𝑥 2 + (2𝑥𝑦)𝑖
2 2 −𝑥 2 +2𝑥𝑦𝑖)𝑖 2 −𝑥 2 )𝑖
e−𝑧 𝑖 = 𝑒 (𝑦 = 𝑒 −2𝑥𝑦+(𝑦
2
Im (𝑒 −𝑧 𝑖 ) = 𝑒 −2𝑥𝑦 sin(𝑦 2 − 𝑥 2 )

PROBLEMA 27.Hallar todas las soluciones de 𝑒 𝑧 = 1 + 𝑖

Solución.

Sea 𝑧 = 𝑥 + 𝑦𝑖 , luego:

𝑒 𝑧 = 𝑒 𝑥 (cos 𝑦 + 𝑖 sin 𝑦)= 1 + 𝑖

Igualando parte real e imaginaria:

𝑒 𝑥 cos 𝑦 = 1 …(1)
𝑒 𝑥 sin 𝑦 = 1 …(2)

Dividiendo (2) entre (1):


𝜋
tan 𝑦 = 1 𝑦 = 2𝑛𝜋 + 4 ; n ∈ ℤ … (3)

Sumando (1)2 𝑦 (2)2 , tenemos:


1
𝑒 2𝑥 = 2 𝑥 = 2 𝑙𝑛2 …(4)

De (3) y (4):
1 𝜋
Z= 𝑙𝑛2 + (2𝑛𝜋 + ) 𝑖 ; n ∈ Z
2 4

Nota: 𝑙𝑛2 denota logaritmo neperiano de 2.

PROBLEMA 28.
(1+𝑖)𝑛
a) Hallar(1−𝑖)𝑛−2
2𝜋 2𝜋
b) Si w𝑤 = cos + 𝑖 sin ; hallar (1 + 𝑤)𝑛
3 3
1 √3𝑖 1 √3𝑖
c) 𝑤1 = 2 + 𝑤2= 2 − , hallar 𝑤1𝑛 + 𝑤2𝑛 ; n∈ ℤ
2 2

Solución.

a) Se sabe que: 1 + 𝑖 = √2𝑒 𝑖𝜋/4 ; 1 − 𝑖 = √2𝑒 𝑖5𝜋/4 .

Remplazando en la expresión:

(1 + 𝑖)𝑛 (√2𝑒 𝑖𝜋/4 )𝑛


=
(1 − 𝑖)𝑛−2 (√2𝑒 𝑖5𝜋/4 )𝑛−2

𝑖𝜋 𝑛
√2𝑒 4
=( 5𝑖𝜋
) (√2𝑒 𝑖5𝜋/4 )2
√2𝑒 4
𝜋 5𝜋
= 𝑒 𝑖𝑛(4 − 4 ) 2𝑒 𝑖5𝜋/2

= 2𝑒 −𝑛𝜋𝑖 𝑒 5𝜋𝑖/2
5𝜋
= 2𝑒 ( 2 −𝑛𝜋)𝑖
5𝜋 5𝜋
= 2 [cos( − 𝑛𝜋) + 𝑖 sin( − 𝑛𝜋)]
2 2
2𝜋 2𝜋 1 √3
b) Se sabe que 𝑤 = cos + 𝑖 sin = −2 + 𝑖 ; entonces:
3 3 2
1 √3 1 √3
1+𝑤 = 1− + 𝑖= + 𝑖
2 2 2 2
Llevando a su forma polar:
1 √3
1+𝑤 =2+ 𝑖 = 𝑒 𝑖𝜋/3
2
Luego:
𝜋𝑖 𝑛𝜋 𝑛𝜋
(1 + 𝑤)𝑛 = (𝑒 3 )𝑛 = 𝑒 𝜋𝑛𝑖/3 = cos + 𝑖 sin
3 3
c) Pasando 𝑤1 y 𝑤2 a su forma polar:
𝑤1 = 𝑒 2𝜋𝑖/3 y 𝑤2 = 𝑤1 = 𝑒 −2𝜋𝑖/3
𝑤1𝑛 + 𝑤2𝑛 = (𝑒 2𝜋𝑖/3 )𝑛 + (𝑒 −2𝜋𝑖/3 )𝑛

= 𝑒 2𝜋𝑛𝑖/3 + 𝑒 −2𝜋𝑛𝑖/3
2𝑛𝜋 2𝑛𝜋 2𝑛𝜋 2𝑛𝜋
= (cos + 𝑖 sin ) + ( cos − 𝑖 sin )
3 3 3 3
2𝑛𝜋
= 2 cos ; ∀ 𝑛 𝜖ℤ
3

PROBLEMA 29. Hallar el valor de la expresión

−3
1 √3
(− + 𝑖) 𝑖𝑖
2 2

Solución.
Pasamos a la forma polar:
1 √3
−2 + 𝑖 = 1. 𝑒 2𝜋𝑖/3 , 𝑖 = 0 + 1𝑖 = 𝑒 𝑖𝜋/2
2

Remplazando la expresión:
−3
1 √3 2𝜋𝑖 𝜋𝑖
(− + 𝑖) (𝑖 𝑖 ) = (𝑒 3 )−3 (𝑒 2 )𝑖
2 2

= 𝑒 −2𝜋𝑖 𝑒 −𝜋/2

= 𝑒 −𝜋/2 (cos 2𝜋 − 𝑖 sin 2𝜋)


𝜋
= 𝑒 − 2 (1 − 0)
= 𝑒 −𝜋/2

PROBLEMA 30.Demostrar que:

a) cos 5𝜃 = 16 cos 5 𝜃 − 20 cos3 𝜃 + 5 cos 𝜃


b) sin 5𝜃 = 16 cos 4 𝜃 sin 𝜃 − 12 cos 2 𝜃 sin 𝜃 + sin 𝜃

Solución:

Teniendo en cuenta que 𝑖 2 = −1 , 𝑖 3 = −𝑖 , 𝑖 4 = 1 , 𝑖 5 = 𝑖 , y aplicando la formula


de Moivre, tenemos: 𝑒 𝑖5𝜃 = (𝑒 𝑖5𝜃 )5 , entonces:

cos 5𝜃 + 𝑖 sin 5𝜃 = (cos 𝜃 + 𝑖 sin 𝜃)5


= cos 5 𝜃 + 5cos4 𝜃(𝑖 sin 𝜃) + 10 cos 3 𝜃 ( 𝑖 sin 𝜃)2 + 10 cos 2 𝜃(𝑖 sin 𝜃)3
+ 5 cos 𝜃(𝑖 sin 𝜃)4 + (𝑖 sin 𝜃)5

= (cos5 𝜃 − 10 cos3 𝜃 sin2 𝜃 + 5 cos 𝜃 sin4 𝜃) + (5 cos4 𝜃 sin 𝜃


− 10 cos2 𝜃 sin3 𝜃 + sin5 𝜃)

a) Igualando a la parte real entre si:


cos 5𝜃 = cos5 𝜃 − 10 cos 3 𝜃 sin2 𝜃 + 5 cos 𝜃 sin4 𝜃
= cos5 𝜃 − 10 cos3 𝜃(1 − cos 2 𝜃) + 5 cos 𝜃(1 − cos2 𝜃)2
= 11 cos5 𝜃 − 10 cos3 𝜃 + 5 cos 𝜃(1 − 2 cos 2 𝜃 + cos4 𝜃)
= 16 cos 5 𝜃 − 20 cos3 𝜃 + 5 cos 𝜃
b) Igualando la parte imaginaria entre si:
sin 5𝜃 = 5 cos 4 𝜃 sin 𝜃 − 10 cos2 𝜃 sin3 𝜃 + sin5 𝜃
= 5 cos 4 𝜃 sin 𝜃 − 10 cos2 𝜃 (1 − cos2 𝜃) sin 𝜃 + (1 − cos 2 𝜃)2 sin 𝜃

= 5 cos 2 𝜃 sin 𝜃 − 10 cos2 𝜃 sin 𝜃


+ 10 cos4 𝜃 sin 𝜃 + sin 𝜃 − 2 cos 2 𝜃 sin 𝜃 + cos 4 𝜃 sin 𝜃

= 16 cos4 𝜃 sin 𝜃 − 12 cos 2 𝜃 sin 𝜃 + sin 𝜃

PROBLEMA 31.Hallar la formula reducida para:


a) 1 + cos 𝑥 + cos(2𝑥) +…+ cos(𝑛 − 1)𝑥
b) sin 𝑥 + sin(2𝑥) +…+ sin(𝑛 − 1)𝑥
c) cos 𝑥 + cos(3𝑥) +…+ cos(2𝑛 − 1)𝑥
d) sin 𝑥 + sin(3𝑥) +…+ sin(2𝑛 − 1)𝑥

Solución.

Sabemos que 𝑒 𝑖𝑘𝑥 = cos 𝑘𝑥 + 𝑖 sin 𝑘𝑥 . De acuerdo a lo que necesitamos hallar,


tenemos:
𝑛−1
𝑒 𝑖0𝑥 + 𝑒 𝑖𝑥 + 𝑒 𝑖2𝑥 + ⋯ + 𝑒 𝑖(𝑛−1)𝑥 = ∑𝑘=0 𝑒 𝑖𝑘𝑥 …(1)

Pero por propiedad de sumatorias:


𝑛−1 𝑛−1 1−𝑒 𝑖𝑛𝑥
∑𝑘=0 𝑒 𝑖𝑘𝑥 = ∑𝑘=0 𝑒 𝑖(𝑛−1)𝑥 = …(2)
1−𝑒 𝑖𝑥

Aplicando (2) en (1):


1−𝑒 𝑖𝑛𝑥
𝑒 𝑖0𝑥 + 𝑒 𝑖𝑥 + 𝑒 𝑖2𝑥 + ⋯ + 𝑒 𝑖(𝑛−1)𝑥 = 1−𝑒 𝑖𝑥

Desarrollando tenemos:

(cos 0 + 𝑖 sin 0) + (cos 𝑥 + 𝑖 sin 𝑥) +…+[cos(𝑛 − 1)𝑥 + 𝑖 sin(𝑛 − 1)𝑥] =


1−(cos 𝑛𝑥+𝑖 sin 𝑛𝑥)
1−(cos 𝑥+𝑖 sin 𝑥)

Racionalizando y ordenando toda la expresión:


[1 + cos 𝑥 + cos 2𝑥 +… + cos(𝑛 − 1)𝑥] + 𝑖[sin 𝑥 + sin 2𝑥 + … + sin(𝑛 − 1)𝑥] =
[1 − cos 𝑥 − cos 𝑛𝑥 + cos(𝑛 − 1)𝑥] + 𝑖[sin(𝑛 − 1)𝑥 − sin 𝑛𝑥 + sin 𝑥]
2(1 − cos 𝑥)

Igualando las partes reales e imaginarias entre si:


1−cos 𝑥−cos 𝑛𝑥+cos(𝑛−1)𝑥
1 + cos 𝑥 + cos 2𝑥 +…+ cos(𝑛 − 1)𝑥 =
2(1−cos 𝑥)

b) senx + senlx + ... + sen{n-1)x = sen{n-l)x- sen nx + senx


2(1 - cosx)

En forma análoga calculamos (c) y (d), teniendo en cuenta que:

PROBLEMA 32.
Dado z = (5 - i)4 (1 + i ) . Probar que: π = 4arctg(1/5) - arctg(1/239)

Solución:
Pasando z a su forma polar, utilizando dos formas:
Primera forma. Pasamos (5 - i)4 y (1 + i) a su forma polar:

l + i = √2e iπ/4 y 5 - i = √26e ia


Donde oc = arc tg(-1/5) = - arc tg(1/5) => (5 - i)4 = 262 eí4a
Luego tenemos:
z = (5 - i)4 (1 + i) = 262 √2 eí4 eiπ/4= 262 √2eí (4a+π/4)
Donde:
a = arc tg(-1/5)

Segunda forma. Efectuamos (5 - í)4(l + i) y después lo pasamos a su forma


polar:

z = (5 - i)4 (1 + i) = (5 - i )2 (5 - i)2 (1 + i) = (24 -10 i)2 (1 + i)


= 4(119-120i)(1 + i) = 4(239 - i) =4√57122 eip
= 4xl69√2eíp
Donde:

(3 = arc tg(-1/239)

Igualando (1) y (2):

π = -are tg(1/239) + 4arc tg(1/5)

PROBLEMA 33. Demostrar que:

π = 3arc tg(1/4) + arc tg(1/20) + arc tg(1/1985)

Solución:

Sabemos que si z = a + ib, entonces: arg(z) = arc tg(b/a)


También, z en su forma polar es: z = |z|e'e, donde 9 = arg(z), entonces:
zn = \z\nelnS ; donde n9 = a>g(z")
De la ecuación principal, se tiene:
-| = 3arc tg(1/4) + arc tg(1/20) + arc tg(1/1985)

=> -arc tg(1/1985) = 3arc tg(1/4) + are tg(1/20)

Operando en la parte (II):


3arc tg(1/4) + arc tg(1/20) - ^ = 3 arc tg(1/4) + arc tg(1/20) + arc tg(-1)
= 3arg (l + 4i) + arg(1 + 20i) + arg(1 – i)
= arg[ (1 + 4i)3 + arg(1 + 20i) + arg(1 - i)
= arg [(l + 4i)3(l + 20i)(l-i)]
= arg (1985-i)
= arc tg(-l/1985)
= -arc tg(l/1985)
Como de la expresión (II) se llega a la expresión (I), se concluye que la
ecuación inicial se cumple, con lo cual quedaría demostrada la veracidad de
la expresión principal.

PROBLEMA 34.
Considérese la parte real e imaginaria de (a + io)e'e, donde δ= arg(a + ib).
Pruébese:

a) a cos θ - bsen θ = √a2 +b 2 cos(θ + δ)


a senθ + bcos θ = √a2 +b 2 sen(θ + δ)

b) cosθ – sen θ = √2cos(θ+ π/4)

cos θ + senθ= √2sen(θ +Π/4)

Solución:

a) Efectuando:

(a + ib)(cosθ + i sen θ) = (a cosθ - bsen θ) + i(asen θ + b cosθ)


Por otro lado, pasamos a + ib a su forma polar:

(a + ib) =√a2 +b 2 e iδ , donde δ =arg(a+ib)

Entonces:

(a + ib)ei δ =(√a2 +b 2 e iδ ) e iθ = √a2 +b 2 e i(δ+θ)

Igualando (1) y (2):


(a cos θ - b sen θ) + i(a sen θ + b cos θ) = √a2 + b2 [cos (θ + δ) + isen (θ + δ)]
Igualando parte real e imaginaria:

a cos θ - b sen θ = √a2 + b2 cos (θ + δ) Y

a senθ + bcosθ = √a2 + b2 sen(θ + δ)


b) Al comparar las expresiones:
cos θ - sen θ = √2 cos(θ + π/4)
cosθ + sen θ = √2 sen( θ + π/4)

con las de (a), se deduce que éstas son un caso particular, donde a = b = 1 ,

 a + ib = 1 + i = √2eiδ , donde δ= π/4

Reemplazando estos valores en las fórmulas de la parte (a), se obtiene la


demostración pedida.

PROBLEMA 35. Si z es un punto sobre la circunferencia |z -1| = 1, probar


que:

arg(z -1) = 2arg(z) = (2/3)arg(z 2 -z)


Usando consideraciones geométricas.
Solución:
Vemos que |z -1| = 1 es la circunferencia de centro (1; 0) y radio 1.

Sea z = a + ib, entonces: y

α = arg(z) = arc tg(b/a) z

∞ θ
a x

y por lo tanto z -1 = ( a -1) + ib, entonces:

θ= arg(z-1)=arctg(b)

tgθ = b
a-1
Según la figura, θ = 2α, entonces:
arg(z - l) = 2arg(z)
Además:
arg(z2 - z ) =arg[z(z -1)]
= arg(z) + arg(z-l)
=α+θ
= α + 2α
= 3α
= 3arg(z)
Entonces:
arg(z2 - z ) = 3arg(z) => 2arg(z) - 2arg(z2 – z)
De (1) y (2):

arg(z -1) = 2arg(z) = 2arg(z 2 - z)


RAICES DE UN NÚMERO COMPLEJO
PROBLEMA 36. Hallar los valores de z en: z2 + 2iz -4 = 0
Solución:
Multiplicando a la expresión, z2 + 2iz - 4 = 0 por (-1):
-z2-2iz + 4 = 0
((iz)2-2iz + l) + 3 = 0
(iz-l)2=-3
i z - l = ±√3 i
Por ( i ) : - z - i = ±√3
 z = ±√3- i

 z1 = √3- i
z2 = - √3- i

PROBLEMA 37. Hallar todos los valores de las raíces siguientes:


a) 3√i b) 4√-i c) √3 + 4 i d) √21i-20 e) 4√16

Solución:
En todos los ejercicios, k X Z+
Sea z = i, pasando a su forma polar: z = 0 +1 i = e iπ/2
Luego, las raíces cúbicas de i vienen dadas por:
π π
2kπ+ 2kπ+
2 2
Wk = cos ( ) + isen ( ) ; k = 0; 1; 2
3 3

Para:

π π √3 1
k=0 =>. W 0 = = cos 6 + isen 6 = + 2i
2

5π 5π √3 1
k=1 =>. W 1 = = cos + isen =− + 2i
6 6 2

3π 3π
k=2 =>. W 2 = = cos + isen = 0 − i = −i
2 3
b) Sea z = -1 = -1 + 0i = eiπ . Las raíces cuartas viene dadas por:

2kπ+π 2kπ+π
Wk = cos ( ) + isen ( ) ; k = 0; 1; 2; 3
4 4
π π √2 √2
k=0 =>. W 0 = = cos 4 + isen 4 = + i
2 2

3π 3π √2 √2
k=1 =>. W 1 = = cos + isen = + i
4 4 2 2

5π 5π √2 √2
k=2 =>. W 2 = = cos + isen =− − i
4 4 2 2

7π 7π √2 √2
k=3 =>. W 3 = = cos + isen = − i
4 4 2 2

c) Sea z = 3 + 4 i = 5eiθ , θ = arc tg(4/3). Las raíces cuadradas vienen


dadas por:

2kπ+θ 2kπ+θ
Wk = √5 [cos ( ) + isen ( )] ; k = 0; 1
4 4

Para:

k= 0 => w0 =√5e i(θ/2) = √ 5 [cos(θ/2) + isen(θ/2)]


= √ 5 [(2/√5) + i(1/√5)
=2+i

2π+θ 2π+θ
Wk = √5 [cos ( ) + isen ( )]
2 2
2
= √5 [cos(π+θ/2) + isen(π+θ/2)]
2
= √5 [-cos(θ/2) + isen(θ/2)]
= -(2 + i)

d) z = 21 i - 20 = 29eiθ; θ = arc tg(-21/20). Las raíces cuadradas vienen dadas


por:
2 2kπ+θ 2kπ+θ
W k = √29 [cos ( ) + isen ( )]
2 2

Donde k =0; 1 y donde θ pertenece al segundo cuadrante.


Para k = 0 :

W0 = √ 2 9 [cos(θ/2) + isen(θ/2)]

2
√29(3 + 7i)
= 2
√58

(3 + 7i)
= 2
√2

Pues: cosθ = -20/29 ; senθ = 21/29


Para k = 1:

2 2π+θ 2π+θ
W 1= √29 [cos ( ) + isen ( )]
2 2

= √ 2 9 [-cos(θ/2) + isen(θ/2)]

(3 + 7i)
=− 2
√2

e) Sea z - 1 6 = 16e'° . Las raíces cuartas son:

wk = [2 cos(kn/2) + i sen(kn/2)] ; k = Q ; \ ; 2

Para:
k = 0 => w0 = 2
w 1 =2e í , t / 2 =2i
vi/q = 2e
PROBLEMA 38.

Hallar las raíces cuadradas de z =-15-18i Y


Solución

Z=-15-8i=17eiƟ;

Donde Ɵ = arc tg(-8/-15) = arc tg(8/15)



Del grafico se observa que Ɵ pertenece al tercer
-8
cuadrante, entonces cos Ɵ=-15/17, ya que X

17
8
Ɵ
15

15 8
Además, como
3π π Ɵ 3π
Π<Ɵ<=  <2<
2 2 4

Entonces Ɵ/2 pertenece al segundo cuadrante. Luego:


Ɵ −√1+cosƟ −1 Ɵ √1+cosƟ 4
cos( 2)=𝑥 = = y sen( 2)=𝑥 = =
2 17 2 √17

Las raíces de z vienen dadas por:


2𝑘𝜋+Ɵ 2𝑘𝜋+Ɵ
wk = √17 [cos 𝜃 ( 2
) + 𝑖𝑠𝑒𝑛 ( 2
)] ; k=0;1

Para:

k = 0  w0 = = √17 [cos(Ɵ/2) + 𝑖𝑠𝑒𝑛(Ɵ/2)] = -1+4i

k = 1  w1 = = √17 [cos(π + Ɵ/2) + 𝑖𝑠𝑒𝑛(π + Ɵ/2)] = 1-4i

PROBLEMA 39.

Hallar las raíces cuadradas de: z = −8 √2+8 √2𝑖.

Solución
Pasamos z a su forma polar:
z = −8 √2+8 √2𝑖 =16ei(3π/4)

luego ,las raíces cuadradas de z vienen dadas por;

wk= 2 [cos (
2𝑘𝜋+3𝜋/4
) + 𝑖𝑠𝑒𝑛 (
2𝑘𝜋+3𝜋/4
)] ;
Y
4 4

donde k =0 ; 1 ; 2 ; 3
z 8√2
3𝜋 3𝜋
k = 0  w0 = 2 [cos ( 16 ) + 𝑖𝑠𝑒𝑛 ( 16 )]
3
𝜋 3𝜋 𝜋 3𝜋
= 4
k = 1  w1 = 2 [cos (2 + ) + 𝑖𝑠𝑒𝑛 ( 2 + )]
16 16

3𝜋 3𝜋 −8√2 X
= 2 [−𝑠𝑒𝑛 ( 16 ) + 𝑖𝑐𝑜𝑠 ( 16 )]

3𝜋 3𝜋
k = 2  w2 = 2 [cos (𝜋+ ) + 𝑖𝑠𝑒𝑛 (𝜋 + )]
16 16

3𝜋 3𝜋
= 2 [−𝑐𝑜𝑠 ( 16 ) − 𝑖𝑠𝑒𝑛 ( 16 )]

3𝜋 3𝜋 3𝜋 3𝜋
k = 3  w3 = 2 [cos (2 + ) + 𝑖𝑠𝑒𝑛 ( 2 + )]
16 16

3𝜋 3𝜋
= 2 [𝑠𝑒𝑛 ( 16 ) − 𝑖𝑐𝑜𝑠 ( 16 )]

Para calcular las funciones de 3π/16 se utiliza;

√2− √2
3𝜋 1+cos 3𝜋/4
cos 8 = √ =
2 2

√2+ √2− √2
3𝜋 1+cos 3𝜋/8
cos 16 = √ =
2 2

√2− √2− √2
3𝜋
cos 16 = = 2

PROBLEMA 40.

a) Si w0 , w1 y w2 son números complejos . probar por sustitución directa, que si


w2  0, entonces;
−𝑤1 ±√w 21−4𝑤2 𝑤0
𝑧 = 2𝑤2

Es la solución de la ecuación: w2 𝑧 2 +w1 z+w0 = 0

b) Resolver: 0,1𝑧 2 -0,3 z-i = 0


Solución:

a) Reemplazando en la ecuación w2 𝑧 2 +w1 z+w0 = 0, se tiene:

2
−𝑤1 ±√w 21−4𝑤2 𝑤0 −𝑤1 ±√w 21−4𝑤2 𝑤0
w2[ ] +w1[𝑧 = ] + w0 = 0
2𝑤2 2𝑤2

de donde se deduce que :

w2 𝑧 2 +w1 z+w0 = 0

b) La ecuación se puede escribir como: 𝑧 2 -3z-10i = 0

Aplicándola parte(a):

3±√9−40𝑖
𝑥=
2

Calculamos primero las raíces de:


5 + 4𝑖
√9 + 40𝑖 = { o
−5 − 4𝑖
Reemplazando obtenemos: z = 4+2i o z = -1-2i

PROBLEMA 41. Sea z = -i , p = 1/3, y q = 3

a) Hallar (𝑧 𝑝 )𝑞 b) hallar (𝑧 𝑞 )𝑝

Solución:
Pasando z a la forma polar: z = -i = 0-1i = ei3π/2

a) Las raíces cubicas de z, (p = 1/3), son:


2𝑘𝜋+3𝜋/2 2𝑘𝜋+3𝜋/2
Wk = e (2kπ+3π/2)/3 = cos ( ) + 𝑖𝑠𝑒𝑛 ( ) ; k=0 ;1 ;2.
3 3

De donde obtenemos para:


𝜋 𝜋
K = 0  w0 =cos + isen 2 = i
2

7𝜋 7𝜋
K = 1  w1 =cos + isen = e7πi/6
6 6

11𝜋 11𝜋
K = 2  w2 =cos + isen = e11πi/6
6 6
Ahora elevando al cubo cada raíz encontrada(q = 3), tenemos:

w03 = (−𝑖)3 – i ………..(1)


3
w13 = ( 𝑒 7𝜋𝑖/2 ) = 𝑒 7𝜋𝑖/2 = – i ………..(2)
3
w23 = ( 𝑒 11𝜋𝑖/2 ) = 𝑒 11𝜋𝑖/2 = – i ………..(3)

de (1), (2) y(3) concluimos que :


(𝑧 𝑝 )𝑞 =-i

b) Elevando z al cubo (q = 3), tenemos:


3
𝑧 3 = ( 𝑒 3𝜋𝑖/2 ) = 𝑒 𝑖9𝜋/2= 𝑒 𝑖𝜋/2

Luego, tomando la raíz cubica (p = 1/3) de 𝑧 3 , tenemos:


1/3 2𝑘𝜋+𝜋/2 2𝑘𝜋+𝜋/2
Wk = 3√(𝑧 3 ) = ( 𝑒 𝜋𝑖/2 ) = 𝑐𝑜𝑠 ( )+ 𝑖𝑠𝑒𝑛 ( )
3 3

Donde k = 0 ; 1 ; 2

Para:
𝜋 𝜋 √3 1
K = 0  w0 = 𝑐𝑜𝑠 ( 6 )+ 𝑖𝑠𝑒𝑛 (6 )= + 2i ………………….(4)
2

5𝜋 5𝜋 √3 1
K = 1  w1 = 𝑐𝑜𝑠 ( 6 )+ 𝑖𝑠𝑒𝑛 ( 6 )= + 2i ………………….(5)
2

3𝜋 3𝜋
K = 2  w2 = 𝑐𝑜𝑠 ( 6 )+ 𝑖𝑠𝑒𝑛 ( 6 )= -i ………………….(6)

Luego, (𝑧 𝑞 )𝑝 tiene soluciones distintas dados por (4) , (5) y (6).

PROBLEMA 42. Hallar los valores de z  C tal que:

(|𝑥|2 − 2𝑧 + 1)2 = 16(3 – 4i)

Solución:
Sea z = a + bi.

Luego remplazando en la ecuación del anunciado:


[(𝑎2 + 𝑏 2 ) − 2(𝑎 + 𝑏𝑖) + 1]2 = 16(3 – 4i) ………..(*)

 [(𝑎 − 1)2 − 𝑏 2 − 2𝑏𝑖] = √48 − 64𝑖 ………(1)


Luego de √48 − 64𝑖 calculamos sus raíces, mediante:
4
48 − 64𝑖 =80𝑒 𝑖 ; = arc tg(− 3)

2𝜋𝑘+ 2𝜋𝑘+
Wk =4 √5 [cos ( ) + 𝑖𝑠𝑒𝑛 ( )] ; k = 0, 1
2 2

Para k = 0

W0 = 4 √5[cos(Ɵ/2) + 𝑖𝑠𝑒𝑛(Ɵ/2)] = 8- 4i

= 4 √5[2/ √5 − 1/ √5𝑖]= 8 -4i

Para k = 1
2𝜋+ 2𝜋+
W1 =4 √5 [cos ( ) + 𝑖𝑠𝑒𝑛 ( )]
2 2

 
= 4 √5 [cos (𝜋 + 2) + 𝑖𝑠𝑒𝑛 (𝜋 + 2)]

 
= 4 √5 [− cos (2) − 𝑖𝑠𝑒𝑛 (2)]

 
= 4 √5 [− ( ) − (− ) 𝑖]
√5 √5

= -8 + 4i …………..(3)

Igualando (1) y (2):

(𝑎 − 1)2 + 𝑏 2 – 2bi = 8 – 4i

𝑎1 = 3
 b1 = 2  {
𝑎2 = −1
Igualando (1) y (3)

(𝑎 − 1)2 + 𝑏 2 – 2bi = 8 – 4i

𝑎3 = 2 √3𝑖 + 1
 b2 = −2  {
𝑎4 = −2 √3𝑖 + 1
Final mente reemplazando en z = a + bi obtenemos los posibles valores de z:
Z1 = 3 + z1

Z2 = -1 + 2i

Z3 = 1+ (2 √3𝑖 − 2)𝑖 ……….., no cumplen (*)


Z1 = 3 - (2 √3𝑖 + 2)𝑖 ……….., no cumplen (*)

PROBLEMA 43.

Sea w una raíz n-esima de la unidad, tal que w  1.

a) Probar que n+ , n  1 se cumple 1 + w + 𝑤 2 +….+ 𝑤 𝑛−1 = 0


b) Aplicar(a) y resolver: 1 + z+ 𝑧 2 +𝑧 3 = 0

Solución:

a) Sea s = 1 + w + 𝑤 2 +….+ 𝑤 𝑛−1 ………….(1)

Se tiene: ws = v + c+ 𝑤 3 +…..+ 𝑤 𝑛 …………..(2)

Restando (1) y (2), miembro a miembro


1−𝑤 𝑛
S(1-w) = 1- 𝑤 𝑛  , w  1.
1−𝑤

Como w es raíz n-esima de la unidad, entonces 𝑤 𝑛 = 1. Luego:

1- 𝑤 𝑛  s=0

De (1) tenemos:

1 + w + 𝑤 2 +….+ 𝑤 𝑛−1 = 0
1−𝑧 4
b) 1 + z + 𝑧 2 + 𝑧 3 = = 0 y como z  1, se tiene 𝑧 4 = 1
1−𝑧

4
Resolviendo: 𝑧 4 = 1  √𝑧 4 = √1
4

Calculando la raíz cuartas de 1 encontramos:


Z1 = i , Z2 = -1 , Z3 = -i

El valor de Z0 = 1 se descarta, pues por condición z  1

PROBLEMA 44. Resolver z = zn-1 ,n  1

Solución:

Expresando su forma polar: z = re-i y z =re i, luego:


𝑛−1
re-i = ( 𝑟𝑒 𝑖 ) = 𝑟 𝑛−1 𝑒 (𝑛−1)𝑖

la última relación se cumple si solo si:


1˚. 𝑟 = 𝑟 𝑛−1  𝑟 𝑛−2 = 1 ; (r  1)  r = 1 |𝑧| = 1 ; y

2˚. 𝑒 −𝑖 = 𝑒 (𝑛−1)𝑖  𝑒 (𝑛−1)𝑖+𝑖 =1  (n – 1)i + i =2kπi

 π = 2kπ
2𝑘
 k = ; k = 0; 1;………..; n-1
𝑛

Luego,
2𝑘 2𝑘
Z = cos ( ) + 𝑖𝑠𝑒𝑛 ( ) ; k = 0; 1;………..; n-1
𝑛 𝑛

¿Qué analogía se puede establecer entre este resultado y las n raíces de la


unidad?

NOTA: si r = 0  |𝑧| = 0  z = 0. Luego, z = 0 también satisface la ecuación dada

Obsérvese lo que pasa si trabajamos con z  z n1  zz  z n  z n  z 2

PROBLEMA 45
a) Resolver la ecuación: (z+1)m –(z-1)m = 0, sobre los complejos, con

m
b) Resolver para n entero positivo:

 3 z 
n

  (3  z )
n

 1 i 

SOLUCIÓN

a) Operando la expresión
b)
( z  1) m  ( z  1) m  0
( z  1) m  ( z  1) m
 z 1 
m

  1 ; z  1 y z  1
 z 1 
z 1
sea w   wm  1
z 1
Luego resolviendo la ecuación:
 2 k   2 k  i (2 k / m )
wk  cos    i sem   e ; k  0;1; 2;.......;(m  1)
 m   m 
 z 1  i (2 k / m )
wk    e
 z  1 k
1  e(i (2 k / m )
 zk  ; k  0;1;...........;(m  1)
1  ei (2 k / m )
son soluciones para k  0;1; 2;..........;(m  1), considerando que wk  1
n
 3 z 
B.    wn  1
 (1  i)(3  z ) 
3 z
Sea w  W n 1
(1  i)(3  z )
Liego:

 2 k   2 k  i (2 k / n )
wk  cos    i sen  e ; k  0;1; 2;...;(n  1)
 n   n 
Pero,
 3 z 
wk     ei (2 k / n )
 (1  i )(3  z )  k

Despejando: zk se tiene:

3(1  i)ei (2 k / n )  3
zk  ; k  0;1;...;(n  1)
1  (1  i)ei (2 k / n )

O en forma equivalente:
3(1  i ) wk  3
zk  ; k  0;1;...;(n  1)
1  (1  i) wk

PROBLEMA 46. Hallar rodas las raíces de: (1  z)5  (1  z)5


Solución:
(1  z )5  (1  z )5
 1 z 
5

   1  e ; z  1, z  1
10

 1 z 
 1 z   2 k   2k 
   cos    sen   ; k  0;1; 2;3; 4
 1  z k  5   5 
Despejando zk
 1 z  i (2 k /5) ei (2 k /5)  1
   e  z  ; k  0;1; 2;3; 4
 1  z k ei (2 k /5)  1
k

PROBLEMA 47 Hallar z tal que:


a) Sea conjugado con su cuadrado
b) Sea conjugado con su cubo
Solución:
Sea z  rei  (rei )  z  rei , donde r  z  z
2
a) De la condición del problema: z  z 2  z
Re emplazando : rei  (rei ) 2  r 2 e  i (2 )
Esta igualdad se cumple si sólo si :
1. r  r 2  r  1 ó r  0; y
2 k
2. ei  e  i (2 )  3  2k   k  ; k  0;1; 2.
3
2 k
Trabajando con r  1 y  k  ; k  0;1; 2. obtenemos las respuestas
3
para:
k  0  z  ei0  ei 0  1
 2   2 
k  1  z  ei1  ei (2 /3)  cos    i sen  
 3   3 
1 3
  i
2 2
 4   4 
k  2  z  ei2  ei (4 /3)  cos    i sen  
 3   3 
1 3
  i
2 2
Si r  0  z  0  z  0
Luego, las saluciones son :
1 3 1 3
z  0; z  1; z    i ; z  i
2 2 2 2
b) En forma similar se obtiene:
z  0; z  1; z  1; z  i; z  i

NOTA: Operar el problema, trabajando por ejemplo de la manera


siguiente:
2 3 3
Para (a) : z  z  z.z  z  z y vea lo que ocurre.
2

PROBLEMA 48. Si p es una raíz de z1/n , hallar las otras raíces de z.

Solución:

Sea z  rei , Luego las raíces n-ésimas de z están dadas por:


  2k  0   2k  0  
z1/ n  wk  r1/ n cos    i sen    ; k  0;1;...; n  1 ...(1)
  n   n 
Elevando la “n”:
z   wk   r  cos(2k   )  i sen(2k   )
n

 rei (2 k  )
Luego,
i ( 2 k  )
z  rei  re  rei (2 k ) .ei
Sacando la n-ésima en (2) para buscar una relación con (1) tenemos:
z1/ n  r1/ n (ei )1/ n (ei 2k )1/ n  (rei )1/ n (ei 2k )1/ n ...3
Pero: p  z1/n (raíz fija, no depende de k)
 p   rei 
1/n

Reemplazando en (3):
  2k   2k 
z1/ n  p  eik   p cos    i sen 
1/ n

  n   n 
PROBLEMA 49. Demostrar que si z1  z2  z3  z4  0 y z1  z2  z3  1 ,
entonces z1  z2 y z3 son vértices de un triangulo equilátero; esto es,
hay que probar:
z1  z2  z1  z3 z3  z2  z1  z2  z3  z2
2 2

En efecto:
2
 
z1  z2   z2  z1  z1  z1   z2  z1  ( z1  z3 ) ....(1)

z1  z3   z  z   z  z    z  z  (z  z )
2
1 3 1 3 1 3 1 3 ...(2)

Igualando (1) y (2):


z2  z1 z2  z2 z1  z1  z3 z1  z1 z3  z1  z3
2 2 2 2

Simplificando se tiene:
z1 z2  z2 z1  z3 z1  z1 z3

 z1 z3  z1 z2  z1 z2 z1 z3
 
 z1 z3  z2  z1  z2  z3 

De enunciado, z1   z2  z3  z1   z2  z3 Remplazando:
  z2  z3   z3  z2   z2 z3  z3  z2  z3 z2  z3 z2  z2 z3
2 2

z 2  z3  z 2  z3   z2  z3 z2  z2 z3  z3  z3 z2  z 2 z 3
2 2

De donde resulta que (3) y (4) son iguales, y de esto concluimos que:
z1  z2  z1  z3

Analógicamente se obtiene que:

z1  z3  z2  z3

PROBLEMA 50. Verificar que las 3 raíces cúbicas de la unidad son los
vértices de un triángulo equilátero inscrito en la circunferencia unidad.
Solución:

1 3 1 3
Las raíces cúbicas de la unidad son: w1  1; w2    i y w3  i
2 2 2 2

Sumamos las tres raíces:


 1 3   1 3 
w1  w2  w3  1     i      i   0
 2 2   2 2 

De la expresión se comprueba que:


1 + w + w2 =0
Como w1  w2 y w3 son raíces cúbicas de la unidad, entonces:

w1  w2  w3  1

1 3 1 3
Luego, por el problema 49, w1  1; w2    i y w3    i son
2 2 2 2
vértices de un triángulo equilátero inscrito en la circunferencia unidad.
PROBLEMA 51. Probar que las raíces n-ésimas de la unidad son los
vértices de un polígono regular de n lados, inscrito en la circunferencia
unidad.
Solución:
Las raíces n-ésimas de la unidad vienen dadas por:
wk  e2 k i / n
 2 k   2 k 
 cos    i sen  ; k  0;1;...; n  1
 n   n 

Además,
wk  ei 2 k / n  1

Esto indica que los wk están sobre la circunferencia de radio 1 y centro


en el origen. Probemos que son los vértices de un polígono regular de
n lados.
En efecto:
wk 1  wk  ei 2( k 1) / n  ei 2 k / n
 e i 2 k  / n e i 2 / n  1
 ei 2 / n  1

La relación (*) indica que el módulo de la diferencia de dos raíces


consecutivas no depende de k, de modo que todos los lados del
polígono formado, tomando como vértices las raíces n-ésimas de la
unidad, son iguales.
PROBLEMA 52. Si los números complejos:
 13   25   25   37 
z  ( z  i ) cos    (1  i ) cos   y w  (1  i ) cos    (1  i) cos  
 24   24   24   24 

Son vértices consecutivos de un polígono regular de n lados, centrados en (0; 0);


encontrar el valor de zn + w n .

Solución:

De las relaciones trigonométricas tenemos:

 25    25   13   13 


cos    sen     sen     sen  
 24   2 24   24   24 
 37    37   25 
cos    sen      sen  
 24   2 24   24 

Expresando z y w de otra forma:


 13   25 
z  (1  i ) cos    (1  i )  sen  
 24   24 
 13   13 
 (1  i ) cos    i (1  i )  sen  
 24   24 
  13   13  
 (1  i ) cos    i sen  
  24   24  
2e i /4 .e13 i /24
 z  2e19 i /24

De la forma expresamos w:
25𝜋 25𝜋
𝑤 = (1 − 𝑖)𝑐𝑜𝑠 ( ) − (1 − 𝑖)𝑠𝑒𝑛 ( )
24 24
25𝜋 25𝜋
= (1 − 𝑖) cos ( ) + 𝑖(1 − 𝑖)𝑠𝑒𝑛 ( )
24 24
25𝜋 25𝜋
= (1 − 𝑖) [cos ( ) + 𝑖𝑠𝑒𝑛 ( )]
24 24

𝑤 = √2𝑒 𝜋𝑖/4 . 𝑒 25𝜋𝑖/24 = √2𝑒 31𝜋𝑖/24

Se observa de (1) y (2) que :|𝑧| = |𝑤| = √2 , (esto es “viendo“ la forma polar).

Luego, z y w son vértices consecutivos de un polígono regular de “n” lados,


entrados en (0;0)con modulo (de los vértices) igual a √2 .Entonces podemos decir
que tales vértices son raíces de un numero complejo v; es decir,𝑣 = |𝑣|𝑒 𝑖𝜃

Las raíces n-esimas de 𝑣 están dadas por la relación :

1⁄ 2𝑘𝜋 + 𝜃 2𝑘𝜋 + 𝜃
wk= |𝑣| 𝑛 [cos ( ) + 𝑖𝑠𝑒𝑛( )] ; k=0;1;……; (𝑛 − 1) … . (3)
𝑛 𝑛
1⁄
Pero, |𝑣| 𝑛 = √2, luego reemplazamos en (3)

w 2𝑘𝜋+𝜃 2𝑘𝜋+𝜃
𝑘=√2[cos( )+𝑖𝑠𝑒𝑛( )] ; 𝑘=0;1;…… ;(𝑛−1)
𝑛 𝑛

Como z y w son vértices consecutivos (raíces consecutivas ), tomamos: 𝑤𝑘 = 𝑧 y


𝑤𝑘 = 𝑤 . Luego:
2𝑘𝜋+𝜃 19𝜋
𝑤𝑘 = 𝑧 √2𝑒 (2𝑘𝜋+𝜃)𝑖/𝑛 = √2𝑒 19𝜋𝑖/24 24
= 24

2(𝑘+1)𝜋+𝜃 31𝜋
𝑤𝑘+1 = 𝑤 √2𝑒 [2(𝑘+1)𝜋+𝜃]𝑖/𝑛 = √2𝑒 31𝜋𝑖/𝑛 𝑛
= 2

Remplazando (4) de (5) tenemos: n=4 .Luego:

4 19𝜋𝑖 4 19𝜋𝑖
𝑛 4
𝑧 = 𝑧 = (√2 ) (𝑒 24 ) = 4𝑒 6

4 31𝜋 4 31𝜋𝑖
𝑧 𝑛 = 𝑤 𝑛 = (√2 ) (𝑒 24 ) = 4𝑒 6

19𝜋𝑖 12𝜋𝑖
𝑧 𝑛 + 𝑤 𝑛 = 4𝑒 6 (1 + 𝑒 6 )
19𝜋𝑖 19𝜋𝑖
= 4𝑒 6 (1 + 1) = 8𝑒 6

PROBLEMA53.Si 𝑧, 𝑤 y𝑣 son números complejos tales que:

𝑧 + 𝑤 + 𝑣 = 0 ; |𝑧| = |𝑤| = |𝑣| = 1 𝑦 𝐴𝑟𝑔(𝑣) > arg(𝑤) > 𝐴𝑟𝑔(𝑧)

Hallar los argumentos de 𝑤 y 𝑧 , Si Arg(𝑧) =30

Solución:

como𝑧 + 𝑤 + 𝑣 = 0 y |𝑧| = |𝑤| = |𝑣| = 1; entoncesseguna el problema 49, vemos


que 𝑧 y 𝑤 son los vértices de un triangulo equilátero inscrito en la circunferencia
unidad.
Como no nos dicen que algunos de los números es el cuadrado del otro (es decir:
𝑧 = 𝑤 2 ?, 𝑧 = 𝑣 2 ? O 𝑤 = 𝑣 2 , etc.), entonces no podemos pensar que 𝑧 ,𝑤 y 𝑣 sean
raíces cubicas de la unidad

Por analogía con el problema anterior, podemos suponer que 𝑧 , 𝑤 y 𝑣 son las
raíces cubicas de un numero complejo 𝑢.

Ahora por dato: 𝑧 = |𝑧|𝑒 𝜋𝑖/6 𝑧 = 𝑒 𝜋𝑖/6 𝑢 = 𝑧 3 = (𝑒 𝜋𝑖/6 )3 = 𝑒 𝜋𝑖/6 = 1

Las raíces cubicas de 𝑢 son:


𝜋 𝜋
2𝑘𝜋+ 2𝑘𝜋+
2 2
= cos ( ) + 𝑖𝑠𝑒𝑛( ); k= 0;1;2
3 3

De donde:

𝜋 𝜋 √3 1
𝑤0 = 𝑧 = 𝑐𝑜𝑠 + 𝑖𝑠𝑒𝑛 = +
6 6 2 2
5𝜋 5𝜋 √3 1
𝑤1 = 𝑤 = 𝑐𝑜𝑠 + 𝑖𝑠𝑒𝑛 =− +
6 6 2 2
3𝜋 3𝜋
𝑤2 = 𝑣 = 𝑐𝑜𝑠 + 𝑖𝑠𝑒𝑛 = −1
2 2
Luego, como 𝑤0 , 𝑤1 y 𝑤2 cumplen las condiciones del problema, se tiene que:
5𝜋 3𝜋
𝐴𝑟𝑔(𝑤) = 𝑦𝐴𝑟𝑔(𝑣) =
2 2
1
PROBLEMA 54: Encontrar las raíces enésimas (𝑛 < 18) de 𝑧𝜖 𝐶, si − 2 (3 +
𝑖√3) es una de ellas y si se conoce que:

11𝜋 17𝜋 𝑛 𝑛
[(1 − 𝑖) cos ( ) − (1 − 𝑖)cos( )] = 𝑖 2−𝑛 2 ⁄2
12 12

Solución:

17𝜋 11𝜋
Trabajando con la condición y teniendo en cuenta que cos ( 1 ) = −𝑠𝑒𝑛 ( 12 ) ,
2
se tiene
𝑛 𝑛
11𝜋 11𝜋 11𝜋 11𝜋 11𝜋 11𝜋 11𝜋 11𝜋
[cos( ) − 𝑖𝑐𝑜𝑠 ( ) + 𝑠𝑒𝑛 ( ) + 𝑖𝑠𝑒𝑛( )] = [[cos ( ) + 𝑖𝑠𝑒𝑛( )] − 𝑖 [cos ( ) + 𝑖𝑠𝑒𝑛( )]]
12 12 12 122 12 12 12 12

𝑛
11𝜋𝑛 1 1 𝑖(11𝜋𝑛) 𝑛 −𝑖𝜋 𝑛 𝑖(2𝑛)𝜋
=𝑒 12 [√2( − 𝑖)] = 𝑒 12 𝑒 2 (𝑒 4 )𝑛 = 22 𝑒 3
√2 √2

Pero del enunciado también se sabe que:


𝑛 𝑖𝜋 𝑛 𝑛𝜋 𝑛
𝑖 2−𝑛 22 = (𝑒 2 )2−𝑛 22 = 𝑒 𝑖(𝜋− 2 ) 22

Igualando (1) y (2), se obtiene:

𝑒 2𝜋𝑛𝑖/3 = 𝑒 𝑖(𝜋−𝑛𝜋/2)
2𝜋𝑛 𝑛𝜋
− (𝜋 − ) = 2𝑘𝜋
3 2
6(2𝑘 + 1)
𝑛= ; 𝑘 = 0; 1; 2;
7

Efectuando la prueba (𝑛𝜖𝑍 + ), tenemos

Para 𝑘 = 0 𝑛 = 6/7 (se descarta)

Para 𝑘 = 3 𝑛=6 (se acepta, ya que 𝑛 < 18)

Para 𝑘 = 10 𝑛 = 18 (se descarta, ya que 𝑛 < 18)

Luego las raíces que nos piden son las raíces sextas de z

Por dato,(−1/2)(3 + 𝑖√3) es una raíz sexta

Pasando a la forma polar


1
(− ) (3 + 𝑖√3) = √3𝑒 𝑖(7𝜋/6)
2
Por lo tanto
6
𝑧 = [√3𝑒 𝑖(7𝜋/6) ] = 33 𝑒 𝑖(7𝜋)

= −27 = 27𝑒 𝑖𝜋

Las raíces sextas de 𝑧 están dadas por


2𝑘𝜋 + 𝜋 2𝑘𝜋 + 𝜋
𝑤𝑘 = [cos( ) + 𝑖𝑠𝑒𝑛( )] ; 𝑘 = 0 ; 1 ; 2; … ; 5
6 6

RAICES RACIONALES DE UN POLINOMIO

PROBLEMA 55.Determinar 𝑝 de tal manera que:2𝑝𝑥 2 − 4𝑝𝑥 + 5𝑝 = 3𝑥 2 + 𝑥 − 8


tenga el producto de sus raíces igual a dos veces la suma.

Solución:
Ordenando el polinomio
(2𝑝 − 3)𝑥 2 − (4𝑝 + 1)𝑥 + (5𝑝 + 8) = 0

Por propiedad de las raíces:


Suma de raíces
4𝑝 + 1
2𝑝 − 3

Producto de raíces
5𝑝 + 8
2𝑝 − 3
De (1) y (2), y aplicando al condición del problema
5𝑝 + 8 4𝑝 + 1
= 2( )
2𝑝 − 3 2𝑝 − 3
Resolviendo se obtiene 𝑝 = 2

PROBLEMA 56:Determinar el polinomio 𝑃(𝑥) de grado 7, sabiendo que:

i) para 𝑥 = 3, se tiene : 𝑃(𝑥) = 𝑃(𝑥) = 𝑃"(𝑥) = 𝑃""(𝑥) = 0 𝑦 𝑃""(𝑥 ≠ 0)

ii) para 𝑥 = −2 , se tiene : 𝑃(𝑥) = 0 𝑦 𝑝"(𝑥) ≠ 0

iii) para 𝑥 = 4, se tiene: 𝑃(𝑥) = 𝑃(𝑥) = 0 𝑦 𝑃"(𝑥) ≠ 0

donde 𝑃(𝑥); 𝑃"(𝑥) ; 𝑃""(𝑥) ; 𝑦 𝑃(𝑥) denotan derivadas.

Solución

De acuerdo a los datos se observa que es una aplicación directa del teorema de
factos de multiplicidad 𝑘

De (i), concluimos que 𝑥 = 3 es una raíz de multiplicidad 4; luego, el polinomio


𝑃(𝑥) tiene un factor de la forma (𝑥 − 3)4

De igual modo, de (ii) obtenemos el factor (𝑥 + 2) y de (iii) el factor (𝑥 − 4)2

Luego el polinomio de grado 7 es:

𝑃(𝑥) = 𝑎(𝑥 − 3)4 (𝑥 + 2)(𝑥 − 4)2 ; 𝑎 es una constante

PROBLEMA 57. La operación de dividir el polinomio

𝑃(𝑥) = 𝑎0 𝑥 5 + 𝑎1 𝑥 4 + 𝑎2 𝑥 3 + 𝑎3 𝑥 2 + 𝑎4 𝑥 + 𝑎5

Entre (x-1), da los resultados que se observa en la siguiente tabla:

𝑎0 𝑎1 𝑎2 𝑎3 𝑎4 𝑎5
1 𝑏0 0 −45 13 17 9

Si se sabe que P(-1)=-125, factor izar el polinomio 𝑃(𝑥)

Solución:
Aplicando el método de la división sintética y observando la tabla dada, vemos que
:

𝑎0 = 𝑎0 ; 𝑎1 + 𝑎1 = 0 𝑎1 = −𝑎1

𝑎2 + 0 × 1 = −45 𝑎2 = −45
𝑎3 + (−45) × 1 = 13 𝑎 = 58

𝑎4 + 13 × 1 = 9 𝑎4 = 4

𝑎5 + 17 × 1 = 9 𝑎5 = −8

Luego el polinomio que de la forma

𝑃(𝑥) = 𝑎0 𝑥 5 − 𝑎0 𝑥 4 − 45𝑥 3 + 58𝑥 2 + 4𝑥 − 8

Como P(-1) =-125 entonces

𝑎0 (−1)5 − 𝑎0 (−1)4 − 45(−1)3 + 58(−1)2 + 4(−1) − 8 = −125

𝑎0 = 108

Luego, 𝑃(𝑥) = 108𝑥 5 − 108𝑥 4 − 45𝑥 3 + 58𝑥 2 + 4𝑥 − 8

Las posibles raíces racionales son de la forma 𝑟 = 𝑝/𝑞 , donde 𝑝 es un divisor de -


8 y 𝑞 es divisor de 108

Como 𝑃(0) = −8 𝑦 𝑃(1) = 9 > 0, enonces existe por lo menos una raíz real en
1
el intervalo [2 ; 1]

Eligiendo 𝑥 = 𝑟 = 2/3 , efectuando la tabla

108 − 108 − 45 58 4 8
2
72 − 24 − 46 8 8
3

108 − 36 − 64 12 12 0

Como P=(2/3)=0 x=2/3 es una raíz . Luego, el polinomio 𝑃(𝑥) es:

𝑃(𝑥) = (𝑥 − 2/3)(108𝑥 4 − 36𝑥 3 − 69𝑥 2 + 12𝑥 + 12)

Trabajando ahora con 𝑄(𝑥) = 108𝑥 4 − 36𝑥 3 − 69𝑥 2 + 12𝑥 + 12 y haciendo


nuevamente la prueba con 𝑥 = 2/3

108 − 36 − 69 12 12

2/3 72 24 − 30 − 12
108 36 − 45 − 18 0

Como Q(2/3) =0 x=2/3es una raíz , el polinomio P(x) puede escribirse como
𝑃(𝑥) = (𝑥 − 2/3)2 (108𝑥 3 + 36𝑥 2 − 45𝑥 − 18)

Comprobando nuevamente que 𝑥 = 2/3 es raíz , 𝑃(𝑥) puede escribirse como:

2 3
𝑃(𝑥) = (𝑥 − 3/2)3 (108𝑥 2 + 108𝑥 + 27) = 27 (𝑥 − ) (4𝑥 2 + 4𝑥 + 1)
3
𝑃(𝑥) = (3𝑥 − 2)3 (2𝑥 + 1)2

PROBLEMA 58.Probar que la suma y el producto de todas las raíces de

𝑃(𝑧) = 𝑎𝑛 𝑧 𝑛 + 𝑎𝑛−1 𝑧 𝑛−1 + ⋯ + 𝑎1 𝑧 + 𝑎0 ; 𝑎𝑛 ≠ 0


𝑎𝑛 −1 𝑎
Son: − y −1𝑛 . 𝑎 0 , respectivamente
𝑎 𝑛

Solución:

Sean 𝑧1 , 𝑧2 ; . . ; 𝑧𝑛 las raíces. Luego, P(z) en funci ón de las raíces se escribe

𝑃(𝑧) = 𝑎𝑛 (𝑧 − 𝑧1 )(𝑧 − 𝑧2 ) … (𝑧 − 𝑧𝑛 ) = 0

Efectuando: 𝑃(𝑥) = 𝑎𝑛 [𝑧 𝑛 − (𝑧1 + 𝑧2 + ⋯ 𝑧𝑛 )𝑧 𝑛−1 + ⋯ + (−1)𝑛 𝑧1 𝑧2 … 𝑧𝑛 ] = 0

= 𝑎𝑛 𝑧 𝑛 − 𝑎𝑛 (𝑧1 + 𝑧2 + ⋯ + 𝑧𝑛 )𝑧 𝑛−1 + ⋯ + 𝑎𝑛 (−1)𝑛 𝑧1 𝑧2 … 𝑧𝑛 = 0

Igualando coeficientes de𝑍 𝑛−1 :


𝑎𝑛−1
𝑎𝑛−1= −𝑎𝑛 (𝑧1 + 𝑧2 + ⋯ + 𝑧𝑛 ) ⟹ 𝑧1 + 𝑧2 + ⋯ + 𝑧𝑛 = −
𝑎𝑛
Igualando términos independientes:
𝑎0
𝑎0 = 𝑎𝑛 (−1)𝑛 𝑧1 𝑧2 … 𝑧𝑛 ⇒ 𝑧1 𝑧2 … 𝑧𝑛 = (−1)𝑛
𝑎𝑛
PROBLEMA 59. Sea𝑃(𝑧) = 𝑎𝑛 𝑧 𝑛 + 𝑎𝑛−1 𝑧 𝑛−1 + ⋯ + 𝑎1 + 𝑧 + 𝑎0 .
Demostrar que 𝑃(𝑧)es un polinomio de coeficientes reales, si y solo
̅̅̅̅̅̅ = 𝑃(𝑧̅)
si𝑃(𝑧)
Solución:
(⇒)Sea𝑃(𝑧)un polinomio de coeficientes reales; esto es,
𝑎𝑘 ∈ ℝ ; ∀ 𝑘 = 0 ; 1 ; … ; 𝑛
𝑛

Por definicion 𝑃(𝑧) = ∑ 𝑎𝑘 𝑧 𝑘 , luego:


𝑘=1

̅̅̅̅̅̅̅̅̅̅̅̅
𝑛 𝑛 𝑛 𝑛

𝑃(𝑧) = ∑ 𝑎𝑘 𝑧 𝑘 = ∑ ̅̅̅̅̅̅
⇒ ̅̅̅̅̅̅ 𝑎𝑘 𝑧 𝑘 = ∑ 𝑎𝑘 ̅̅̅
𝑧 𝑘 = ∑ 𝑎𝑘 𝑧̅ 𝑘 = 𝑃(𝑧̅)
𝑘=1 𝑘=1 𝑘=1 𝑘=1
(⇐)

̅̅̅̅̅̅ = 𝑃(𝑧̅)
𝑃(𝑧)

̅̅̅̅̅̅̅̅̅̅̅̅
𝑛 𝑛

∑ 𝑎𝑘 𝑧 𝑘 = ∑ 𝑎𝑘 𝑧̅ 𝑘
𝑘=1 𝑘=1

𝑛 𝑛

∑ ̅̅̅̅̅̅
𝑎𝑘 𝑧 𝑘 = ∑ 𝑎𝑘 𝑧̅ 𝑘
𝑘=1 𝑘=1
𝑛 𝑛

𝑎𝑘 ̅̅̅𝑘 = ∑ 𝑎𝑘 𝑧̅ 𝑘
∑ ̅̅̅𝑧
𝑘=1 𝑘=1
𝑛 𝑛

𝑎𝑘 ̅̅̅𝑘 = ∑ 𝑎𝑘 𝑧̅ 𝑘
∑ ̅̅̅𝑧
𝑘=1 𝑘=1

𝑎𝑘 = 𝑎𝑘
̅̅̅
Donde 𝑎𝑘 es un número real ∀ 𝑘 = 0 ; 1 … ; 𝑛 .
PROBLEMA 60

a) Demostrar que el producto de todas las raíces 𝑛-ésimas de la

unidad es (−1)𝑛+1
b) Todo polígono definido en ℂ de grado 𝑛 positivo impar, con

coeficientes reales, tiene por lo menos una raíz real.

c) Si 𝑟𝑒 −𝔦𝜃 (𝑟 > 0 ; 𝜃 ∈ ℝ)es una raíz de𝑃(𝑧), polinomio con

coeficientes enteros, definido en ℂ , de grado 𝑛 positivo;

demostrar que 𝑟𝑒 𝔦(𝜃−2𝑘𝜋) es otra raíz, para 𝑘 ∈ ℤ .

Solución:

a) La ecuación que produce las raíces de la unidad es:

𝑧 𝑛 = 1 ⇔ 𝑧 𝑛 + (−1) = 0

Luego, el polinomio es: 𝑃(𝑧) = 𝑧 𝑛 + (−1) = 0

Por el problema 58, sean: 𝑧1 ; 𝑧2 ; … ; 𝑧𝑛 las raíces de la unidad,

𝑎0 (−1)
⇒ 𝑧1 ; 𝑧2 ; … ; 𝑧𝑛 = (−1)𝑛 = (−1)𝑛 = (−1)𝑛+1
𝑎𝑛 1

Pues 𝑎0 = −1 y 𝑎𝑛 = 1

b) Por teorema, 𝑃(𝑧) tiene 𝑛 raíces. Como sus coeficientes son

reales, entonces sus 𝑛 raíces se distribuyen por parejas: para

cada raíz compleja (y no estrictamente real), su conjugado

también es raíz.

Como el grado 𝑛 es impar, entonces ha de existir una raíz real

(no compleja) necesariamente, ya que si todas sus raíces fuesen

estrictamente complejas, sus conjugados también serían raíces;


el número total de raíces sería un numero par, contradiciendo el

hecho de que el grado 𝑛 es impar.

Luego, 𝑃(𝑧) tiene por lo menos una raíz real, si es que 𝑛 fuese

impar.

c) Por teorema, como𝑃(𝑧) es un polinomio con coeficientes reales;

entonces si 𝑧0 es una raíz de 𝑃(𝑧), también lo es 𝑧̅0 .

Luego, sea 𝑧0 = 𝑟𝑒 −𝔦𝜃 una raíz de 𝑃(𝑧) ⇒ 𝑃(𝑧0 ) = 0

Ahora:

𝑟𝑒 𝔦(𝜃−2𝑘𝜋) = 𝑟[cos(𝜃 − 2𝑘𝜋) + 𝔦 sen(𝜃 − 2𝑘𝜋)] = 𝑟[cos 𝜃 + 𝔦 sen 𝜃] = 𝑟𝑒 𝔦𝜃 = 𝑧̅0

⇒ 𝑃(𝑍0̅ ) = 0 ⇒ 𝑟𝑒 𝔦(𝜃−2𝑘𝜋) es una raíz de 𝑃(𝑧)

PROBLEMA 61. Demostrar que si 𝑃(𝑥)es un polinomio definido enℂ ,

de grado 𝑛positivo, con coeficientes puramente imaginarios, entonces;


2
̅̅̅̅̅̅
𝑃(𝑥̅ )[2𝑃 ̅̅̅̅̅̅
(𝑥) + 𝑃(𝑥̅ )] = − [𝑃 (𝑥)]

Solución:

Sea 𝑥̅la conjugada de 𝑥.

⇒ 𝑝(𝑥) = ∑ (𝔦𝑎𝑘 ) 𝑥 𝑘 ; donde 𝑎𝑘 ∈ ℝ


𝑘=0
𝑛

Luego: 𝑃(𝑥) ) = ∑ (𝔦𝑎𝑘 ) 𝑥 𝑘 ; 𝑦


𝑘=0

̅̅̅̅̅̅̅̅̅̅̅̅̅̅̅
𝑛 𝑛 𝑛 𝑛 𝑛
̅̅̅̅̅̅ 𝑘 ̅̅̅̅̅̅̅̅̅̅
𝑘
𝑃(𝑥) = ∑(𝔦𝑎𝑘 ) 𝑥 = ∑(𝔦𝑎𝑘 )𝑥 ) = ∑(𝔦𝑎 ̅̅̅̅
̅ 𝑘 ) 𝑥 = ∑(−𝔦𝑎𝑘 ) 𝑥̅ = − ∑(𝔦𝑎𝑘 ) 𝑥̅ 𝑘 = −𝑃(𝑥̅ )
𝑘 𝑘

𝑘=0 𝑘=0 𝑘=0 𝑘=0 𝑘=0

Formamos:

̅̅̅̅̅̅ + 𝑃(𝑥̅ ) = −2𝑃(𝑥̅ ) + 𝑃(𝑥̅ ) = −𝑃(𝑥̅ ) = 𝑃(𝑥)


2𝑃(𝑥) ̅̅̅̅̅̅

Partimos de la parte izquierda para efectuar la demostración:

̅̅̅̅̅̅
𝑃(𝑥̅ )[2𝑃 ̅̅̅̅̅̅]2
(𝑥) + 𝑃(𝑥̅ )] = 𝑃(𝑥̅ )[−𝑃(𝑥̅ )] = −[𝑃(𝑥̅ )]2 = −[𝑃(𝑥)

PROBLEMA 62. Calcular las raíces racionales de:

𝑃(𝑥) = 240𝑥 5 + 572𝑥 4 − 564𝑥 3 − 1257𝑥 2 − 31𝑥 + 60 = 0

Solución:

Las posibles raíces racionales son las de la forma 𝑟 = 𝑝⁄𝑞, donde es

un divisor de 60 y𝑞 un divisor de 240 .

Vemos que 𝑝(1) < 0 y; 𝑝(2) > 0 luego, existe por lo menos una raíz

real en〈1; 2〉 .

Tomamos las raíces racionales comprendidas en dicho intervalo; así

𝑥 = 3⁄2 es raíz.
Para comprobar efectuamos las operaciones de la tabla:

-
240 572 -564 1257 -31 60

3⁄2 360 1398 1251 -9 -60

240 932 834 -6 -40 0

⇒ 𝑃(𝑥) = (x − 3⁄2)(240x 4 + 932x 3 + 834x 2 − 6x − 40) = 0

⇔ 𝑃(𝑥) = (2x − 3)(120x 4 + 466x 3 + 417x 2 − 3x − 20) = 0

También: 𝑃(0) > 0 y 𝑃(−1) < 0 ; luego existe por lo menos una raíz

real en 〈−1; 0〉.

También se deduce que 𝑥 = −1/4 satisface. Así:

𝑃(𝑥) = (2𝑥 − 3)(𝑥 + 1/4)(1203𝑥 3 + 436𝑥 2 + 308𝑥 − 80) = 0

⇔ 𝑃(𝑥) = (2𝑥 − 3)(4𝑥 + 1)(30𝑥 3 + 109𝑥 2 + 77𝑥 − 20) = 0

De manera similar encontramos 𝑥 = −5/2 y queda:


(𝑃(𝑥) = (2𝑥 − 3)(4𝑥 + 1) (2𝑥 + 5)(15𝑥 2 + 17𝑥 − 4) = 0

Resolviendo la ecuación de segundo grado, vemos que las cinco

raíces racionales son:

𝑥1 = 3/2 ; 𝑥2 = 1/4; 𝑥3 = −5/2; 𝑥4 = −4/3 ; 𝑥5 = 1/5

PROBLEMA 63. Resolver la ecuación 𝑥 8 + 4𝑥 6 − 4𝑥 2 − 1 = 0 .

Solución:

Las posibles raíces racionales son: 1 y −1. Se deducen que ambas

satisfacen, luego:

𝑃(𝑥) = 𝑥 8 + 4𝑥 6 − 4𝑥 2 − 1 = 0

= (𝑥 − 1)(𝑥 + 1)(𝑥 6 + 5𝑥 4 + 5𝑥 2 + 1) = 0

Ahora factorizamos:

𝑄(𝑥) = 𝑥 6 + 5𝑥 4 + 5𝑥 2 + 1

= (𝑥 6 + 1) + 5𝑥 2 (𝑥 2 + 1)

= (𝑥 2 + 1)(𝑥 4 − 𝑥 2 + 1) + 5𝑥 2 (𝑥 2 + 1)

= (𝑥 2 + 1)(𝑥 4 + 4𝑥 2 + 1)

= (𝑥 2 + 1)(𝑥 2 + 2 − √3)(𝑥 2 + 2 + √3)

En (1):

𝑃(𝑥) = (𝑥 − 1)(𝑥 + 1)(𝑥 2 + 1)(𝑥 2 + 2 − √3)(𝑥 2 + 2 + √3) = 0

Calculo de las raíces:

De (𝑥 − 1) y (𝑥 + 1) :𝑥1 = 1 ; 𝑥2 = −1
De 𝑥 2 + 1 = 0 ; 𝑥3 = 𝑖 ; 𝑥4 = −𝑖

De 𝑥 2 + 2 − √3 = 0 ∶ 𝑥5 = −√2 − √3 𝑖 ; 𝑥6 = √2 − √3 𝑖

De 𝑥 2 + 2 + √3 = 0 ∶ 𝑥7 = −√2 + √3 𝑖 y 𝑥8 = √2 + √3 𝑖

PROBLEMA 64. Para el polinomio

𝑃(𝑥) = (2𝑚 − 𝑛)𝑥 𝑛 + 𝑛𝑥 𝑛−𝑚 − 𝑛𝑥 𝑚 + (𝑛 − 2𝑚)¿de qué multiplicidad

es la raíz 1?

Solución:

Comprobando, se verifica que𝑃(1) = 0

Primera derivada:

𝑃′ (𝑥) = 𝑛(2𝑚 − 𝑛)𝑥 𝑛−1 + 𝑛(𝑛 − 𝑚)𝑥 𝑛−𝑚−1 − 𝑛𝑚𝑥 𝑚−1 ⇒ 𝑃′ (1) = 0

Segunda derivada:

𝑃" (𝑥) = 𝑛(𝑛 − 1)(2𝑚 − 𝑛)𝑥 𝑛−2

+ 𝑛(𝑛 − 𝑚)(𝑛 − 𝑚 − 1)𝑥 𝑛−𝑚−2

− 𝑛𝑚(𝑚 − 1)𝑥 𝑚−2 ⇒ 𝑃"(1) = 0

Pero como𝑃′′′ (1) ≠ 0; entones 1 es raíz de multiplicidad 3.

PROBLEMA 65. Justifique la verdad o falsead de las proposiciones:


18
a) Si𝓇 − 𝓈 𝑖 (𝓇 ; 𝓈 ∈ ℝ − {0})es un valor de √1 − 𝑖 , entonces 𝓇 +

𝓈 𝑖 también lo es.
b) 𝓇ℯ 𝜋𝑖/5 ; −𝓇ℯ −2𝜋𝑖/5 ; ̅̅̅̅̅̅̅̅̅̅
𝓇ℯ 𝑖9𝜋/5 (𝓇 > 0)representan las 3 raíces

cúbicas de un número complejo.

c) Si los coeficientes del polinomio 𝑃(𝑥)(definido en ℂ) son

puramente imaginarios y si 𝑧 ∈ (ℂ − ℝ)es raíz de𝑃(𝑥) ,

entonces 𝑧̅también es raíz.

d) Para todo 𝑎 ; 𝑏 ; 𝑐 ∈ ℂ − {0}, el número

𝑎2 |𝑏|2 |𝑖𝑐|2 − 𝑖 18 |𝑖𝑎|2 𝑏 2 𝑐 2


∈ ℝ
𝑖 21 (𝑎2 𝑏 2 |𝑐|2 − |𝑎|2 |𝑖𝑏|2 𝑐 2 )

Solución:
18
a) Como 𝑧 = 𝓇 − 𝓈 𝑖 es un valor de √1 − 𝑖
18
⇒ 𝒛 = √1 − 𝑖 ⇒ 𝑧 18 = 1 − 𝑖 ⇒ 𝑧18 + (𝑖 − 1)

=0 … (1)

⇒ 𝑃(𝑧) = 𝑧 18 + (𝑖 − 1) = 0

Como 𝑃(𝑧)es un polinomio de coeficientes complejos, entonces

𝓇 + 𝓈 𝑖no es su raíz luego, la proposición es FALSA.


3
b) i)(−𝓇ℯ −𝑖2𝜋/15 ) = −𝓇 3 ℯ −𝑖2𝜋/5

2𝜋 2𝜋
= −𝓇 3 [cos ( ) − 𝑖 sen ( )]
5 5
𝜋 𝜋
= 𝓇 3 [− sen ( ) + 𝑖 cos ( )]
10 10

= 𝑧
3
ii)(𝓇ℯ 𝑖𝜋/5 ) = 𝓇 3 ℯ 𝑖3𝜋/5

3𝜋 3𝜋
= 𝓇 3 [cos ( ) + 𝑖 sen ( )]
5 5
𝜋 𝜋
= 𝓇 3 [− sen ( ) + 𝑖 cos ( )]
10 10

= 𝑧

̅̅̅̅̅̅̅̅̅̅̅̅ 3
iii)(𝓇ℯ 𝑖9𝜋/5 )3 = (𝓇ℯ −𝑖9𝜋/5 )

= 𝓇 3 ℯ −𝑖27𝜋/5

27𝜋 27𝜋
= 𝓇 3 [cos ( ) − 𝑖 sen ( )]
5 5
𝜋 𝜋
= 𝓇 3 [− sen ( ) + 𝑖 cos ( )]
10 10

Luego, la proposición es VERDADERA.

c) la proposición es VERADDERA; porque los coeficientes del

polinomio son de la forma 𝑟𝑖, se puede factorizar el valor de 𝑖y

quedará un polinomio de coeficientes reales; y por lo tanto, si

𝑧es una raíz del polinomio, también lo será𝑧̅.

𝑎2 |𝑏|2 |𝑖𝑐|2 − 𝑖 18 |𝑖𝑎|2 𝑏2 𝑐 2 𝑎2 𝑏𝑏̅𝑐𝑐̅−(−1)9∗ 𝑎𝑎̅𝑏2 𝑐 2 𝑎𝑏̅𝑐̅+𝑎̅𝑏𝑐


d) = =
𝑖 21 (𝑎2 𝑏2 | 𝑐|2 −|𝑎|2 |𝑖𝑏|2 𝑐 2) 𝑖(𝑎2 𝑏2 𝑐𝑐̅−𝑎𝑎̅𝑏𝑏̅𝑐 2 ) 𝑖(𝑎𝑏𝑐̅−𝑎̅𝑏̅𝑐)

Luego, la proposición es VERDADERA.

PROBLEMA 66.
a) Si𝑧 = 105 − 74 ; 3𝑒 𝑖𝜋/5 es raíz 25-ésima de 𝑧 3 + 2|𝑖 7 𝑧|3 + 3𝐼𝑚(𝑧̅) + 𝑧 3 −

3. se desea conocer si con −3𝑒 𝑖14𝜋/5 ocurre lo mismo.

b) Hallar el término constante del polinomio 𝑃(𝑥)del menor grado,

definido en ℂ , con coeficiente puramente imaginarios, si se sabe

que 2⁄3 ; −1⁄2 ; ± (𝑖 + √7)son raíces de𝑃(𝑥).

Solución:

a) Sea𝑤 = 𝑧̅ 3 + 2 |𝑖 7 𝑧|3 + 2𝐼𝑚 (𝑧̅) + 𝑧 3 − 3

Como 𝑧1 = 3𝑒 𝑖𝜋/5 es raíz25-ésima de𝑤, entones:

25
𝑤 = 𝑧125 = (3𝑒 𝑖𝜋/5 ) = 325

Sea:
𝑧2 =-3ei14π/5 =3(-1)ei14π/5 =3eiπ ei14π/5 =3eiπ ei4π/5 =3ei(9π/5)
Si 𝑧2 es raíz, entonces 𝑎225 = 𝑤, veamos:
𝑎225 = (3ei(9π/5) )25 = 325 ei45π = 325 ei5π = 𝑤
Luego, 𝑧2 también es raíz 25-ésima de 𝑤 = 𝑧̅ 3 + 2|𝑖 7 𝑧|3 + 2𝐼𝑚(𝑧̅) + 𝑧 3 − 3
1) El polígono buscado tiene la forma: p(x) = (𝑎𝑛 𝑖)𝑥 𝑛 + (𝑎𝑛+1 𝑖)𝑥 𝑛+1 + ⋯ + (𝑎0 𝑖),
donde 𝑎0 ; 𝑎1 ; … ; 𝑎𝑛 son números reales.
p(x) Puede escribirse también de la siguiente forma:
p(x) = i[an x n + an−1 x n−1 + + ⋯ + a1 x + a0 ] = iQ(x),
Donde los coeficientes de Q(x)son números reales. Luego, si raíces de Q(x) (y
en consecuencia, las raíces de p(x)) son números complejos (estrictamente),
sus conjugadas también serán raíces de p(x).
Luego, las raíces de p(x) son:
2 1
𝑥1 = ; 𝑥2 = ; 𝑥3 = i + √7; 𝑥4 = 1 − √7; 𝑥5 = −i − √7; 𝑥6 = −i + √7
3 2
El polinomio p(x) de menor grado tendrá 6 raíces, entonces n = 6 y p(x)en
función de sus raíces tiene la forma:
p(x) = i(x − x1 )(x − x2 )(𝑥 − 𝑥3 )(𝑥 − 𝑥4 )(𝑥 − 𝑥5 )(𝑥 − 𝑥6 )
El término constante se obtiene haciendo 𝑥 = 0, así tenemos:
𝑝(0) = 𝑖(−𝑥1 )(−𝑥2 )(−𝑥3 )(−𝑥4 )(−𝑥5 )(−𝑥6 ) = −64𝑖/3
Problema 67. Descomponer:
p(x) = 3x 7 + (11/6)x 6 − 11x 5 + (37/2)x 4 + 18𝑥 3 + 28𝑥 2 − 8𝑥 − 8
En el mayor numero de factores con coeficientes reales.
Solución:
Como p(x) se puede expresar de la siguiente forma:
p(x) = (1/2)[6x 7 + 11x 6 − 22x 5 − 37x 4 + 36x 3 + 56x 2 − 16x − 16]
Trataremos de factorizar la expresión entre corchetes, mediante la regla de Ruffini:
p(x) = (1/2)(x + 2)(2x + 1)(3x − 2)(x 4 − 3x 2 + 4)
por teoría sabemos que se puede factorizar x 4 − 3x 2 + 4 = 0 en dos factores
cuadráticos de la forma (x 2 + x + b), para lo cual calculamos las raíces de:
x 4 − 3x 2 + 4 = 0 …(α)

Veamos, si hacemos que x 2 = y; para luego reemplazarlo en (α) se tiene:


y 2 − 3y + 4 = 0
Se obtienen las raíces:
3+√7i 3−√7i
𝑦= ; 𝑦=
2 2

O equivalentemente:
3+√7i
𝑥2 = …(1)
2
3−√7i
𝑥2 = …(2)
2

Las raíces de la ecuación (1):


3 + √7i √7
𝑥2 = = 2ℯ iθ ; 𝜃 = arc tg
2 3
Las raíces están dadas por:
2kπ + θ 2kπ + θ
wk = √2 [cos ( ) + i sen( )] ; k = 0 ; 1
2 2

Para:
k = 0 ⟹ w0 = √2[cos(θ/2) + i sen(θ/2)]
2π+θ 2π+θ
k = 1 ⟹ w1 = √2 [cos ( ) + i sen( )]
2 2

Pero:θ = arc tg(√7/3), θ pertenece al primer cuadrante.


O también: 𝑡𝑔θ = √7/3, entonces : cos θ = 3/4 ; sen(√7/4)
√7 1
⟹ cos(θ/2) = 2√2 y sen(θ/2) = 2√2

Reemplazando estos valores en (3) y (4) respectivamente:


√7 1 √7 i
w0 = √2 [2√2 + i 2√2] = +2
2
√7 1 √7 i
w1 = √2 [− 2√2 + i 2√2] = − −2
2

Con la ecuación (1) en realidad proviene de la ecuación (α) (grado 4), la cual es
un polinomio con coeficientes reales; y como las raíces obtenidas son complejas,
entonces las otras dos que faltan, correspondientes, correspondientes a (2), son
las conjugadas de las anteriores.
√7 i √7 i
Entonces para (2), las raíces son: −2 ; − +2
2 2

Luego, (α) en función de sus raíces se expresa del modo siguiente:


√7 + i −√7 − i √7 − i −√7 + i
x 4 − 3x 2 + 4 = [x − ( )] [x − ( )] [x − ( )] [x − ( )]
2 2 2 2
Conjugadas
Multiplicamos las funciones conjugadas, para obtener factores cuadráticos de la
forma: x 2 + 𝒶x + b. Luego tenemos:
x 4 − 3x 2 + 4 = (x 2 − √7x + 2)(x 2 + √7x + 2)
⟹ p(x) = (1/2)(x + 2)(2x + 1)(3x − 2)(x 2 − √7x + 2)(x 2 + √7x + 2)
Problema 68. Si 4ℯ 5πi/3, es raíz de p(x) = 2x 8 − 8x 7 + 32x 6 + 54x 2 + 𝒶x + b ,
donde 𝒶, b son enteros; descomponer p(x) en el mayor número posible de factores
con coeficientes reales.
Solución:
Por condición del enunciado, si 4ℯ 5πi/3 es una raíz de p(x), entonces se cumple:
p(4ℯ 5πi/3 ) = (2𝒶 + b − 432) − √3𝑖 (2𝒶 + 432) = 0
De donde;
2𝒶 + b − 432 = 0
⟹{ ; ⟹ 𝒶 = −216 ; b = 864
2𝒶 + 432 = 0
Además, como 𝑧 = 4ℯ 5πi/3es una raíz de p(x) que tiene coeficientes reales,
entonces el conjugado 𝑧 = 4ℯ −5πi/3 también es otra raíz de p(x).
Por lo tanto, (𝑥 − 4ℯ 5πi/3 )(𝑥 − 4ℯ −5πi/3 )son factores de p(x). Desarrollando.
ℯ 5πi/3 + ℯ −5πi/3
(𝑥 − 4ℯ 5πi/3 )(𝑥 − 4ℯ −5πi/3 ) = 𝑥 2 − 8 [ ] 𝑥 + 16
2
= 𝑥 2 − 8 cos(5𝜋/3) 𝑥 + 16
= 𝑥 2 − 4𝑥 + 16
Así se tiene que:
p(x) = 2(𝑥 2 − 4𝑥 + 16)𝑄(𝑥)
Y efectuando la división de p(x) entre (𝑥 4 − 3𝑥 2 + 9):
p(x) = 2(𝑥 2 − 4𝑥 + 16)(𝑥 6 + 27)
⟹ p(x) = 2(𝑥 2 − 4𝑥 + 16)(𝑥 2 + 3)(𝑥 4 − 3𝑥 2 + 9)
Ahora calculamos las raíces de (𝑥 4 − 3𝑥 2 + 9):
x 4 − 3x 2 + 4 = 0 …(α)
Hacemos 𝑥 2 = 𝑦, luego reemplazamos en (𝛼):
𝑦 4 − 3𝑦 + 9 = 0
De donde se obtienen las raíces:
3 + 3√3i 3 − 3√3i
𝑦= ; 𝑦=
2 2

O equivalentemente:
3+3√3i
𝑥2 = …(1)
2
3−3√3i
𝑥2 = …(2)
2

Para(𝟏):
3 + 3√3i
𝑥2 = = 3ℯ iπ/3
2
Las raíces están dadas por:
2kπ + π/3 2kπ + π/3
wk = √3 [cos ( ) + i sen( )] ; k = 0 ; 1
2 2
Para:
π π 3 √3
k = 0 ⟹ w0 = √3 [cos + i sen ] = + 𝑖
6 6 2 2
π π 3 √3
k = 1 ⟹ w1 = √3 [cos (π + ) + i sen(π + )] = − − 𝑖
6 6 2 2
Como la ecuación (1) proviene de la ecuación (𝛼) (𝑔𝑟𝑎𝑑𝑜 4), la cual es un
polinomio con coeficientes reales y como las raíces obtenidas son complejas;
entonces las otras dos raíces que faltan, correspondientes a la ecuación (2)son
las conjugadas de las anteriores. Así:
3 √3 3 √3
Para (2): las raíces son: – 𝑖 ; - + 𝑖
2 2 2 2

Luego, (α) en función de sus raíces, se expresa del modo siguiente:


3 √3 3 √3 3 √3 3 √3
x 4 − 3𝑥 2 + 9 = [𝑥 − (2 + 𝑖)] [𝑥 − (− 2 − 𝑖)] [𝑥 − (2 − 𝑖)] [𝑥 − (− 2 + 𝑖)]
2 2 2 2

Conjugadas
Multiplicamos los factores conjugados para obtener los factores
cuadráticos con coeficientes reales. Así:
𝑥 2 − 3𝑥 2 + 9 = (𝑥 2 − 3𝑥 + 3)(𝑥 2 + 3𝑥 + 3)
Luego,
P(x) = 2(𝑥 2 − 4𝑥 + 16)(𝑥 2 + 3)(𝑥 2 − 3𝑥 + 3)(𝑥 2 + 3𝑥 + 3)
PROBLEMA 69. Dado el polinomio:
P(x) = 3𝑥 6 + (1 + 3𝑖)𝑥 5 + 𝑖𝑥4 + 12𝑥 2 + 4(1 + 3𝑖)𝑥 + 4𝑖
Tal que P (𝑒 −𝑖𝜋/2 ) = 0. Expresar P(x) como un producto de factores, el máximo
número posible de los cuales sea con coeficientes reales.

Solución:

Por definición: 𝑒 −𝑖𝜋/2 = −𝑖, entonces:

P(-𝑖)=0 P(x) =(𝑥 + 𝑖)𝑄(𝑥)

Dividiendo P(x) entre (𝑥 + 𝑖) se obtiene:

P(x) = (𝑥 + 𝑖)(3𝑥 5 + 𝑥 4 + 12𝑥 + 4)

= (𝑥 + 𝑖)[𝑥 4 (3𝑥 + 1) + 4(3𝑥 + 1)]

= (𝑥 + 𝑖)(𝑥 4 + 4)(3𝑥 + 1)

Pero se tiene que: 𝑥 4 + 4 = (𝑥 2 + 2𝑖)(𝑥 2 − 2𝑖)

(I) (II)
Operando en (I) para obtener sus raíces:

𝑥 2 + 2𝑖 = 0 𝑥 2 = −2𝑖 = 2𝑒 𝑖(3𝜋/2)
2𝑘𝜋 + 3𝜋/2 2𝑘𝜋 + 3𝜋/2
𝑊𝑘 = √2 [𝑐𝑜𝑠 ( ) + 𝑖 𝑠𝑒𝑛 ( )] ; 𝑘 = 0 ; 1
2 2
3𝜋 3𝜋
Para: k=0 𝑤 = √2 [𝑐𝑜𝑠 ( 4 ) + 𝑖 𝑠𝑒𝑛 ( 4 )] = −1 + 𝑖
0

3𝜋 3𝜋
K=1 w = √2 [𝑐𝑜𝑠 (𝜋 ) + 𝑖 𝑠𝑒𝑛 (𝜋 + )] = 1 − 𝑖
1 4 4

Operando en (II):

𝑥 2 − 2𝑖 = 0 𝑥 2 = 2𝑖

De donde se obtiene sus raices: -1- 𝑖 y 1+ 𝑖

Luego en (2):

𝑥 4 + 4 = [𝑥 − (−1 + 𝑖)][𝑥 − (−1 − 𝑖)][𝑥 − (1 + 𝑖)]

= (𝑥 2 + 2𝑥 + 2)(𝑥 2 − 2𝑥 + 2)

Finalmente en (1):

P(x)=(𝑥 + 𝑖)(3𝑥 + 1)(𝑥 2 + 2𝑥 + 2)(𝑥 2 − 2𝑥 + 2)

Problema 70. Dado el problema:


1 1 2 2
𝑃(𝑥) = 2(3 + 𝑖)𝑥 5 + (17𝑖 + 33)𝑥 4 + (24 + 7𝑖)𝑥 3 + (27 + 14𝑖)𝑥 2 − (5𝑖 + 12)𝑥 − 4(𝑖 + 2)
3 3 3 3

Encontrar todas sus raices.


Solucion:

Reuninedo los factores del polinomio que contiene 𝑖:


1 1 2 2
𝑃(𝑥) = 2(3 + 𝑖)𝑥 5 + (17𝑖 + 33)𝑥 4 + (24 + 7𝑖)𝑥 3 + (27 + 14𝑖)𝑥 2 − (5𝑖 + 12)𝑥 − 4(𝑖 + 2)
3 3 3 3
=
17 7 28
6𝑥 5 + 11𝑥 4 + 8𝑥 3 + 18𝑥 2 − 8𝑥 − 8 + 𝑖 (2𝑥 5 + 𝑥4 + 𝑥3 + 𝑥2 −
3 3 3
10
𝑥 − 4)
3

Luego, P(x) tiene la forma P(x) = Q(x) + 𝑖𝑅(𝑥)


De donde se deduce que si r = x0 es una raiz de P(x) entonces r = x0 es una raiz
o cero de Q(x) y R(x), respectivamente. Luego el problema se reduce a encontrar
las raíces o ceros comunes de Q(x) y R(x).

Veamos esto:

6 11 8 18 -8 -8

2/3 4 10 12 20 8
6 15 18 30 12 0
6 17 7 28 -10 -12

2/3 4 14 14 28 12
6 21 21 42 18 0
Factorizamos el 3 y tenemos:
2 𝑖
𝑃(𝑥) = 3 (𝑥 − ) [2𝑥 4 + 5𝑥 3 + 6𝑥 2 + 10𝑥 + 4 + (2𝑥 4 + 7𝑥 3 + 7𝑥 2 + 14𝑥 + 6)]
3 3
Buscando nuevamente el cero común:

2 5 6 10 4

-1/2 -1 -2 -2 -4
2 4 4 8 0

2 7 7 14 6

-1/2 -1 -3 -2 -6
2 6 4 12 0
Tenemos entonces:
2 1 𝑖
𝑃(𝑥) = 3 (𝑥 − ) (𝑥 + ) [2𝑥 3 + 4𝑥 2 + 4𝑥 + 8 + (2𝑥 3 + 6𝑥 2 + 4𝑥 + 12)]
3 2 3
2 1 1
𝑃(𝑥) = 6 (𝑥 − 3) (𝑥 + 2) [𝑥 3 + 2𝑥 2 + 2𝑥 + 4 + 3 (𝑥 3 + 3𝑥 2 + 2𝑥 + 6)]

Como los polinomios que quedan son de grado 3, procedemos a encontrar sus
ceros ó raíces, por separado:

𝑥 3 + 2𝑥 2 + 2𝑥 + 4 = (𝑥 + 2)(𝑥 2 + 2) = (𝑥 + 2)(𝑥 − √2𝑖)(𝑥 + √2𝑖)

𝑥 3 + 3𝑥 2 + 2𝑥 + 6 = (𝑥 + 3)(𝑥 2 + 2) = (𝑥 + 3)(𝑥 − √2𝑖)(𝑥 + √2𝑖)

Finalmente se tiene:

2 1
𝑃(𝑥) = 6 (𝑥 − ) (𝑥 + ) [(𝑥 + 2)(𝑥 − √2𝑖)(𝑥 + √2𝑖)
3 2
𝑖
+ (𝑥 + 3)(𝑥 − √2𝑖)(𝑥 + √2𝑖)]
3
2 1 𝑖
𝑃(𝑥) = 6 (𝑥 − ) (𝑥 + ) (𝑥 − √2𝑖)(𝑥 + √2𝑖) [(𝑥 + 2) + (𝑥 + 3)]
3 2 3
Luego las raíces son:
2 1 3
x1 = ; x2 = − ; x3 = √2 𝑖 ; x4 = −√2 𝑖 ; x5 = − (7 + 𝑖)
3 2 10
RAICES IRRACIONALES DE UN POLINOMIO
PROBLEMA 71. Gráficar y hallar todas las raíces reales de:

𝑃(𝑥) = 2𝑥 3 − 6𝑥 2 − 5 = 0, con un error menor o igual que 10−2

Solucion:

Como el grado de P(x) es impar, tiene al menos una raíz real.

Ahora como 𝑃(𝑥) = 2𝑥 3 − 6𝑥 2 − 5, entonces su derivada:

𝑃′(𝑥) = 6𝑥(𝑥 − 2)

𝑃′(𝑥) > 0 para x > 2 ; 𝑃′(𝑥) < 0 para 0 < x < 2 y 𝑃′(𝑥) > 0 para x < 0, luego, 𝑃(𝑥) es
creciente en x ∈ 〈−∞ ; 0〉 ∪ 〈2 ; ∞〉 y 𝑃(𝑥) es decreciente en x ∈ 〈0 ; 2〉.

Un esbozo del gráfico es:

y
-1
1 2 3 4 x
Como P(x) = -5<0 y P(x)=27>0;

Entonces P(x) tiene al menos una

-5 raíz 𝑟 ∈< 3; 4 > .

Por el análisis efectuado, P(x)


Tiene una sola raíz real

almenos una raíz 𝑟 ∈< 2; ∞ >

-13

Aplicando el método de Newton, eligiendo x0 = 3 como valor inicial:

k x𝑘 𝑃(x𝑘 ) 𝑃′ (x𝑘 )

0 3 -5 18

1 3,28 1,02 25,195

2 3,24 0,039 24,105

3 3,23 -0,194

Como 𝑃(3,24) > 0 𝑦 𝑃(3,23) < 0, entonces la raíz 𝑟 ∈< 3,23 ; 3,24 >. Veamos el
error:
|x4 − x3 | = 0,01 = 10−2 𝑟 = 3,23

PROBLEMA 72. Sea 𝑓(𝑥) = 2𝑥 2 − 𝑥 4 − 8𝑥 3 − 5𝑥 2 − 16𝑥 + 40


a) Calcular la raíz racional
b) calcular las raíces irracionales con un error menor que 0.01
Sugerencia: Bosquejar el grafico del polinomio.
Solución:
a) Usando el método de las división sintética, tenemos que:

𝑓(𝑥) = (2𝑥 − 5)(𝑥 4 + 2𝑥 3 − 𝑥 2 − 8), siendo la raíz racional 𝑥 = 5/2


b) Analizamos los ceros o raíces de 𝑔(𝑥) = 𝑥 4 + 2𝑥 3 + 𝑥 2 − 8. Asi:

𝑔´ (𝑥) = 2𝑥(2𝑥 + 1)(𝑥 + 1)

Los puntos críticos son −1/2; −1 𝑦 0.

Vemos que 𝑔´ (𝑥) varía de acuerdo al siquiente esquema:

Luego:
𝑔(𝑥) Es decreciente para 𝑥 < −1 } → 𝐸𝑛 𝑥 = −1 hay un mínimo
𝑔(𝑥) Es creciente para −1 < 𝑥 < −1/2} → 𝐸𝑛 = −1/2 hay un
máximo
−1/2 < 𝑥 < 0
𝑔(𝑥) Es decreciente para } → 𝐸𝑛 𝑥 = 0 hay un
mínimo
𝑔(𝑥) Es creciente para 𝑥 > 0} → 𝐸𝑛 𝑥 = 0 hay un mínimo
Para:
1 1 1
𝑥 = −1 → 𝑔(−1) = −8 ; 𝑥 = → 𝑔 (− ) = −8 + ; 𝑥 = 0 → 𝑔(0)
2 2 16
= −8
Con lo que la grafica aproximada es la siguiente:

De donde vemos que 𝑔(𝑥) Solo tiene 2 raíces reales:


𝑥 = 𝑎 ; 𝑥 = 𝑏 (Irracionales, siendo las otras 2 raices complejas).

Calculo de la raíz. 𝑎 ∈ 〈−∞ ; −1〉:


𝑔(−3) = 28 ; 𝑔(−2) = −4
El cambio del signo implica que 𝑎 ∈ 〈−3 ; −2〉:
𝑔(𝑥𝑛 )
Usando el método de Newton de aproximaciones: 𝑥𝑛+1 = 𝑥𝑛 − ;
𝑔´ (𝑥𝑛 )

para lo cual tomamos como primera aproximación 𝑥0 = −2,5


Encontramos que 𝑎 = −2,25 es una raíz de 𝑔(𝑥) y por lo tanto de 𝑓(𝑥),
con
Un error menor que 0.01.
Calculo de la raíz irracional 𝑏 ∈ 〈0 ; ∞〉.
𝑔(1) = −4 ; 𝑔(2) = 28 → 𝑏 ∈ 〈1 ; 2〉.
Tomando 𝑥0 = 1,5 como primera aproximación a la raíz y aplicando el
método de Newton, obtenemos 𝑏 = 1,25 con error menor que 0.01.
PROBLEMA 73. Dada la ecuación:
16 2 2 4
(10/3)𝑥 5 + 7𝑥 4 + ( )𝑥 + ( )𝑥 − ( ) = 0
3 3 3
a) Hallar todas las raíces racionales.
b) Usar el método de Newton para calcular las raíces irracionales
con un error o igual a 0.0001.

Solución:
La ecuación es equivalente a :
𝑓(𝑥) = 10𝑥 5 + 21𝑥 4 + 16𝑥 2 + 2𝑥 − 4 = 0
Aplicampos el método de la división sintética y tenemos:
𝑓(𝑥) = (5𝑥 − 2)(2𝑥 + 1)(𝑥 3 + 2𝑥 2 + 2)
a) Las raíces racionales son 2/5 y −1/2
b) Las raíces irracionales de 𝑓(𝑥) son las de 𝑔(𝑥) = 𝑥 3 + 2𝑥 2 + 2 ,
𝑔(𝑥) es un factor de 𝑓(𝑥) que por lo menos tiene una raíz
irracional.

Graficaremos 𝑔(𝑥) para decidir si es la única raíz irracional.

𝑔´ (𝑥) = 𝑥(3𝑥 + 4) → puntos críticos −4/3 𝑦 0

Si 𝑥 ∈ 〈−∞; −4/3〉 ; 𝑔´ (𝑥) > 0; 𝑔(𝑥) creciente


4
Si 𝑥 ∈ 〈− ; 0〉 ; 𝑔´ (𝑥) < 0; 𝑔(𝑥) decreciente
3

Si 𝑥 ∈ 〈0; −∞〉 ; 𝑔´ (𝑥) > 0; 𝑔(𝑥) creciente


Entonces, existe un número máximo en −4/3 , 𝑔(−4/3) = 86/27
Existe un mínimo en
𝑥 = 0 𝑦 𝑔(0) = 2
Luego, la grafica es:
Según esta grafica, 𝑔(𝑥) tiene una única raíz irracional 𝑎 ∈ 〈−3; −2〉
𝑔(−2) = 2 > 0 ; 𝑔(−3) = −7 → 𝑎 ∈ 〈−3; −2〉
Usando el método de Newton y eligiendo 𝑥0 = −3 ,concluimos que 𝑎 =
−2,3593 es la
Raíz irracional con un error menor que 0.0001.
PROBLEMA 74. Encontramos una raíz quinta real de −𝜋 con un error
no mayor que 0.05
Solucion:
5
Sea 𝑥 = √−𝜋 → 𝑥 5 = −𝜋 → 𝑥 5 + 𝜋 = 0
Por otro lado, sea:
𝑃(𝑥) = 𝑥 5 + 𝜋 𝑦 𝑝¨ (𝑥) = 5𝑥 4
Aplicando el método de Newton- Raphson:
𝑝(𝑥𝑛 ) 𝑥𝑛5 + 𝜋 4 𝜋
𝑥𝑛+1 = 𝑥𝑛 − , = 𝑥𝑛 − = [𝑥𝑛 − ]
𝑝 (𝑥𝑛 ) 5𝑥𝑛4 5 4𝑥𝑛4
Eligiendo 𝑥0 = 1 ,obtenemos:
𝑥1 = −1,4283184 ; 𝑥2 = −1,29936208 ; 𝑥3 = −1,25926 ; 𝑥4 =
−1,2572804
Luego 𝑥 = −1,26 sera la raíz pedida con la aproximación deseada.

PROBLEMA75.
Interpretar geométricamente: 𝑧1 − 𝑧2 + 𝑧3 − 𝑧4 = 0
Solucion:
La igualdad se puede expresar como:
𝑧1 − 𝑧2 = 𝑧4 − 𝑧3 → |𝑧1 − 𝑧2 | = |𝑧4 − 𝑧3 | ….(1)
𝑧1 − 𝑧4 = 𝑧2 − 𝑧3 → |𝑧1 − 𝑧4 | = |𝑧2 − 𝑧3 | .…(2)

Al interpretar geométricamente (1) y (2) como la distancia entre dos


puntos, osea entre dos complejos, vemos que los lados del
cuadrilátero formado por 𝑧1 ; 𝑧2 ; 𝑧3 𝑦 𝑧4 son
Iguales dos a dos y los segmentos dirigidos al ser iguales, son
paralelos. (Notese que las propiedades geométricas de los vectores se
siguen cumpliendo en los números complejos). Luego, el cuadrilátero
es un paralelogramo.
Ademas se tiene:
𝑧1 +𝑧3 𝑧2 +𝑧4
= , lo cual indica que las diagonales se cortan en su punto
2 2
medio.
(ver figura).
𝑧1 +𝑧3 1 𝑧1 +𝑧2 𝑧3 +𝑧4
= ( + ) , lo cual indica que el punto de las diagonales
2 2 2 2
es el punto medio del segmento que une los puntos medios de dos
lados opuestos.

PROBLEMAS PROPUESTOS

FORMA CARTESIANA DE UN NÚMERO COMPLEJO – DIFERENCIA Y


COCIENTE
1. Simplificar:
7−𝑖 3−i 5−2i
a) ∙ ∙
3+i 7+𝑖 5+2𝑖

5+3i
b) z = 1−i
4+i 2i
4−i+
3−i

3
2 𝑖
2−𝑖 𝑖
c) 𝑧 = [[( )] ]
3+𝑖

d) si 𝑧 = 2 − 3𝑖; 𝑤 = 4 + 𝑖; 𝑢 =3- 2𝑖,


hallar:
z−w
d.1) v1 =
z−u

𝑧 2 −𝑤 2 −𝑢2
d.2) 𝑣2 =
𝑧 2 +𝑤 2 +𝑢2

𝑢2 −𝑧 2 𝑢−𝑤
d.3) 𝑣3 = ∙
𝑢+𝑤 𝑧−𝑤
𝑢−𝑤−𝑧
d.4) 𝑣4 =
𝑤−𝑢−𝑧
1−𝑖
e) 𝑧̅ = (3 − 𝑖) + 3
2+𝑖+ 𝑖

−3𝑖
1−𝑖 2𝑖
f) 𝑧 = [( ) ]
1+𝑖

g) [1 − 𝑅𝑒(𝑧) + 𝑖𝐼𝑚(𝑧)][1 − 𝑅𝑒(𝑧) − 𝑖𝐼𝑚(𝑧)]

2. Encontrar:

a) Re(z) e Im(z) para la parte (b) del problema anterior:

b) Re(z) e Im(z) para la parte (c) del problema anterior.

c) Re(𝑣1 ) para la parte (d.1)del


problema anterior.

d) Im(𝑣2 ) para la parte (d.2)del problema anterior.

e) Re(𝑣3 )− 𝐼𝑚(𝑣4 ) para las partes (d.3) y (d.4) del problema anterior.

f) Re(z) e Im(z) para la parte (e) del problema anterior.

g) Re(𝑧̅) e Im(𝑧̅) para la parte (f) del problema anterior.

3. Hallar todos los z que satisfacen la relación:

𝑧. (1 + 𝑎𝑖) = 1 − 𝑎𝑖
4. demostrar que:
𝑧1 𝑧
a) 𝑅𝑒 (𝑧 )+𝑅𝑒 (𝑧+𝑧2 ) = 1
1 +𝑧2 2

𝑧1 𝑧2
b) 𝐼𝑚 (𝑧 ) + 𝐼𝑚 (𝑧 )=0
1 +𝑧2 1 +𝑧2

1
c) 𝑅𝑒(𝑧1 𝑧̅2 ) = 2 (𝑧1 𝑧̅2 − 𝑧̅1 𝑧2 )
1
d) 𝐼𝑚(𝑧1 𝑧̅2 ) = (𝑧1 𝑧̅2 − 𝑧̅1 𝑧2 )
2𝑖

MÓDULO DE UN NÚMERO COMPLEJO CONJUNTOS ESPECIALES


INTERPRETACION GEOMETRICA.

5. Hallar los 𝑧 = 𝑥 + 𝑖𝑦 que satisfacen la condición dada:

a) |𝑧| = 𝑅𝑒(𝑧) + 1

b) |𝑧 − 3𝑖| − |𝑧 + 2𝑖 | < 9

c) 𝑅𝑒(𝑧) + 𝐼𝑚(𝑧) < 1

d) |𝑧 + 2| − |𝑧 − 2| > 5

e) |𝑧 − 3| − |𝑧 − 4| ≤ 5

f) |1 + 𝑧 2 | < |2𝑧|

6. Demostrar que:
|𝑧 |−|𝑧2 | 𝑧1 −𝑧2
| |𝑧1 |
| ≤ 1 . Probar que: | |≤1
1 +𝑧2 1−𝑧̅1 𝑧2

¿En que caso se cumple la igualdad?

8. si |𝑧1 | < 1 y |𝑧2 | < 1 . Probar que:


|𝑧1 + 𝑧2 | < |1 + 𝑧̅1 𝑧2 |

9. Demostrar que:

|1 − 𝑧̅1 𝑧2 |2 -|𝑧1 − 𝑧2 |2 =(1 − |𝑧2 |2 )(1 − |𝑧2 |2 )

10. Sean 𝑧1 ; 𝑧2 y sea 𝑢2 = 𝑧1 𝑧2 Probar que:


𝑧1 + 𝑧2 𝑧1 + 𝑧2
|𝑧1 | + |𝑧2 | = | − 𝑢| + | + 𝑢|
2 2
11. Si 𝑢(𝑧1 𝑧2 )2 . Demuestre que:
𝑧1 + 𝑧2 𝑧1 + 𝑧2
|𝑧1 | + |𝑧2 | = | |+| + 𝑢|
𝑧2 𝑧1
12. Demostrar que:

𝑧1 |𝑧1 |
| |≤ ; 𝑧2 ≠ ±𝑧3
𝑧2 + 𝑧3 ||𝑧2 | − |𝑧3 ||

13. Demostrar que:


1 𝑧1 𝑧2
|𝑧1 + 𝑧2 | ≥ (|𝑧1 | + |𝑧2 |) | + |
2 |𝑧1 | |𝑧2 |

14. Probar que:

||𝑧1 |𝑧2 + |𝑧2 |𝑧1 | 2|𝑧1 𝑧2 |



𝑧1 + 𝑧2 |𝑧1 | + |𝑧2 |

15. Demostrar que:


|𝑧0 + 𝑧1 + ⋯ + 𝑧𝑛 | ≥ |𝑧0 | − |𝑧1 | − |𝑧2 | − ⋯ − |𝑧𝑛 |

16. Si: 𝑧1 + 𝑧2 + 𝑧3 + 𝑧4 = 0 y
|𝑧1 | = |𝑧2 | = |𝑧3 | = |𝑧4 |

Demostrar que 𝑧1 ; 𝑧2 ; 𝑧3 𝑦 𝑧4 son los vértices de un rectángulo o coinciden dos a


dos.
Sugerencias: Ver miscelánea de ejercicios resueltos.

17. Hallar la condición para que 3 puntos 𝑧1 ; 𝑧2 𝑦𝑧3 , distintos dos a dos, estén sobre
una misma recta (“sean colineales”).

18. Hallar condición para que 4 puntos 𝑧1 ; 𝑧2 ; 𝑧3 𝑦𝑧4 , distintos dos a dos, estén
sobre una circunferencia o una recta.
19. Demostrar que si:

𝑧12 + 𝑧22 + 𝑧32 = 𝑧2 𝑧3 + 𝑧3 𝑧1 + 𝑧1 𝑧2 ,


Entonces 𝑧1 ; 𝑧2 𝑦 𝑧3 forman un triangulo equilátero.
20. Los puntos representados por 𝑧1 ; 𝑧2 𝑦 0 Son no colineales. Demostrar que
todo 𝑧 ∈ ℂ, se tiene:

𝑧 = 𝑟𝑧1 + 𝑠𝑧2 𝑦 𝑟 𝑦 𝑠 ∈ ℝ y esta representación es única.

21. Sea a y b 2 constantes complejas. Si b≠0, probar que: 𝑎 + 𝑎̅ + 𝑏𝑧̅ + 𝑏̅𝑧 = 0 es


la ecuación de una recta, donde z es un número complejo variable.

22. Sea a, b y c constantes complejas, z una variable compleja. Probar que:

𝑎 + 𝑎̅ + 𝑏𝑧̅ + 𝑏̅𝑧 + (𝑐 + 𝑐̅)𝑧𝑧̅ = 0


Es la ecuación de una circunferencia o un caso degenerado de circunferencia.

FORMA POLAR DE UN NÚMERO COMPLEJO EXPONENCIAL


COMPLEJA.
23. Sean 𝑧1 ; 𝑧2 𝑦 𝑧3 los vértices de un triangulo y 𝑝1 ; 𝑝2 𝑦 𝑝3 números
reales positivos tal que 𝑝1 + 𝑝2 + 𝑝3 = 1 .
a) Probar que 𝑧 = 𝑝1 𝑧1 + 𝑝2 𝑧2 + 𝑝3 𝑧3 esta en el interior o en la frontera
del triangulo.
b) ¿Dónde esta z, si 𝑝1 = 𝑝2 = 𝑝3
c) En forma análoga, si z es un punto exterior de un cuadrilátero
convexo con vértices 𝑧1 ; 𝑧2 ; 𝑧3 𝑦 𝑧4 ,¿Qué condición debe cumplir z?
24. Hallar la forma polar de los números complejos del problema
1.
25. Simplificar:
a) (1⁄2 + 𝑖 ⁄2)16
6
b) (√3 + 𝑖)
1+𝑖 12
c) [ ]
√3+𝑖

(𝟑+𝟑𝒊)𝒏−𝟑
d)
(𝟑−𝟑𝒊)𝒏

26. Hallar:
𝑛 𝑛
a)𝑅𝑒(𝑒 𝑖𝑧 ), 𝑛 ∈ ℤ b)𝐼𝑚(𝑒 𝑖𝑧 )

27. Hallar z, tal que: |𝑒 𝑖𝑧 | < 1.

28. Sea 𝑧1 = |𝑧1 |𝑒 𝑖∝ 𝑦 𝑧2 = |𝑧2 |𝑒 𝑖𝛽 . Si se sabe que:𝑧̅1 𝑧2 =


|𝑧1 ||𝑧2 |𝑒 −𝑖(𝛼−𝛽) ,de donde: 𝑅𝑒(𝑧̅1 𝑧2 ) = |𝑧1 ||𝑧2 | cos(𝛽 − 𝛼) … (1)
𝐼𝑚(𝑧̅1 𝑧2 ) = |𝑧1 ||𝑧2 | sen(𝛽 − 𝛼) … (2)

La expresión (1) representa el “producto escalar” de los vectores


positivos 𝑧1 𝑦 𝑧2 , y la expresión (2) representa el “producto vectorial”
de 𝑧1 𝑦 𝑧2
Demostrar que:
a) 𝑅𝑒(𝑧̅1 𝑧2 ) = 𝑅𝑒(𝑧̅2 𝑧1 ) (ley conmutativa del “producto escalar”).
b) 𝑅𝑒(𝑧̅1 𝑧1 ) = |𝑧1 |2 ; 𝑅𝑒(𝑧̅1 𝑧1 ) = 0 ⇔ 𝑧1 = 0
c) 𝑅𝑒[𝑧̅1 (𝑧2 + 𝑧3 )] = 𝑅𝑒(𝑧̅1 𝑧2 ) + 𝑅𝑒(𝑧̅1 𝑧3 ) (ley distributiva del “producto escalar”)

d) |𝑅𝑒(𝑧̅1 𝑧2 )| ≤ |𝑧1 ||𝑧2 | (Desigualdad de Schwarz).


e) 𝐼𝑚(𝑧̅1 𝑧2 ) = −𝐼𝑚(𝑧̅2 𝑧1 ) (ley anticonmutativa del “producto vectorial”).

29. Sea 𝑧 = 𝑥 + 𝑖𝑦 = 𝑟𝑒 𝑖𝜃 . Demostrar:

a) 𝑟 𝑛 cos 𝑛𝜃 = 𝑥 𝑛 − (𝑛2)𝑥 𝑛−2 𝑦 2 + (𝑛4)𝑥 𝑛−4 𝑦 4 …

b) 𝑟 𝑛 sin 𝑛𝜃 = (𝑛1)𝑥 𝑛−1 𝑦 − (𝑛3)𝑥 𝑛−3 𝑦 3 + ⋯


RAICES DE NUMEROS COMPLEJOS

30. Sea 𝑧 = 2 , hallar:


3
a) (𝑧 1⁄6 ) b) (𝑧 3 )1⁄6
31. Hallar todos los posibles valores de 𝑧 2⁄4 , si:
a) 𝒛 = 𝟏 b) 𝒛 = 𝟏 + 𝒊 c) 𝒛 = 𝒊
32. Hallar las soluciones de:
1+𝑖 1−𝑖
𝑧6 − ( ) 𝑧3 − ( ) = 𝑧 3𝑖
√2 √2

33. Demuestre que la ecuación:


𝑧𝑧̅ + 2𝑘|𝑧| + 𝑘 2 = 0

Tiene soluciones si y solo si k es un numero real negativo. Diga en que


casos no existe solución; en caso que exista solución, ¿Cuántas
existen? .Interprételo geométricamente.
34. Supóngase que un reloj se coloca en una circunferencia de radio
unidad, de manera que el centro del reloj coincida con el origen de
los ejes coordenados, y que cuando el horario marca z el minutero
marca 𝑧 12 , si z ≠ 0.Hallar los z para los cuales se cumplen:
a) las manecillas señalan la misma dirección, es decir: z=𝑧 12 (cos 0 +
𝑖 sen 0).o también 𝑧 = 𝑧 12 . O lo que es lo mismo: 𝑧 11 − 1 = 0.
b) las manecillas señalan direcciones opuestas.
c) las manecillas sean perpendiculares entre si.
d) las manecillas forman entre si un ángulo igual al cualquier múltiplo
de 90°.
𝑞 𝑝 𝑞
35. sean p y q dos números enteros. Probar que ( √𝑧) 𝑦 √𝑧 𝑝 tienen
los mismos valo-res si y sólo si p y q son primos entre sí:
36. Usar el problema anterior y para 𝑧 = 𝑖, hallar:
3
a)(𝑧 1⁄2 ) b) (𝑧 3 )1⁄2
37. Hallar todos los z tales que:
(𝑧̅)2 = 𝑧 𝑛−1 ; 𝑛 ∈ ℤ ; 𝑛 ≠ 3

38. probar que si 𝑧0 es la raíz cubica de z y si 1, 𝑤 𝑦 𝑤 2 son las raíces


cúbicas de la unidad, entonces 𝑧0 , 𝑧0 𝑤 𝑦 𝑧0 𝑤 2 con las tres raíces
cúbicas de z. ¿Se puede generalizar el resultado para raíces
mayores que la cúbica?

39. Usar el problema anterior y para 𝑧 = −27 hallar sus raíces cúbicas.
40. Hállese los vértices de un polígono regular de n lados, si su centro
se halla en el punto 𝑧 = 0 y se conoce uno de sus vértices 𝑧1 .
ECUACIONES POLINOMIALES
41. resolver la ecuación:
9𝑥 8 + 33𝑥 7 − 89𝑥 6 − 339𝑥 5 − 3𝑥 4 +123x 3 + 21x 2 − 9x − 2=0

Sabiendo que admite la raíz 𝑥 = √2 + √3


Sugerencia: calcular, primeramente, las raíces racionales.
42. calcular la raíz racional de:

𝑝(𝑥) = 12𝑥 5 − 56𝑥 4 + 105𝑥 3 − 66𝑥 2 −27𝑥 + 20 = 0

43. Hallar la raíz real de:

𝑝(𝑥) = 𝑥 4 + 2𝑥 3 + 𝑥 2 − 8 ,

Con un error menor que 0.01

44. calcular las raíces de las siguientes ecuaciones: a) x 3 −


x 2 − 2x + 1 = 0 b) 3𝑥 3 + 4x 2 − 8x − 1 = 0 c) x 4 x 3 −
4x 2 − 3x + 3 = 0 d) x 4 − 4, 4x 3 + 9, 43x 2 − 14,86x + 7,15 = 0
e) 3x 3 + x 𝟐 − 4x + 2 = 0 f) x 5 − x 3 − 2x 2 + 1 = 0 g)
4 2 3 2
x + 5x + x − 3 = 0 h) 5x − 3x + x − 1 = 0 i)
3x 5 + 2x 4 − 3x − 2 = 0 j) 3x 4 + 5x 2 − 8x + 2 = 0 k)
5 3 5 2
2x − 3x + 4x − 2 = 0 l) 2x − x + x − 5 = 0

45. calcular las raíces de la ecuación:

𝑥 3 − 6𝑖𝑥 + 4(1 − 𝑖) = 0

46. sea:
𝑛−1
𝑝(𝑧) = 𝑎𝑛 𝑧 𝑛 + 𝑎𝑛 − 1𝑧 + ⋯ + 𝑎1 𝑧 + 𝑎0 y Sea 𝑎𝑛 ≠ 0 ≠ 𝑎0
n 1
 an 1 x  ...  a1 x  a0  0
n
an x
tiene una solución positiva r.

a) P(z) no se anula en el circulo z  r

47. Si los coeficientes de la ecuación:

P( z )  an z n  an 1 z n 1  ...  a1 z  a0  0
son positivos y no crecientes; es decir,

0  an  an1  an2  ...  a1  0

entonces la ecuación no tiene raíz en el círculo z  1 .

48. Dado el sistema complejo:

a11 z1  a11 z2  ...  a1n zn  0


a21 z1  a21 z2  ...  a2 n zn  0

an1 z1  an1 z2  ...  ann zn  0

donde, a jk  x jk  iy jk ; zk  ck  id k
Probar que este sistema tiene una solución distinta a la trivial, si el
determinante:
a11 z1  a11 z2  ...  a1n zn  0
a21 z1  a21 z2  ...  a2 n zn  0
0

an1 z1  an1 z2  ...  ann zn  0

Y viceversa.

PROBLEMAS VARIOS

49.

a) Expresar cos nx en función de cos x , para


n  2;3;5;7

b) Expresar sin nx en función de sin x , para


n  3;5; 7

sin nx
b) Expresar en función de cos x , para
sin x
n  2;3; 4

c) Expresar  cos x  como combinación lineal de


n

1, cos x;cos 2 x;...;cos nx , para


n  2;3;5
d) Expresar  sin x  como combinación lineal de
n

sin x;sin 2 x;...;sin nx , para


n  2;3;5

50. Probar que:


a)
cos  x  y  z   cos x cos y cos z  cos x sin y sin z  cos y sin x sin z  cos z sin x sin y
b) Hallar una expresión similar para:

sin  x  y  z 

51. Probar la desigualdad triangular en el espacio n dimensional

 x1  y1   ...   xn  yn   x12  x22  ...  xn2  y12  y22  ...  yn2


2 2

52. Hallar el limite, si existe, de las siguientes sucesiones:

a) 1; i; 1; i;1; i; 1; i;...

b) 1; i 2 ; 13 ; i 4 ; 15 ; i 6 ; 1 7 ; i 8 ; 19 ;...

1 i  1 i   1 i 
2 3

c) ;  ;  ;...
2  2   2 

53. Demostrar que, para todo s  n se cumple:


n 2 n 2 s 1 2

a)  n  2  ak   ak   ak  as
k 1 k 1 k 1

n 2 n 2 s 1 2

b) n ak   ak   ak  as
k 1 k 1 k 1

Você também pode gostar